Vinay Kumar Differential Calculus For JEE Main and Advanced PDF

You might also like

Download as pdf or txt
Download as pdf or txt
You are on page 1of 139

1

CHAPTER

Limits

We denote neighbourhoods by N(a), N1(a), N2(a), etc.


1.1 Introduction
Since a neighbourhood N(a) is an open interval symmetric
Let us introduce the notion of limit of a function which plays about a, it consists of all real x satisfying a – δ < x < a + δ for
an important role in mathematical analysis and the study some δ > 0. The positive number δ is called the radius of the
ps
of calculus. The concept of limit of a function is one of the neighbourhood.
el
fundamental ideas that distinguishes calculus from algebra
We designate N(a) by N(a, δ) if we wish to specify its radius.
and trigonometry.
eh

The inequalities a – δ < x < a + δ are equivalent to –δ < x – a


We use limits to describe the way a function f varies. Some < δ, and to |x – a| < δ. Thus, N(a, δ) consists of all points x
je

functions vary continuously i.e. small changes in x produce whose distance from a is less than δ.
iit

only small changes in f(x). Other functions can have values


that jump or vary erratically. Meaning of x → a
@

Sometimes we face with problems whose solutions involve the The symbol x → a is called as 'x tends to a' or 'x approaches a'.
use of limits. According to a formula of geometry, the area of It implies that x takes values closer and closer to 'a' but not 'a'.
a circle of radius r is πr2. How is such a formula derived? The
usual way is to inscribe regular polygons in the circle, find
the areas of these polygons, and then determine the “limiting
value” of these areas as the number of sides of the polygons Sometimes we need to consider values of x approaching 'a'
increase without bound. Thus, even such seemingly simple from only one side of 'a'.
formula as that for the area of a circle depends on the concept If x approaches 'a' from the left of 'a' then we use the symbol:
of limit for its derivation. x → a– or x → a – 0.
We also use limits to define tangent to graphs of functions. Similarly, if x approaches 'a' from the right of 'a' then we use
This geometrical application leads to the important concept of the symbol : x → a+ or x → a + 0.
derivative of a function, which quantifies the way a function's
value changes.
The need for evaluating the limit of a function arises in science
and engineering when we come across situations where a Limit of a Function
function (denoting a physical quantity) is not defined at x = a, Let us consider a function y = f(x) of a continuous variable x.
however the value of the function as x takes values very very Suppose that the independent variable x approaches a number
close to ‘a’ symbolize a useful physical quantity, for example 'a'. This means that x is made to assume values which become
instantaneous velocity, acceleration etc. arbitrarily close to 'a' but are not equal to 'a'. To describe such a
situation we say that x tends to 'a' or x approaches 'a' and write
Neighbourhood of a Point x → a. If there is a number  such that as x approaches 'a',
Any open interval containing a point a as its midpoint is called either from the right or from the left, f(x) approaches , then
a neighbourhood of a.  is called the limit of f(x) as x approaches 'a'.

FREE BOOKS FOR JEE & NEET =>(@iitjeeadv)


1.2 Differential Calculus for JEE Main and Advanced

Informal Definition of Limit differing by little from zero. It is not a statement about the value
Let f(x) be defined on an open interval about 'a', except possibly of f(x) when x = 0. When we make the statements we assert
at 'a' itself. If f(x) gets arbitrarily close to  for all x sufficiently that, when x is nearly equal to zero, f(x) is nearly equal to . We
close to 'a', we say that f approaches the limit  as x approaches assert nothing about what happens when x actually equal to 0.
'a', and we write
lim f ( x )  . Caution
x a

(the abbreviation "lim" means "limit")


A wrong statement about limits :
It is evident that if a function has a limit for x → a, this limit is The number  is the limit of f(x) as x approaches a if f(x) gets
unique, since the values of the function corresponding to the closer to  as x approaches a.
values of x approaching 'a' must become arbitrarily close to Consider f(x) = x + [x] +1 at x = 0.
a constant and hence cannot be simultaneously close to more As x approaches 0, the function f(x) gets closer to
than one constant number. 1/2, from both sides but 1/2 is not the limit because f(x) does
An alternative notation for lim f ( x )   is f(x) →  as x → a not get arbitrarily close to 1/2. For example the function cannot
x a
attain 0.4995 or 0.5002 by using x sufficiently close to 0.
which is usually read as "f(x) approaches  as x approaches a".
Suppose we are asked to sketch the graph of the function f
Study Tip x3 − 1
given by f(x) = , x ≠ 1.
x −1
1. A number  is said to be a limiting value only if it is finite
and real, otherwise we say that the limit does not exist or For all values other than x = 1, we can use standard curve-
sketching techniques. However, at x = 1, it is not clear what
ps
dne (for brevity).
to do. To get an idea of the behaviour of the graph of f near
2. Note that 'a' need not be in the domain of f.
el
x = 1, we can use two sets of x-values – one set that approaches
Even if 'a' happens to be in the domain of f, the
eh

1 from the left and one set that approaches 1 from the right, as
value f(a) plays no role in determining whether shown in table.
lim f ( x )  . It is only the behaviour of f(x) for x
je

x a
iit

near a that concerns us.


3. We also note that the three statements,
@

lim f(x) = , lim (f(x) – ) = 0, lim | f(x) –  | = 0, are all


x →a x →a x →a
equivalent.
x
Let us consider the function f(x) = .
x
This function is equal to 1 for all values of x except x = 0.
It is not equal to 1 when x = 0; it is in fact not defined at x = 0. When we plot these points, it appears that the graph of f is a
parabola that has a hole at point (1, 3), as shown in the figure.
For when we say that f(x) is defined for x = 0 we mean that we
Although x cannot equal 1, we can move sufficiently close to
can calculate its value for x = 0 by putting x = 0 in the formula
1, and as a result f(x) moves arbitrarily close to 3.
which defines f(x). In this case we cannot. When we put x = 0
in f(x) we get 0/0, which is meaningless. Using limit notation, we write lim f(x) = 3.
x→1
x
Thus f(x) = is a function which differs from y = 1 solely in It is read as "the limit of f(x) as x approaches 1 is 3."
x
that it is not defined for x = 0.
x
We have lim
x→0
= 1, since x/x is equal to 1 so long as x differs
x
from zero, however small the difference may be.
On the other hand there is of course nothing to prevent the
limit of f(x) as x tends to zero from being equal to f(0), the
value of f(x) for x = 0.
Thus if f(x) = x, then f(0) = 0 and lim
x →0
f ( x ) = 0.

Note: That the statement lim f ( x ) =  is a statement


x →0
about the values of f(x) when x has any value distinct from but

FREE BOOKS FOR JEE & NEET =>(@iitjeeadv)


Limits 1.3

x 2 − 3x + 2 x3 − 1 ( x 2 + x + 1)( x − 1) x2 + x + 1
Example 1: Evaluate lim = = .
x→2 x−2 x2 −1 ( x + 1)( x − 1) x +1
Solution: Consider the function f defined by So the behaviour of (x3–1)/(x2–1) for x near 1, but not equal to
x 2  3x  2 x2 + x + 1
f(x) = ,x≠2 1, is the same as the behaviour of .
x +1
x2
The domain of f is the set of all real numbers except 2, which x3 − 1 x2 + x + 1 .
lim = lim
has been excluded because substitution of x = 2 in the expression x →1 x 2
−1 x →1 x +1
0
for f(x) yields the undefined term .
0 Now, as x approaches 1, x2 + x + 1 approaches 3 and x + 1
On the other hand, x 2 – 3x + 2 = (x – 1) (x – 2) and
( x − 1)( x − 2) x2 + x + 1 3
f(x) = = x – 1, provided x ≠ 2
approaches 2. Thus, lim = , from which it
x−2
x →1 x +1 2
....(1) x3 − 1 3
The graph of the function y = x – 1 is a straight line L; so the follows that lim = .
x →1 x 2 − 1 2
graph of f(x) is the line L with a hole at the point (2, 1).
We can also express this as follows:
x3 − 1 3
As x → 1, 2 → .
x −1 2
Example 3: Find the limit of the function
ps
x + 1 , x < 0
f (x) =  as x→ 0.
el

2 − x , x ≥ 0
eh

Solution: Using the graph of y = f(x), we see that the


je

function approaches 1 as x approaches 0 from the left of 0. Also


the function approaches 2 as x approaches 0 from the right of 0.
iit
@

Although the function f is not defined at x = 2, we know its


behaviour from values of x near 2. The graph makes it clear
that if x is close to 2, then f(x) is close to 1. In fact, the values of
f(x) can be brought arbitrarily close to 1 by taking x sufficiently
close to 2.
x 2 − 3x + 2
We express this fact by writing lim = 1, which
x→2 x−2
Since the function does not approach the same level from both
x 2  3x  2
means that the limit of is 1 as x approaches 2. sides of x = 0, lim f ( x ) does not exist.
x2 x →0
x3 −1
Example 2: Let f(x) = 2
x −1
. How does f(x) behave
Formal Definition of Limit
when x is near 1 but is not 1 itself ? The conceptual problems in trying to give an exact meaning
Solution: There are two influences acting on the fraction to the expression lim f(x) = b revolve around phrases such as
x →a
(x3 – 1)/(x2 – 1) when x is near 1. On the one hand, the numerator “arbitrarily close,” “sufficiently near,” and “arbitrarily small.”
x3 –1 approaches 0; thus there is an influence pushing the After all, there is a no such thing in any absolute sense as a small
fraction towards 0. On the other hand the denominator x2 –1 positive real number. The number 0.000001 is small in most
also approaches 0; division by a small number tends to make a contexts, but in comparison with 0.000000000001 it is huge.
fraction large. How do these two opposing influences balance However, we can assert that one number is smaller than another.
out?
Moreover, the actual closeness of one number x to another number
We rewrite the quotient (x3 – 1)/(x2 – 1) as follows: When x ≠ 1, a is just the distance between them : it is |x – a|. One way to say that
we have a function f takes on values arbitrarily close to a number  is to

FREE BOOKS FOR JEE & NEET =>(@iitjeeadv)


1.4 Differential Calculus for JEE Main and Advanced

state that, for any small positive real number ε, there are numbers In Figure 2 the vertical band shown is not narrow enough
x such that | f(x) –  | < ε. We are stating that no matter what to meet the challenge of the horizontal band shown. But the
positive number ε is selected, 10–7, or 10–17, or 10–127, there are vertical band shown in Figure 3 is sufficiently narrow.
numbers x so that the distance between f(x) and  is smaller
than ε.
ε − δ definition
Let f be a function defined on an open interval containing a
except possibly at a and let  be a real number. The statement
lim f(x) =  means that for each ε > 0 there exists a δ > 0 such
x →a
that if 0 < |x – a| < δ, then |f(x) – | < ε.
The inequality 0 < |x – a | that appears in the definition is just
a fancy way of saying “x is not a.” The inequality |x – a | < δ
asserts that x is within a distance δ of a. The two inequalities Figure 1
may be combined as the single statement 0 < |x – a| < δ,
which describes the open interval (a–δ, a + δ) from which a
is deleted.
Here we wish to say that f(x) is arbitrarily close to  whenever x
is sufficiently close, but not equal to a. What does “sufficiently
close” mean? “ The answer is this : If an arbitrary ε > 0 is
chosen with which to measure the distance between f(x) and ,
then it must be the case that there is a number δ > 0 such that
ps
whenever x is in the domain of f and within a distance δ of
el
a, but not equal to a then the distance between f(x) and  is
less than ε. Figure 2
eh

This is illustrated on the graph of the function y = f(x) as shown


je

in the figure. Since from the inequality |x – a| < δ there follows


the inequality |f(x) – l| < ε, this means that for all points x that
iit

are not more distant from the point a than δ, the points M of
@

the graph of the function y = f(x) lie within a band of width 2ε


bounded by the lines y = –ε and y =  + ε.

Figure 3

Example 4: Prove that lim (3x + 1) = 7.


x→2
Solution: Let an arbitrary ε > 0 be given. For the
inequality |(3x + 1) – 7| < ε to be fulfilled it is necessary to
have the following inequalities fulfilled :
First ε > 0 is chosen arbitrarily. There must then exist a number ε ε ε
|3x – 6| < ε, |x – 2| < , – <x–2<
δ > 0 such that whenever x lies in the interval (a – δ, a + δ), 3 3 3
and x ≠ a then the point (x, f(x)) lies in the shaded rectangle. Thus, given any ε, for all values of x satisfying the
In other words, ε is the challenge. The response is δ. Usually, ε
inequality |x – 2| < = δ, the value of the function 3x + 1 will
the smaller ε is, the smaller δ will have to be. The geometric 3
significance of the definition is shown in the Figure 1. The differ from 7 by less than ε. And this means that 7 is the limit
narrow horizontal band of width 2ε is again the challenge. of the function as x → 2.
The response is a sufficiently narrow vertical band, of width
Example 5: Examine the limit of the function
2δ, such that the part of the graph within that vertical band
(except perhaps at x = a) also lies in the challenging horizontal x2 −1
y= as x tends to 1.
band of width 2ε. x −1

FREE BOOKS FOR JEE & NEET =>(@iitjeeadv)


Limits 1.5

Solution: The function is defined for every value of x Example 6: Consider the function
x2 −1
other than 1 and y = = x + 1, when x≠1. 
 x + 1,
2
x≥0
x −1 g(x) =  lim g(x) ≠ 1.
. Prove that x→0
 −( x + 1), x < 0
2
Firstly consider the behaviour of the values of y for values of 
x greater than 1. Clearly, y is greater than 2 when x is greater
Solution To do this, we must establish the negation of
than 1.
the limit condition. There is an ε > 0 such that, for any δ > 0,
If, x, while remaining greater than 1 takes up values whose there is a number x in the domain of a g such that 0 < |x| < δ
difference from 1 constantly diminishes, then y, while and |g(x) – 1| ≥ ε. There are many possible choices for ε. To be
remaining greater than 2, takes up values whose difference 1
from 2 constantly diminishes also. specific, take ε = . We must now show that for every positive
2
In fact, difference between y and 2 can be made as small as we number δ, there is a nonzero number x in the open interval
like by taking x sufficiently near 1. (–δ, δ), such that the distance between g(x) and 1 is greater
1 δ
For instance, consider the number .001. Then than or equal to . See figure. Take x = – . This number is
2 2
|y – 2| = y – 2 = x + 1 – 2 < .001 ⇔ x < 1.001.
nonzero, lies in (–δ, δ), and furthermore
Thus, for every value of x which is greater than 1 and less than
1.001, the absolute value of the difference between y and 2 is
 δ δ2
g(x) = g  −  = – – 1 < – 1.
less than the number .001 which we had arbitrarily selected.  2 4
Instead of the particular number .001, we now consider any 1
Hence |g(x) – 1| > 2 ≥ .
small positive number ε. Then 2
ps
y – 2 = x – 1 < ε ⇔ x < 1 + ε.
el

Thus, there exists an interval (1, 1 + ε), such that the value of
eh

y, for any value of x in this interval, differs from 2 numerically,


by a number which is smaller than the positive number ε,
je

selected arbitrarily.
iit

Thus, the limit of y as x approaches 1 from the right is 2 and


@

we have lim y = 2.
x→1+
We now consider the behaviour of the values of y for values of
x less than 1. When x is less than 1, y is less than 2.
If, x, while remaining less than 1, takes up values whose
difference from 1 constantly diminishes, then y, while
remaining less than 2, takes up values whose difference from
2 constantly diminishes also. Notice in the definition that the hypothesis 0 < | x – a | simply
Let now, ε be any arbitrarily assigned positive number, however means x ≠ a. This condition releases the point of approach
small. We then have, a from the responsibility of having any image at all; and
even if there is an image f(a) the condition | f(a) –  | < ∈
|y – 2| = 2 – y = 2 – (x + 1) = 1 – x < ε is allowed to fail. Briefly, the idea of limit discounts what
so that for every value of x less than 1 but > 1 – ε, the happens precisely at a, but is vitally concerned with images of
absolute value of the difference between y and 2 is less than the "neighbours" of a.
the number ε. There is no practical need of applying ∈−δ definition
Thus, the limit of y, as x approaches 1, from the left is 2 and everywhere since it involves lengthy and complicated
lim y = 2.
we write x→ calculations. We shall later on derive some simple rules for
1 −
finding limits.
Combining the conclusions arrived at in the last two cases, we
see that corresponding to any arbitrarily assigned positive Example 7: Let the function f be defined as follows:
number ε, there exists an interval (1 – ε, 1 + ε) around 1, such  1
f   = 1 for every nonzero integer n,
that for every value of x in this interval, other than 1 where  n
the function is not defined, y differs from 2 numerically by a f(x) = x for every other real number x. Prove that
number which is less than ε, i.e., we have | y – 2 | < ε for lim f(x) does not exist.
any x, other than 1, such that |x – 1| < ε. x→0

FREE BOOKS FOR JEE & NEET =>(@iitjeeadv)


1.6 Differential Calculus for JEE Main and Advanced

Solution: The graph of f is the line y = x, with the There can, therefore, exist no number which differs from sin
exception that when x = 1/n, n a nonzero integer, y = 1. It is 1/x by a number less than an arbitrarily assigned positive
shown in the figure. number for values of x near 0. Hence lim (sin 1/x) does
x→0
Since f(0) = 0 and f(x) is close to 0 when x is close to 0, x ≠ not exist.
 2 1
1/n, we might suspect that lim f(x) = 0.  x sin , x ≠ 0
x→0 Example 9: f(x) =  x as x→ 0.

1 , x=0
Solution: The function oscillates between the graphs
of y = –x2 and y = x2. As x approaches closer and closer to
zero from either side, the two functions approach 0, and hence
f(x) → 0 as x→ 0.
Example 10: Find the limit of the function
x , x ∈Q
f (x) =  as x→ 2 and x→ 4.
 4 − x , x ∉Q
Solution: We draw the rough sketch of y = f(x).

However, this is not correct. For if we take the neighbourhood


N = (–1/2, 1/2) of 0, there is no deleted neighbourhood D = (-δ, 0)
∪ (0, δ) of 0 such that f(x) is in N for every x in D.
We can always find a positive integer n such that 1/n < δ and for
ps
this number f(1/n) is not in N since f(1/n) = 1. In a similar way,
el

we can show that lim f(x) is not equal to any other number.
eh

x→0 If x approaches 2 by taking rational values, then the function


Therefore, this limit does not exist. f(x) approaches 2 using y = x. And if x approaches 2 by taking
je

Example 8: Prove that lim (sin 1/x) does not exist. irrational values, then the function f(x) also approaches 2 using
iit

x→0
y = 4 – x. Hence, the limit of f(x) as x→ 2 is 2.
Solution: The graph of y = sin 1/x is drawn below.
@

lim f ( x ) = 2
x →2

However, if x approaches 4 by taking rational values, then the


function f(x) approaches 4 using y = x. And if x approaches 4
by taking irrational values, then the function f(x) approaches 0
using y = 4 – x. Since the function does not approach the same
level, the limit lim f ( x ) does not exist.
x →4
There are many such interesting functions that have unusual
The function oscillates between –1 and 1 more and more limit behaviour. An often cited one is the Dirichlet function
rapidly as x approaches closer and closer to zero from either side.
0, if x is rational
If we take any interval containing 0, however small it may be, f(x) = 
then for an infinite number of points of this interval the function 1, if x is irrational.
assumes the values 1 and –1. This function has no limit at any real number.

Practice Problems A

1. For the function graphed here, find the following limits:

FREE BOOKS FOR JEE & NEET =>(@iitjeeadv)


Limits 1.7

(i) lim f(x)



(ii) lim f(x) x if x is rational
x→2 x→1 5. Let f(x) = 
2. For the function graphed here, find the following limits: − x if x is not rational
(a) Does limx→1 f(x) exist ?
(b) Does limx→ f(x) exist ?
(c) Does limx→0 f(x) exist ?
(d) For which numbers a does limx→af(x) exist?

 x 2 if x is an irrational number
6. Let f(x) =  .
1 if x is an rational number
(i) lim f(x) (ii) lim f(x) (iii) lim f(x)
x→−2 x→0 x→−1
Prove that f(x) has a limit at the points x = 1 and
3. If lim f(x) = 5 must f be defined at x = 1 ? x = –1 and does not have a limit at other points.
x→1
If it is, must f(1) = 5 ? Can we conclude anything about 7. Find lim cot −1 x 2 .
the values of f at x = 1 ? Explain. x→0

4. If f(1) = 5, must lim f(x) exist ? If it does, then must lim sin 2 x
x→1 x→1 8. Find lim .
x→0 cos x
f(x) = 5? Can we conclude anything about lim f(x)?
x→1 9. Find lim cos π/x.
x→0
ps
el
eh

the corresponding values yn of the function become arbitrarily


1.2 Concept of Infinity close to the number .
je

Suppose that n assumes successively the values 1, 2, 3,.... Then Let us consider a function of an integral argument. Usually
iit

as n gets larger and larger and there is no limit to the extent such an argument is denoted by the letter n and the values of
@

of its increase. However, large a number we may think of, a the function by some other letter supplied with a subscript
time will come when n has become larger than this number. indicating the value of the integral argument. For instance, if
It is convenient to have a short phrase to express this unending y = f(n) is a function of the integral argument n we write yn =
growth of n and we shall say that n tends to infinity, or n → ∞, f(n). Given such a function, we say that the values
When we say that n ‘tends to ∞’ we mean simply that n is y1 = f(1), y2 = f(2), .... , yn = f(n), .....,
supposed to assume a series of values which increase beyond assumed by the function form a sequence.
all limit.
If there is a sequence y1, y2, y3, ...., this assigns, to every natural
There is no real number ‘infinity’. This implies that the equation
n = ∞ is meaningless. A number n cannot be equal to ∞, because number n, a value yn = f(n). For instance, the terms of the
‘equal to ∞’ means nothing. So far in fact the symbol ∞ means
1 1 1
nothing at all except in the one phrase ‘tends to ∞’, the meaning geometric progression , , .... are the subsequent values
2 4 8
of which we have explained above. Later on we shall learn how
1
to attach a meaning to other phrases involving the symbol ∞, of the function f(n) = .
but we bear in mind 2n
We now discuss the question ‘what properties has f(n) for
(i) that ∞, by itself means nothing, although phrases
sufficiently large values of n ?’ i.e. ‘how does f(n) behaves as
containing it sometimes mean something,
n tends to ∞ ?’
(ii) that in every case in which a phrase containing the symbol
It may occur that, as n increases, the values yn = f(n) become
∞ means something it will do so simply because we have
arbitrarily close to a number . Then we say that the number  is
previously attached a meaning to this particular phrase by
the limit of the function f(n) of the integral argument n or that the
means of a special definition.
sequence y1, y2, .... , yn, .... has the limit , as n → ∞, and write
Limit of a Sequence lim f (n )   or lim y n  
n n
Definition A number  is said to be the limit of the function
y = f(n) of the integral argument n or the limit of the sequence Consider the function 1/n for large values of n. Instead of
y1, y2, ...., yn, if for all sufficiently large integral values of n saying ‘1/n is small for large values of n’ we say ‘1/n tends

FREE BOOKS FOR JEE & NEET =>(@iitjeeadv)


1.8 Differential Calculus for JEE Main and Advanced

to 0 as n tends to ∞ . Similarly we say that ‘1– (1/n) tends to Note: The limit  may itself be one of the actual values
1 as n tends to ∞’. of f(n). Thus if f(n) = 0 for all values of n, it is obvious that
We shall say that ‘the limit of 1/n as n tends to ∞ is 0’, a lim f (n ) = 0.
n →∞
1
statement which we express as lim  0 . On the other hand the limit itself need not (and in general will
n  n
not) be the value of the function for any value of n. This is
We shall also sometimes write ‘1/n → 0 as n → ∞’ which may sufficiently obvious in the case f(n) = 1/n. The limit is zero;
be read ‘1/n tends to 0 as n tends to ∞’. but the function is never equal to zero for any value of n.
In the same way we shall write A limit is not a value of the function. It is something quite
 1 distinct from these values.
lim 1 −  = 1 or , 1 – (1/n) → 1 as n → ∞. For the function f(n) = 1 the limit is equal to all the values of
n →∞  n
f(n).
Now let us consider f(n) = n2. Then ‘n2 is large when n is large’. For f(n) = 1+ (1/n), it is not equal to any value of f(n). For f(n)
And it is natural in this case to say that ‘n2 tends to ∞ as n tends = (sin 12 nπ)/n, (whose limits as n tends to ∞ is easily seen to be
to ∞’, and we write n2 → ∞. 0, since sin 12 nπ is never numerically greater than 1), the limit
Finally consider the function f(n) = – n2. In this case f(n) is is equal to the value which f(n) assumes for all even values of
large, but negative, when n is large and we say that ‘– n2 tends n, but the values assumed for odd values of n are all different
to – ∞ as n tends to ∞’ and write – n2 → − ∞ . from the limit and from one another.
Definition The function f(n) is said to tend to the limit  Definition The function f(n) is said to tend to ∞ (positive
as n tends to ∞ , if, however small be the positive number ε, infinity) with n if, when any number M, however large, is
f(n) differs from  by less than ε for sufficiently large values assigned, we can determine N so that f(n) > M when n ≥ N;
ps
of n; that is to say if, however small be the positive number that is to say if, however large M may be, f(n) > M for sufficient
ε, we can determine a number N(ε) corresponding to ε, such large values of n.
el

that f(n) differs from  by less than ε for all values of n greater Another form of statement is, if we can make f(n) as large as
eh

than or equal to N(ε). we please by sufficiently increasing n.


The definition may be stated more shortly as follows : if, given A function may be always numerically very large when n is
je

any positive number, ε, however small, we can find N(ε) so very large without tending either to ∞ or to –∞. A sufficient
iit

that | f(n) –  | < ε when n ≥ N(ε), then we say that f(n) tends illustration of this is given by f(n) = (–1)nn.
to the limit  as n tends to ∞ , and write
@

Example 1: Find lim sin nθπ, if θ is rational.


n→∞
lim f (n ) =  Solution: Let f(n) = sin nθπ and θ = p/q, where p and q
n →∞
are positive integers. Let n = aq + b, here a is the quotient and
The definition of a limit may be illustrated geometrically b the remainder when n is divided by q.
as follows. The graph of f(n) consists of a number of points Then sin(npπ/q) = (–)ap sin(bpπ/q).
corresponding to the values n = 1, 2, 3.... Suppose, for example, p even. Then, as n increases from 0 to
Draw the line y = , and the parallel lines y =  – ε, y =  + ε q – 1, f(n) takes the values
at distance ε from it. Assume that for each positive ε , a number
N can be found, depending on ε , such that the part of the graph pπ 2pπ (q −1)pπ
0, sin , sin ......., sin
to the right of n = N lies within the band. Then we say that ‘as q q q
x approaches ∞, f(n) approaches ’ and write
When n increases from q to 2q–1 these values are repeated and
lim f (n ) =  so also as n goes from 2q to 3q – 1, 3q to 4q–1, and so on. Thus
n →∞
the values of f(n) form a cyclic repetition of a finite series of
different values. It is evident that when this is the case f(n) cannot
tend to a limit, nor to ∞ , nor to –∞, as n tends to infinity. The
limit does not exist.

Oscillating Functions
When f(n) does not tend to a limit, nor to ∞ , nor to –∞, as n tends
to ∞, we say that f(n) oscillates as n tends to ∞.
The simplest example of an oscillatory function is given by f(n)
= (–1)n , which is equal to 1 when n is even and to –1 when n
is odd. In this case the values recur cyclically.

FREE BOOKS FOR JEE & NEET =>(@iitjeeadv)


Limits 1.9

Now consider f(n) = (–1)n + n–1, the values of which are of an endless band of width 2ε. Assume that for each
–1 + 1, 1 + 12 , –1 + 13 , 1 + 14 , –1 + 15 , .... positive ε, a number D can be found, depending on
ε, such that the part of the graph to the right of x = D lies
When n is large every value is nearly equal to 1 or
within the band. Then we say that “as x approaches ∞, f(x)
–1, and obviously f(n) does not tend to a limit or to ∞ or to –∞
approaches ” and write as
and therefore it oscillates, but the values do not recur. It is to lim f(x) = 
be observed that in this case every value of f(n) is numerically x→∞

less than or equal to . We have a similar definition for lim f(x) = .


x→ −∞
If f(n) oscillates as n tends to ∞, then f(n) will be said to oscillate Note that the definition of the limit of a sequence can be
finitely or infinitely according as is or is not possible to assign regarded as a special case of the definition of the limit of a
a number K such that all the values of f(n) are numerically function, as its argument becomes infinitely large and assumes
less than K, only integral values.
i.e. | f(n) | < K for all values of n. Definition We assume that f(x) is defined for all x greater
For example, consider f(n) = n{1 + (–1)n}. than some number c. If for each number E there is a number
In this case the values of f(n) are D such that for all x > D it is true that f(x) > E, then we write
lim
0, 4, 0, 8, 0, 12, 0, 16,.... = ∞.
x→∞ f(x)

The odd terms are all zero and the even terms tend to ∞. Thus For each possible choice of a horizontal line, say, at height E,
f(n) oscillates infinitely. if we are far enough to the right of f, we stay above that line.
That is , there is a number D such that if x > D, then f(x) > E, as
But the function f(n) = sin nθπ, θ is rational, oscillates finitely.
illustrated in the figure.
Now consider the sequence
ps
π 2π 3π nπ
el
sin , sin , sin , ...., sin , ....
2 2 2 2
eh


It does not possess a limit since, for n = 1, 2, 3, 4, y n = sin
je

2
iit

consecutively takes on the values 1, 0, –1, 0 which then are


repeated indefinitely in the same order as n increases. Therefore
@

the number to which yn tends, as n → ∞, does not exist.


We have similar definitions for lim f(x) = − ∞,
Limit of a Function as x → ∞ x→∞

Definition Let f(x) be defined for all x beyond some lim f(x) = ∞ and lim f(x) = − ∞.
x → −∞ x → −∞
number c. If for each positive number ε, there is a number D
such that for all x > D it is true that For example, if a > 1, the function y = ax becomes infinite for
| f(x) –  | < ε , then we write x→∞
lim f(x) = . x → ∞ i.e. lim ax = ∞.
x →∞
If 0 < a < 1 we have lim ax = ∞.
The positive number ε is the challenge, and D is a response. x → −∞
The smaller ε is, the larger D usually must be chosen. The Further, lim log a x = ∞ , if a > 1
x →∞
geometric meaning of the precise definition of x→∞
lim f(x) = 
is shown in the figure. lim log a x = − ∞ , if 0 < a < 1

x →∞

Note: We say that a limit, such as xlim


→∞
f(x), exists when
the limit is a real number, but not when the limit is ∞ or –∞.

Bounded Function
The function f(x) in a given interval of x is said to have
(i) a lower bound if there exists a finite number A, such that
f(x) ≥ A, for all values of x in the given interval.
Draw two lines parallel to the x axis, one of height (ii) an upper bound if there exists a finite number B such that
 + ε and one of height  – ε . They are the two edges f(x) ≤ B, for all values of x in the given interval.

FREE BOOKS FOR JEE & NEET =>(@iitjeeadv)


1.10 Differential Calculus for JEE Main and Advanced

A function is said to be bounded in an interval if it has a lower Infinite Limits


bound as well as an upper bound. Suppose that when x → a the "limiting behaviour" of a function
Thus, a function is bounded if its values are contained between y = f(x) is such that its absolute value increases unlimitedly
two numbers A and B i.e. A ≤ f(x) ≤ B. then we say that the function f(x) approaches an infinitely large
Then, the graph of the function y = f(x) lies between the lines quantity as x → a.
y = A and y = B. Definition A function y = f(x) is said to approach infinity
Alternatively, a function f(x) is called bounded in a given as x → a if for all the values of x lying sufficiently close to a
interval of x if these exists a number M > 0 such that |f(x)| ≤ the corresponding values of the function f(x) become greater
M for all values of x in the given interval. If there is no such than any given arbitrarily large positive number.
number M, the function is called unbounded. If a function f(x) approaches infinity as x → a we write
The graph of a bounded function (considered within the given lim f ( x ) = ∞ .
interval) is completely contained between the straight lines y x→a

= –M and y = M. A function y = f(x) which approaches infinity as x → a does


For instance, the functions y = sin x and y = cos x are bounded not have a limit in the ordinary sense. To extend the ordinary
for all x while the function y = ax when considered on the terminology and to characterize the behaviour of the function
whole x–axis, is unbounded. whose absolute value |f(x)| increases indefinitely we say that
It should be stressed that when speaking of the boundedness the function y = f(x) tends to infinity or has an infinite limit.
of a function it is necessary to indicate the interval on which Suppose that a function y = f(x) tending to infinity as x → a
the function is considered. only assumed positive (negative) values in a neighbourhood
of the point c, then we say that the function f(x) approaches
ps
1 plus (minus) infinity as x → a. In these cases we write,
For example, the function y = is bounded in the interval
el
x respectively,
(1, ∞) and unbounded in the interval (0, 1).
eh

lim f ( x ) = ∞ and lim f ( x ) = − ∞


 π x→a x→a
je

The function y = tanx is bounded in the interval  0,  and


 4
 π Consider the following examples:
iit

unbounded in the interval  0,  .


 2
x +1
@

Definition The function f(x) is called bounded as x → a if (i) The function y = approaches infinity as x → 1, it
x −1
there exists a neighbourhood of the point a, in which the given
tends to – ∞ if x remains less than 1 and to ∞ if x remains
function is bounded. greater than 1.
Definition The function y = f(x) is called bounded as
x → ∞ if there exists a number N > 0 such that for all values of 1
(ii) The function y = has the limit ∞ as x → 0.
x satisfying the inequality x > N, the function f(x) is bounded. x2
From the definition of a bounded function f(x) it follows that −1
(iii) lim = − ∞.
if xlim f ( x ) = ∞ or lim f ( x ) = ∞ that is, if f(x) is an infinitely x →1 ( x − 1) 2
→a x →∞
large function, it is unbounded. The converse is not true: an
unbounded function may not be infinitely large. Note: In what follows, when speaking about a limit, we
For example, the function y = x sin x as x → ∞ is unbounded shall mean that the limit is finite unless it is stipulated that it
because, for any M > 0, values of x can be found such that |x is infinite.
sin x|>M. But the function y = x sin x is not infinitely large x2 − 4 x2 − 4
because it becomes zero when x = 0, π, 2π, .... For example, since lim = 4, we say that lim
x→2 x − 2 x→2 x − 2

Note: 1
exists. However, although lim 2 = ∞ , we do not say that
x→0 x
1. If a function f(x) increases as x → ∞ and remains bounded 1
then it possesses a limit. lim 2 exists.
x→0 x
2. The product of a bounded function by an infinitesimally Consider some more examples :
small quantity is an infinitesimally small quantity.
Let f(x) be an infinitesimally small quantity and g(x) a bounded lim cot −1 x = π , lim x 2 = ∞ , lim sin x dne.
x →∞ x →−∞ x →∞
function in a neighbourhood of the point a to which x tends i.e.
|g(x)| ≤ M. Then we have lim f ( x ) g ( x ) = 0 . Example 2: Show that lim e − x sin x = 0.
x→a x →∞

FREE BOOKS FOR JEE & NEET =>(@iitjeeadv)


Limits 1.11

Solution: –e–x ≤ e–x sin x ≤ e–x The function oscillates between –∞ and ∞.
Hence the limit does not exist.
2x 2
Example 4: Evaluate lim .
x →∞ x2 + 1
Solution: We have
2x 2 2( x 2 + 1) − 2 2
= = 2−
x +1
2
x +1
2
x +1
2

We can see from the graph that lim e − x sin x = 0. 2


x →∞ and as x → ∞, →0.
x2 + 1
Example 3: Evaluate lim 2xsin–1sin(x).
x→∞
π x π 2x 2
Solution: − .2 ≤ 2 sin–1sin(x) ≤ 2 x
x
Hence, x →∞ x 2 + 1 = 2 .
lim
2 2

Concept Problems A

1. Find the following limits for the function shown in the Find (i) lim f(x), (ii) lim f(x), (iii) lim f(x).
figure: x2 x→0 x→3
ps
Y
3. Find lim [αn], where α is any positive number and [.]
el
n→∞
denotes G.I.F.
eh

1
0 1 2 X
4. Find lim {n –(–1)n} + n{1–(–1)n}
je

n→∞

5. Find lim (sin nθπ)/{n(n–(–1)n)}, where θ is any real


iit

n→∞
(i) lim f(x) (ii) lim f(x) number.
@

x3

x→3

lim f(x) lim f(x) 6. Find lim sin nθπ, if θ is integral.


(iii) x
(iv) x→∞ n→∞

1
 1 7. Find lim .
 x  2 , x  2 n →∞ 1 1
cos nπ + n sin 2 nπ
2
 2 2
2. Let f(x) =  x 2  5  2  x  3
 1
lim
 x  13 x  3 8. Find n →∞  21 1  .
 n  cos nπ + n sin 2 nπ
 2 2 

(v) Sum rule


1.3 Theorems on Limits lim (f + g) (x) = lim [f(x) + g(x)]
x →a x →a
Let a and b be real numbers and let n be a positive integer.
(i) lim b = b (ii) lim x = a (iii) lim xn = an = xlim
→a
f(x) + xlim
→a
g(x) =  + m,
x→a x→a x→a
Let f and g be two functions such that i.e., the limit of the sum of two functions is equal to the
lim f(x) = , lim g(x) = m. sum of their limits.
x →a x →a (vi) Difference rule
Then lim (f – g) (x) = lim [f(x) – g(x)]
x →a x →a
(iv) Constant Multiple rule
lim k(f(x)) = k. lim f(x) = k., = xlim
→a
f(x) – xlim
→a
g(x) =  – m,
x →a x →a
where k is independent of x. i.e., the limit of the difference of two functions is equal to
A constant factor may be taken outside the limit sign. the difference of their limits.

FREE BOOKS FOR JEE & NEET =>(@iitjeeadv)


1.12 Differential Calculus for JEE Main and Advanced

The limit of an algebraic sum of two, three or, in general, (a) if q(a) ≠ 0, then lim f(x) = f(a).
any definite number of functions is equal to the algebraic x →a
sum of the limits of these functions. (b) if q(a) = 0 but p(a) ≠ 0, then lim f(x) does not exist.
x →a
(vii) Product rule
lim (fg) (x) = lim [f(x). g(x)] (c) if q(a) = 0 and p(a) = 0, then lim f(x) may or may not
x →a x →a x →a
exist. This will be dealt later on in detail.
= xlim
→a
f(x). xlim
→a
g(x) = m,
i.e., the limit of the product of two functions is equal to Example 1: Evaluate
the product of their limits. (i) lim (x + 2)
x→2
(ii) lim x(x – 1)
x→2
The limit of a product of two, three or, in general, any
lim x + 4
2
definite number of functions is equal to the product of the (iii) x→2 (iv) lim cos (sin x)
limits of these functions. x+2 x→0

(viii) Quotient rule x 2 + 3x + 2


lim (f / g) (x) = lim [f(x) / g(x)] (v) lim
x →a x →a x→1
x2 −1
= xlim
→a
f(x) / xlim
→a
g(x) =  / m, (m ≠ 0) Solution:
(i) x + 2 being a polynomial in x, its limit as x → 2 is given
i.e., the limit of the quotient of two functions is equal to lim (x + 2) = 2 + 2 = 4.
by x→2
the quotient of their limits provided the limit of the divisor
is not zero. (ii) Again x(x – 1) being a polynomial in x, its limit as x → 2
(ix) Power rule is given by
lim x(x – 1) = 2 (2 – 1) = 2.
ps
n
For any positive integer n, lim f ( x ) = n lim f ( x ),
x →a x →a
x→2

(iii) We have lim x + 4 = (2) + 4 = 2.


2 2
el
provided xlim f ( x ) > 0 when n is even.
x→2 x + 2
→c 2+2

( ) ( )
eh

m/n
Also lim (f ( x ) )
m/n
= lim f ( x ) where m/n is a rational (iv) lim cos (sin x) = cos lim sin x = cos 0 = 1.
je

x →a x →a x→0 x →0
number and the limit on the right exists.
iit

(v) Note that for x = 1, the numerator of the given expression is a


The limit of a power is the power of the limit. nonzero constant 6 and the denominator is zero. Therefore,
@

(x) Composition rule 6


the given limit is of the form . Hence, we conclude that
If f is continuous* at x = , then 0
lim x + 3x + 2 does not exist.
2

lim g(f(x)) = g  lim f ( x ) = g () . x→1


x2 −1
x→ a  x → a 
Example 2: Evaluate the following limits:
 
Specially, xlim n (f(x) = n  lim f ( x ) = n 
→a
x→a  (i) lim (2x2 – 3x + 4) (ii) x 3 + 2x 2 − 1
provided  > 0. x→5 lim
x→−2 5 − 3x
*
We shall learn about continuity in the next chapter. Solution:
A function is f(x) said to be continuous at x = a if xlim
→a
lim (2x2 – 3x + 4)
(i) x→5
f(x) = f(a).
lim (2x2) – lim (3x) + lim 4
= x→5
These results are of fundamental importance but their x→5 x→5
formal proofs will not be given in this book.
lim x2 – 3 lim x + lim 4
= 2 x→5 x→5 x→5
Note:
= 2(52) – 3(5) + 4 = 39
(i) For any polynomial p(x) = c0 + c1x + .... + cnxn and any
(ii) We start by using laws of limit, but their use is fully jus-
real number a,
tified only at the final stage when we see that the limits of
lim p(x) = c + c a + .... = c an = p(a). the numerator and denominator exist and the limit of the
x →a 0 1 n
denominator is not 0.
(ii) Consider the rational function
p( x ) x 3 + 2 x 2 − 1 xlim ( x 3 + 2 x 2 − 1)
f(x) = where p(x) and q(x) are polynomials. For any →−2
q( x ) lim =
x →−2 5 − 3x lim (5 − 3x )
real number a, x →−2

FREE BOOKS FOR JEE & NEET =>(@iitjeeadv)


Limits 1.13

vanish at x = 0, we obtain the limit by substituting x = 0 in


lim x 3 + lim 2 x 2 − lim 1
= x →−2 x →−2 x →−2 the function.
lim 5 − 3 lim x
x →−2 x →−2
03  3.02  0  3 3 1
lim x + 3x − x − 3 =
3 2
 
( −2)3 + 2( −2) 2 − 1 1 x→0
x2 + x − 6 02  0  6 6 2
= =– .
5 − 3( −2) 11
Study Tip
Example 3: Find lim
3
2 x 2 − 10 .
x→3 Although such substitution produces the correct answer in
2
Solution: Because lim (2x – 10) = 2(3 ) – 10 = 8 and 2 this particular case, in many limits involving non-elementary
x→3 functions, it produces either an incorrect answer or no answer
lim 3 x = 2 we can write lim 3
2 x 2 − 10 = 2. at all.
x →8 x→3
Also do not assume from these problems that lim f(x) is
x→a
Direct Substitution invariably f(a).
We learned that the limit of f(x) as x approaches a does not n (1 + x ) − x 2 + 2
To evaluate the lim , we notice that the
depend on the value of f at x = a. It may happen, however, that x →0 cos(sin x ) + 1 + sin −1 x
the limit is precisely f(a). In such cases, we say that the limit
can be evaluated by direct substitution. That is n (1 + x ) − x 2 + 2
function is elementary and x = 0 lies in
cos(sin x ) + 1 + sin −1 x
lim f(x) = f(a). (substitute a for x)
ps
x →a the domain. Hence, limit is equal to the function's value at x = 0,
el
Such well behaved functions are continuous at a – we will
examine this concept more closely in the next chapter. Here we n (1 + x ) − x 2 + 2 0−0+2
eh

i.e. lim = =1.


discuss some limits that can be evaluated by direct substitution. x →0 cos(sin x ) + 1 + sin −1 x 1+1+ 0
je

In the next section, we will discuss techniques for evaluating


limits for which direct substitution fails. To evaluate lim [ x ] we cannot use direct substitution property
iit

x →2
Basic Elementary Functions as lim [ x ] = [2] = 2, since [x] is a non-elementary function. We
@

x →2
The following functions are called basic elementary functions : know that this limit does not exist.
(i) y = xp (ii) y = ax Sometimes, using direct substitution property on non-elementary
(iii) y = logax (iv) y = trigo x functions may give correct results as in lim [ x 2 ] = [0]2 = 0, but
(v) y = trigo–1 x. it is not advisable to use in general. x →0

Elementary Functions x can be treated as an elementary function since we can


write x = x 2 .
Elementary function is one which may be represented by
a single formula y = f(x), where f(x) is made up of basic
elementary functions and constants by means of finite number Thus, lim x x − 2 = 2 2 − 2 = 0.
x→2
of operations of addition, substraction, multiplication, division
and composition. x2 − 4
Note that piecewise defined functions are non-elementary. For Example 5: Find lim
x→1
π .
cos x
example, [x], {x}, sgn x are non-elementary functions. 4
Solution: The given function is an elementary function
Limit of Elementary Functions and the point x = 1 lies in the domain, and thus
Theorem For all elementary functions, limit at any point in
the domain is equal to the function's value at that point. x2 − 4 12 − 4
lim π = π =–3 2 .
x→1 cos x cos ·1
lim x + 3x − x − 3 .
3 2
4 4
Example 4: Find x→0
x2 + x − 6 But the limit of this function as x → 2 can not be found by
Solution: Since the given function is an elementary means of the direct substitution since the denominator turns
function and the polynomial in the denominator does not into zero at the point x = 2.

FREE BOOKS FOR JEE & NEET =>(@iitjeeadv)


1.14 Differential Calculus for JEE Main and Advanced

Concept Problems B


1. Show that lim
1
= 10. 5. Show that lim cos x  4 tan x  1 .
2  x  2x 4
x →0.01 x→0
x 2
2
2. lim 3x .
Evaluate x→−1 1
2x − 1 6. Why doesn't the limit lim x sin  0 follow from
x x 0
lim cos x the product law of limits with f(x) = x and g(x) =
3. Evaluate x .
sin x − π sin(1/x) ?
x
4. Evaluate lim .
x0 cos 1 x

exists a corresponding number δ > 0 such that for all x


1.4 One-Sided Limits satisfying a – δ < x < a ⇒ |f(x) – | < ε and we call it the
For functions that are defined piecewise, a two-sided limit at some left hand limit.
x, where the formula changes, is best obtained by first finding x
For instance, we see that lim  1 because |x|/x is equal
the one-sided limits at the number.
ps
x 0 
x
Consider the behaviour of the function to –1 for all x to the left of zero.
el

x  1 if x  0 Right Hand Limit (R.H.L.)


eh

f (x)  
| x | 1 if x  0
Suppose that f is defined on the interval (a, c) immediately to
je

The graph of y = f(x) is shown below : the right of a. Then we say that the number  is the right hand
limit of f(x) as x approaches a from the right, and we write
iit

Y
f (x)  
@

1 f(a+) = xlim
a  ...(2)

0 X provided that we can make the number f(x) as close to  as we
–1 please merely by choosing the point x in (a, c) sufficiently
close to a.
As x → 0, f(x) does not approach a specific number. However, Formally, lim f ( x )   , if for every number ε > 0 there exists
x a
as x approaches 0 through positive numbers, f(x) → 1. Also, a corresponding number δ > 0 such that for all x satisfying a <
as x approaches 0 through negative numbers, f(x) → –1. This x < a + δ ⇒ |f(x) – | < ε and we call it the right hand limit.
behaviour illustrates the idea of one-sided limits, which will x
now be defined. For instance, we see that lim  1 because |x|/x is equal to
x 0 x 

Left Hand Limit ( L.H.L.) 1 for all x to the right of zero.


Suppose that f is defined on the interval (c, a) immediately to
One-Sided Limits and Two-Sided Limits
the left of a. Then we say that the number  is the left hand limit
of f(x) as x approaches a from the left, and we write Suppose that the function f is defined for x ≠ a in a
neighbourhood of the point a. Then the two-sided limit
f(a–) = lim f ( x )   ...(1) lim f ( x ) exists and is equal to the number  if and
x a

x →a
provided that we can make the number f(x) as close only if the one-sided limits lim f ( x ) and lim f ( x ) both exist
x a 
x a

to  as we please merely by choosing the point x in and are equal to .


(c, a) sufficiently close to a. This theorem is useful in showing that certain limits
We may describe the left hand limit in (1) by saying that f(x) →  do not exist, frequently by showing that the left hand
as x → a–, that is, as x approaches a from the left. The symbol and right hand limits are not equal to each other.
a– denotes the left hand side of a. And conversely, if there exists a limit  of a function at the point
Formally, lim f ( x )   , if for every number ε > 0 there a, then there exist limits of the function at the point a both on
x a the right and on the left and they are equal.

FREE BOOKS FOR JEE & NEET =>(@iitjeeadv)


Limits 1.15

Since the one-sided limits are not equal, the corresponding Study Tip
two-sided limit
x 1. When a function is defined only on one side of a point a, then
lim does not exist.
x →0 x lim f(x) is identical with the one-sided limit, if it exists.
x →a
 x 3
 x 1 For example, if f(x) = x , then f is defined only at and
Let f(x) = 
2  x,
 x1 to the right of zero. Hence, xlim
→0
lim
x = x  0 x = 0. Of 

The given function is defined on the entire number line. Let us course, lim x does not exist, since x is not defined
x  0
compute the one-sided limits of this function at the point x = 1.
when x < 0.
f(1– ) = lim ( x )  1
3
x 1 0
2. A function cannot have an ordinary limit at an endpoint of
its domain. It can have a one-sided limit.
f(1+) = x lim (2  x )  3 The function f(x) = 9 − x 2 has the interval –3 ≤ x ≤ 3 as
1 0
its domain. If a is any number on the open interval (–3, 3),
Thus, f(1–) ≠ f(1+). Hence, the given function has no limit at
then lim 9 − x 2 exists and is equal to 9 − a 2 .
the point x = 1. x →a
Now consider a = 3. Let x approach 3 from the left; then
Example 1: The graph of a function g is shown in the lim 9 − x 2 = 0. For x > 3, 9 − x 2 is not defined.
x  3
figure. Use it to state the values (if they exist) of the following
limits: Hence, lim 9 − x2 = lim 9 − x 2 = 0.
lim g(x) x →3 x  3
(a) x (b) x lim g(x)
ps
 
2 2
Similarly, lim 9 − x 2 = lim 9 − x 2 = 0.
lim g(x)
(c) x→2 (d) lim g(x) x  3 x 3
el

x5
1 |x|
lim g(x) Consider the limit lim sin (e ) .
eh

(e) lim g(x) (f) x 0


x5 x→5
Even if f(0) = π/2 , the function is undefined on both sides
je

Y of x = 0. Hence there is no question of limit at x = 0.


iit

4 [ x 2 ]  2[ x ]  3
In the limit lim 2 , the function is
x 3 [ x ]  4[ x ]  3
@

3 y = g(x)
undefined in the right neighbourhood of x = 3. Hence
R.H.L. cannot be discussed. However, the limit is 1/3 since
1 L.H.L. can be evaluated as 1/3.
1
0 1 2 3 4 5 X
3. There is no shorthand for the two-sided limit of lim
x →0 x

1 1
even though we write lim    and lim   . We
Solution: From the graph we see that the values of g(x) x 0 x 
x 0 x 

approach 3 as x approaches 2 from the left, but they approach may only say that the limit does not exist.
1 as x approaches 2 from the right. Therefore, 2x  1
Note that we do not write lim   . However, it
(a) lim g(x) = 3 and x 1 x  1
x 2

(b) lim g(x) = 1 2x  1


x 2 would be correct to write lim  .
(c) Since the left and right limits are different, we conclude
x 1 x 1
that lim g(x) does not exist. x 3
x→2 Example 2: Find lim and
The graph also show that
x 1 x 3  3x  2
x 3
(d) lim g(x) = 2 and lim .
x5 x 2 ( x  1) 2 ( x  2)
(e) lim g(x) = 2 x 3
x5
Solution: lim
(f) This time the left and right limits are the same and so, we x 1 x 3  3x  2
lim g(x) = 2
have x→5 x 3
= lim 
x 1 ( x  1) 2 ( x  2)
Despite this fact, notice that g(5) ≠ 2.

FREE BOOKS FOR JEE & NEET =>(@iitjeeadv)


1.16 Differential Calculus for JEE Main and Advanced

x 3 The right and left hand limits are equal. Thus, the limit exists
Now, lim   lim f(x) = 0.
and x→4
x 2 x  3x  2
 3

x 3 The graph of f is shown in the figure.


lim 
x  2 x 3  3x  2


Y

x 3
∴ lim  does note exist
x  2 ( x  1) 2 ( x  2)

4. In some cases a function is defined at x = a however limit
does not exist at x = a and in some cases limit x → a exists but 0 4 X
f(a) is not defined. Note these situations in the graph of the
following function y = f(x) :
 sin[ x ]
Y  , for [ x ]  0
Example 4: If f(x) =  [ x ] ,
 0 , for [ x ]  0
2

then find lim f ( x ) where [.] denotes the greatest integer
1 function. x →0
0 1 2 3 4 4 X Solution:
At x = 0 , f (0+) = 1 sin[ x ] sin (1)
At x = 1 , f (1–) = 0 ; f (1+) = 1 lim f ( x )  lim   sin 1 .
x 0 x 0 [x] (1)
At x = 2 , f (2+) = f (2–) = 1 ≠ f(2)
ps
At x = 3 , f (3+) = f (3–) = 2 = f(3) lim f(x) = 0 as it is given that f(x) = 0 for [x] = 0.
and x
el

0
At x = 4 , f (4+) = f (4–) = 1 ≠ f(4)
So, x→0
lim f(x) does not exist.
eh

At x = 5 , f (5–) → ∞ (limit dne).


5. In functions involving the greatest integer function,
je

fractional part function and signum function the following Substitution for One-sided Limits
iit

points must be noted : L.H.L. = xlim f ( x )  lim f (a  h )


a 
h 0
@

(i) f (x) = [ x ] and f (x) = {x} has no limit at all integers.


(ii) y = [f(x)] and y = {f(x)} may not have limit at points Substitute x = a – h, assuming h as a small positive quantity.
where f(x) is an integer. Note that as x → a–, h → 0+. But we often write h → 0.
|x| f ( x )  lim f (a  h )
(iii) f (x) = has no limit at x = 0. R.H.L. = xlim
a 
h 0
x
(iv) y = sgn(f(x)) may not have limit at points where Substitute x = a + h, assuming h as a small positive
f(x) = 0. quantity. As x → a+ , h → 0+.
Non-Existence of Limit x.sgn( x  1)
Example 5: Find lim
x 1
Three of the most common types of behaviour associated with
the non-existence of a limit.
1. f(x) approaches a different number from the right side of Solution: L.H.L. = lim x.sgn( x  1) Put x = 1 – h
x 1
a than it approaches from the left side. = hlim(1  h ) sgn(1  h  1)
0
2. f(x) increase or decreases without bound as x approaches
a. = hlim(1  h ) sgn( h )
0
3. f(x) oscillates too much as x approaches a.
= hlim(1  h )(1)  1 .
 0
 x4 if x4
Example 3: If f(x) =  determine
lim  8  2x
 if x4 R.H.L.= lim x. sgn( x  1) Put x = 1 + h
whether x→4
f(x) exists. x 1
lim
Solution: Since f(x) = x − 4 for x > 4, we have x 4  = hlim(1  h ) sgn(1  h  1)
0
lim
f(x) = x 4 x − 4 = 4 − 4 = 0.


1  h ) sgn h = lim(1  h ) .1 = 1
Since f(x) = 8 – 2x for x < 4, we have = hlim(
0 h 0

lim f(x) = lim (8 – 2x) = 8 – 2.4 = 0. Since, L.H.L. ≠ R.H.L., lim x sgn( x  1) does not exist.
x 4 x 4
x 1

FREE BOOKS FOR JEE & NEET =>(@iitjeeadv)


Limits 1.17

Example 6: Find xlim[ x] lim   2 cos(h)  = – 2


0 
→2 = h

Solution: L.H.L. = lim[2  h ] = lim 1  1 . 


 
R.H.L. = lim  2 sin   h   
5  
h 0 h0 h 0   4 4 
R.H.L. = hlim[ 2  h ]  lim 2  2 .
0 h 0 lim   2 cos(h)  = –2
= h 0 
Since, L.H.L. ≠ R.H.L., lim[ x ] does not exist.
x →2 Since, L.H.L. = R.H.L., the required limit is –2.
Example 7: Evaluate lim{x} .
x →2 Example 10: Evaluate the left hand and right hand limits
of the function
Y
 ( x 2  6 x  9)
 , x3
1

2
1

f(x) =  ( x  3) at x = 3.


+

x
x

Solution: 
–1 –h 0 1 2–h 2 X  0 , x3
Solution: The given function can be written as
{2–h} = 2 – h – 1 = 1 – h
| x  3 |
, x3
f(x) =  ( x  3)
We have {2 – h} = 2 – h – 1 = 1 – h
L.H.L. = lim{2  h}  lim(1  h )  1 .

h 2 h 0  0 , x3

R.H.L. = hlim{2  h}  lim h  0 ∴ L.H.L. = x
lim f(x) = lim (3 – h)
0 h 0 3
ps
h→0
Note that if h as a small positive quantity{–h} = 1 – h | 3 h 3| | h | h
el
 1  = lim = lim = lim = –1
Example 8: Find lim   x    {x 2 }  h 0 (3  h  3) h  0 (  h ) h 0  h
eh

x 2   2 
and R.H.L. = x
lim f(x) = lim (3 + h)
je

3
Solution: L.H.L. =
h→0

| 3 h 3|
iit

|h| h
= lim = lim = lim = 1
= lim 2  {4  4h  h }
2 (3  h  3)
h 0 h →0 h h →0 h
@

h 0
Hence the left hand limit and right hand limit of f(x) at
= hlim 2  {4h  h 2 } = lim 2 + {– h (4 – h)} x = 3 are –1 and 1 respectively.
0 h→0

2  1  (4h  h 2 ) , since 4h – h2 is a small positive cos[ x ], x  0


= hlim
0 Example 11: Let f(x) = 
| x | a , x  0


quantity.
=3 Find the value of a, given that x→0 lim f(x) exists, where
 1 [ . ] denotes the greatest integer function.
R.H.L. = lim   2  h     {(2  h ) 2 }
h 0   2  lim f(x) exists
Solution: Since x→0
= hlim 2  {4  h  4h} 2
lim f(x) = lim f(x).
0 x0  x0 

⇒ lim f(0 – h) = lim f(0 + h)


= hlim 2  {4h  h 2 } h→0 h→0
0
⇒ lim |0 – h| + a = lim cos [0 + h]
lim 2 + 4h + h2 = 2 h→0 h→0
= h→0
⇒ a = cos 0 = 1
Since, L.H.L. ≠ R.H.L., the limit does not exist. ∴ a=1
Example 9: Find lim5 [sin x + cos x]
x
4 Study Tip
     If f(x) is an even function, then
Solution: L.H.L. = lim5  2 sin  x  4  
x   
4 (i) lim f ( x )  lim f ( x )
x 0  x 0 
  5  
L.H.L. = hlim  2 sin   h   lim f ( x )  lim f ( x )
0   4 4  (ii) x a  x  a 

FREE BOOKS FOR JEE & NEET =>(@iitjeeadv)


1.18 Differential Calculus for JEE Main and Advanced

If f(x) is an odd function then


sin −1 x
(i) xlim0 
f ( x )   lim f ( x ) .
x 0 
For x < 0, >1
x
lim f(x) exists, then lim f(x) = 0.
Further if x→0  sin 1 x 
x→0
∴ L.H.L.= lim   = 1. Hence P = 1.
x 0 
(ii) lim f ( x )   lim f ( x )  x 
x a 
x  a 

 1 1 
Example 13: Find lim  tan 1  cot 1 2 
 sin 1 x  x 0  x x 
Example 12: Evaluate lim   where [ . ] denotes
x 0 
 x   1 1 
Solution: lim  tan 1  cot 1 2 
the greatest integer function. x 0  x x 
1 1
 sin 1 x  = lim tan 1  lim cot 1 2
Solution: Let P = lim   x 0 x x 0 x
x 0 
 x 
1 1
= lim tan 1  0 = lim tan 1
sin −1 x x 0 x x 0 x
For x > 0, sin x > x ⇒ –1
>1
x Now we evaluate the one sided limits :
 sin 1 x  1 1 
∴ R.H.L. = lim   =1 L.H.L. = lim tan 
x 0  x 0 
x 2
 x 
1 
R.H.L. = lim tan 1  .
ps
sin −1 x x 0 x 2
The function is even.
Finally, the limit does not exist.
el
x
eh
je

Concept Problems C
iit

@

Y
1. Find lim f(x) f(x)
x / 2 2
Y
f(x) 1
2

–1 0 1 2 3 4 5 X
1
4. The following figure shows the graph of a function f.
Decide which of the given limits exist and evaluate those
0 π/2 π X which do.
2. Find lim f(x) (a) lim f ( x ) (b) lim f ( x )
x 0 x →1

x →1
(c) lim f ( x ) (d) lim f ( x )
Y

x  2 x  2
f(x) Y
2
3
1
2
0 1 2 X
1
3. Find (i) lim f(x), (ii) lim f(x), (iii) lim f(x)
x→0 x→1 x→3 0 1 2 3 4 X

FREE BOOKS FOR JEE & NEET =>(@iitjeeadv)


Limits 1.19

5. The following figure shows the graph of a function f.


x2
Decide which of the given limits exist and evaluate those 10. Given that f(x) = , show that
which do. ( x − 1)( x − 2)
(a) lim f ( x ) (b) lim f ( x ) (i) lim f(x) = ∞, lim f(x) = –∞,

x →1 x →2 x1 x1
(c) lim f ( x ) (d) lim f ( x ) (ii) lim f(x) = –∞, lim f(x) = ∞,
x →3 x  4

x 2 x 2
Y
(iii) lim f(x) = 1 = lim f(x).
3 x x

11. Find the left and right hand limits of the function
2
ψ(x) =   x  1 for x  1, at the point x = 1.

 2 x  1 for x 1
1
0 1 2 3 4 X  2 x  3, x  2

12. Find the limit of f (x) =  at x = 2.
4  x , x  2
2
6. Use the graph of f and g to find the limits that exist. 
Y
y = g(x)  2( x  1) if x  3
Y y = f(x) 2 
13. Find lim f ( x ) where f(x) = 4 if x  3
1 x →3  2
1  x  1 if x 3
0 1 2 X –1 0 1 2 X 14. Using graphs find the limit (if it exists):
ps
–2 x 2  2 ,
 x 1
(a) (x) =  , lim f ( x )
el

 1 , x  1 x →1
lim f(x) + g(x), a = 0, 2
eh

(i) | x 5|
x →a (b) lim
x 5 x  5
je

(ii) lim f (x)


lim tan 2 x .
iit

x→2 1+ g ( x ) (c)
x  / 2
Evaluate the following limits :
@

7. 15. Suppose that f is an odd function of x.


1/( x  5)  x Does knowing that lim f(x) = 3 tell you anything about
(i) lim 10 (ii) lim sin–1  log 3 3  x 0
x 5   

x1
lim f(x) ?
( x  1) −1 1 x 0
(iii) lim (iv) lim tan
x 1 ( x  1)  2 x→0 x 16. Suppose that f is an odd function of x. Does knowing that
lim f(x) = 7 tell you anything about either lim f(x) or
8. Evaluate lim sin–1 (sec x). x  2 x 2
x→0

9. Evaluate lim cos–1(1 + tan x). lim f(x)?


x0 x 2

Practice Problems B

19. Evaluate the limits
17. Evaluate (i) lim f ( x ) (ii) lim f ( x ) sin[cos x ]
x →1 x 1 (i) lim sin–1 [sec x] (ii) lim

x→0 x→0 1+ [cos x ]
x | x 3|
where f ( x )  . where [.] denotes the greatest integer function.
( x  x  6) | x |
2
 2 1
 x sin , x0
18. Evaluate the following limits : 20. If f(x) =  x , find lim f ( x )
(i) lim x sgn( x  1) x 2
 , x0
x →0
x1
  
tan 2 x 21. Evaluate lim 1 – x + [x] – [1 – x],
(ii) lim  tan   x   x1
8 

 
x where [.] denotes the greatest integer function.
4

FREE BOOKS FOR JEE & NEET =>(@iitjeeadv)


1.20 Differential Calculus for JEE Main and Advanced

2 n 1
x2 −1
 [x r ]  n  1 (i) lim
x→1 | x −1 |
r 1
22. Evaluate lim
1  [ x ] | x | 2 x
x →0 ¯ 2{x} − 4
(ii) lim
where [.] denotes the greatest integer function. x→2 [x] − 3
sin{x  10}  | x3 |  x 3 
23. Find lim , (iii) lim     , a > 0
x8 {10  x}
x →a  a  a  
where {.} denotes the fractional part function. x  1 0  x 1
(iv) lim f(x) where f(x) = 
24. Find the left and right hand limits of the function x→1 3 x  2 1 x3
1
φ(x) = 1
at the point x = 2. 27. In each of the following functions :
x  2 x 2 (a) Sketch the graph of the function f.
(b) For each integer n, evaluate the one-sided limits
25. Evaluate the one-sided limits (where [.] denotes the
greatest integer function): lim f ( x ) and lim f ( x ) in terms of n.
x  n x  n
1 [x 2 ] − 1 (c) Determine those values of a for which lim f ( x )
(i) lim (ii) lim x →a
x→1 ( x − 1) 2 x2 −1 exists.
  
x→1

(i) f (x)  
2 if x is not an integer,
1
(iii) lim (iv) lim x(–1)[1/x]
2  (1) if x is an integer,
x
2 − 21/ x
x→0 x→0
26. Evaluate the one-sided limits (where [.] denotes the
ps
(ii) f(x) = [10x]
greatest integer function) : 1
el
(iii) f(x) = x – [x] – .
2
eh
je
iit

1.5 Determinate and Indeterminate (ii) lim(f ( x )  g ( x ))  



x a
@

Forms (iii) lim f ( x )g ( x )   (for b  0)


x a
Until now we have found limits using graphs or the direct
substitution property. We shall now classify the problems of (iv) lim
g( x )
  (for b  0)
limit in two ways: x a f (x)
(i) Determinate form: In which the limit can be predicted f (x)
(v) lim  0.
apriori without resorting to special methods. This is so x a g( x )
named because two problems having the same form have Here we give a list of determinate forms. One should try to
equal limits. understand the meaning of these forms :
2x cos x
For example, lim and lim are in the same (i) 0 + c = c (ii) 0 + 0 = 0
x 1 x x →0 ln | x |
(iii) c + ∞ = ∞ (iv) ∞ – c = ∞
tan
2 (v) c – ∞ = – ∞ (vi) 0 + ∞ = ∞
form
1
and we see that both of them have the limit 0. (vii) 0 – ∞ = – ∞ (viii) ∞ + ∞ = ∞
∞ (ix) 0 × c = 0 (x) ∞ × c = ∞ , c > 0
Hence we call
1
as a determinate form and we are sure (xi) ∞ × ∞ = ∞ (xii) 0 × 0 = 0
∞ c 0
1 (xiii)    , c > 0 (xiv) =0, c ≠ 0
that any problem in form will have limit 0. 0 c

f (x)  0
If lim f(x) = b and lim g(x) = 0 then lim is said (xv) 
 (xvi) 0
x →a x →a x →a g ( x ) 0 
b (xvii) ∞∞ = ∞ (xviii) 0∞ = 0
to be of the form . If b > 0 and g(x) → 0+ then the Consider some examples of determinate form :
limit is ∞. 0
1
(i) lim 2 = ∞
Note: Let lim f ( x )  b, lim g ( x )   , then we have x →0 x
x a x a
1
(i) lim (f(x) + g(x))   (ii) lim 4 = ∞,
x a x →0 x

FREE BOOKS FOR JEE & NEET =>(@iitjeeadv)


Limits 1.21

 1 1  x2 −1 Several indeterminate forms arise from the limit lim [f(x)]g(x)


(iii) lim  2  4  = lim = –∞, x →a
x→0  x x  x→0 x4 1. lim f(x) = 0 and lim g(x) = 0 type 00
x →a x →a
 1 
(iv) lim  2  2  = ∞, 2. lim f(x) = ∞ and lim g(x) = 0 type ∞0
x→0  x  x →a x →a

 1  3. lim f(x) = 1 and lim g(x) = ± ∞ type 1∞


(v) lim 1  x   = ∞, x →a x →a
x 0  x2 
Each of these three cases can be treated either by taking the
 1   1  natural logarithm:
(vi) lim  2  2   1  x  2   = lim (1 – x) = 1,
x 0  x   x  x→0 let y = [f(x)]g(x),
x x
 x   1  1 then n y = g(x) n f(x)
(vii) lim    xlim
x   1  2 x 
    0,
  2  1 / x  or by writing the function as an exponential
2
(viii) lim (1  cos x )( x  2)  lim (1  1) 2  4 . [f(x)]g(x) = eg(x) n f(x)
x 0 x 0 In either method we are led to the indeterminate product g(x)
(ii) Indeterminate form: In which the limit cannot be n f(x), which is of type 0 · ∞.
predicted apriori. Here special methods are required to The reason why these forms carry the label "indeterminate" is not
find the limit. In this case two problems having the same that they cannot be resolved, but rather that there is no predicting
form may not have equal limits. what the limit of the given function will be, or indeed whether it will
For example, the limit of a quotient, exist at all, until the given function receives a thorough investigation.
ps
It is tempting to argue that an indeterminate form of type 0 .
f (x)
lim where lim f(x) = 0 and lim g(x) = 0, is ∞ has value 0 since “zero times anything is zero.” However,
el
x →a g ( x ) x →a x →a
this is fallacious since 0 . ∞ is not a product of numbers, but
rather a statement about limits.
eh

called an indeterminate form of type 0/0. Consider the


following limits in 0/0 form: The form (∞ + ∞), on the other hand, is determinate in the sense
je

x x2 x that unfailingly the sum function has limit ∞ if both summand


1 , lim  0 , lim 3   functions have limit ∞ at the prescribed point of approach.
iit

lim
x 0 x x 0 x x 0 x
The following examples show how easy it is to construct
@

We notice that the result varies quite significantly even if they are in the quotient functions and sum functions with indeterminate
same indeterminate form. The word “indeterminate” here refers forms having an arbitrary real number C as a limit, or having
to the fact that the limiting behaviour of the quotient cannot be no limit whatever.
determined without further study. The expression “0/0” is just a
The form 0/0.
device to describe the circumstance of a limit of a quotient in
C sin x x sin(1 / x )
which both the numerator and denominator approach 0. lim  C while lim does not exist.
x 0 x →0 x
The important indeterminate forms are : x
0 , ∞ , 0 × ∞, ∞ − ∞, 0º, ∞º and 1∞ The form (∞/∞).
0 ∞ Cx x (sin x  2)
lim C while xlim does not exist.


Note that ‘0’ doesn’t mean exact zero but it represents a value x  x x

approaching towards zero. The form (∞ – ∞).


The behaviour of ‘1’ and ‘∞’ are similar. lim (x – (x – C)) = C while lim (x2 – x) does not exist.
x  x 
Indeterminate Products
The above examples were artificially framed to point that
If lim f(x) = 0 and lim g(x) = ∞ (or – ∞), then it isn't clear
x →a x →a "indeterminate" means "unpredictable without special attention
what the value of lim f(x)g(x), if any, will be. There is a to the functions involved".
x →a There is a myth that circulates among students which states
struggle between f and g. If f wins, the answer will be 0; that all indeterminate forms of types 0º, ∞º, and 1∞ have value
if g wins, the answer will be ∞ (or – ∞). Or there may be a 1 because “anything to the zero power is 1” and “1 to any
compromise where the answer is a finite nonzero number. This power is 1.” The fallacy is that 0º, ∞º, and 1∞ are not powers
kind of limit is called an indeterminate form of type 0 · ∞. of numbers, but rather descriptions of limits. The following
We can deal with it by writing the product fg as a quotient : examples show that such indeterminate forms can have any
f g
fg = or fg = positive real value :
1/ g 1/ f
(a) lim [x(ln a)/(1 + ln x)] = a (form 0º)

This converts the given limit into an indeterminate form of x0
0
the type or ∞/∞ . (b) lim [x(ln a)/(1 + ln x)] = a (form ∞º)

0 x

FREE BOOKS FOR JEE & NEET =>(@iitjeeadv)


1.22 Differential Calculus for JEE Main and Advanced

(c) lim [x(ln a)/(1 + ln x)] = 1 (form 1∞) (ii) ∞ ∞ ∞



x→1 (iii) 0 indeterminate
We can easily verify these results. lim f(x) lim g(x) lim [f(x)/g(x)]


x →a x →a x →a
Find whether the following limits are in determinate /
indeterminate form. Also evaluate the limit in case of (i) ≠0 0 ∞
determinate form. (ii)  ∞ 0
2 (iii) ∞  ∞
(i) lim 0 (iv) 0 0 indeterminate
x 0 1  21/ x
2

(v) ∞ ∞ indeterminate
1 2 2
These results are valid even when x tends to
as x → 0, → 0, 21/ x → ∞ , → 0.
∞ a–. a+, ∞ or – ∞.

x
f (x)  5
sin 1 x  /2 Example 1: If lim = 3 then find lim f ( x ) .
(ii) lim  0 , since  →0 x 2 x  2 x→2
x 1 
    Solution: Since the denominator is approaching 0, we
tan x
2 can have limit only through 0 form.
(iii) lim (e x  x )   , since ∞ + ∞ = ∞. 0
x  Hence lim f ( x ) must be 5.
x→2
lim(1  sin x )1/ x
2

(iv) (1∞ indeterminate form)


x 0 f (x)  5
Note that if lim = 4 then also lim f(x) is 5.
1
x2
x 2 x→2
(v) [cos
lim(1  x ]) x2 is not in 1∞ indeterminate form
f (x) 0
x 0
Also if lim 2 = 2 then using form we have
ps
exact 1
0
since (exact 1)∞ = 1 (determinate form),
x→0 x
el
1 f (x)
(vi) [cos
lim(1 x ]) x = 1. lim f ( x )  0 and further lim = 0.
eh

x 0   x0 x→0 x
exact 1
je

 1 1  Example 2: Find the left hand and right hand limits of


(vii) lim    (∞–∞ indeterminate form) 1
iit

x 0  x 2 sin 2 x  the function f ( x )  as x → 3.


[x 2 ]  1  exact 0  x  21/( x  3)
@

(viii) lim 0  0  = 0
x 1 x  1   Solution: If x → 3–, then


Let us try to deduce conclusion about the limits: 1/(x – 3) → –∞ and 21/(x–3) →0.
lim [f(x) + g(x)] , lim [f(x).g(x)] and lim [f(x)/g(x)] based Consequently, lim f ( x )  1 / 3 .
x →a x →a x →a x  3
upon the individual behaviours of f(x) and g(x). Now if x → 3 , lim f ( x )  0 .
+

Assume that  is a positive real number. x  3

lim f(x) lim g(x) lim [f(x) + g(x)] Example 3: Does the function

x →a x →a x →a
 1  1
(i)  ∞ ∞ y =  sin   sin  tend to a limit as x tends to 0?
(ii)  –∞ –∞  x  x
(iii) ∞ ∞ ∞ Solution: The function is equal to 1 except when
(iv) –∞ –∞ –∞ sin(1/x) = 0; i.e. when x = 1/π, 1/2π,.... For these values the
(v) ∞ –∞ indeterminate formula for y assumes the meaningless expression 0/0, and
lim f(x) lim g(x) lim [f(x).g(x)] y is therefore not defined for an infinitely many values of x
x →a x →a x →a

near x = 0. Note that this is not the indeterminate form 0/0.
(i) ≠0 ∞ ∞ Hence the limit does not exist.

Concept Problems D

1. Find whether the following limits are in indeterminate (iii) lim x n x
form. Also indicate the form. x→0
1 1− x 1 1 
(i) lim (ii) lim (iv) lim   2 
x→0 x x→1 1 − x 2 x→0  x x 

FREE BOOKS FOR JEE & NEET =>(@iitjeeadv)


Limits 1.23

(v) lim (sin x)x (vi) lim (n x)x lim e tan x  1 1
x→0 x→0 (iii)  (iv) lim tan–1
x x→0
1
1 tan x x
2 e
lim 1  sin x 
(vii) x→0
x
lim (1)
(viii) x→0 1/x
5. Evaluate the following limits:
2. Suppose that limx → a f(x) = ∞ and limx → a g(x) = c, where sin 1 x 1
(i) lim lim
(ii) x→0
c is a real number. Prove each statement. x→1 x ln | x |
(a) lim [f(x) + g(x)] = ∞
tan
x →a 2
e x ln 2  − 
1
(b) lim [f(x) g(x)] = ∞ if c > 0 (iii) lim (iv) lim x 2 1 − e x  2

x →a x
ex x →0
 
(c) lim [f(x) g(x)] = – ∞ if c < 0
x →a x
6. Show that lim  x    .
3. Let lim f ( x )  0 with f(x) ≠ 0 for x ≠ a, lim g ( x )  b ≠ 0.
  x   e
x a x a
g( x ) f (x) − 5
Prove that lim  7. If xlim
→4
= 1, find xlim
→4
f(x).
x a f (x) x−2
f (x)
4. Prove that the following limits donot exist : 8. If lim 2 = 1, find
x 2 x
1
lim tan x f (x)
(i)  (ii) lim (2) x (i) lim f(x) and (ii) lim
x x→0 x 2 x 2 x
ps
2 tan 2 x
el

Concept Problems C
eh

je

9. Find xlim
x
1 
lim   ln x 
iit

(e)
→a2 |x − a| x
x 0 
@

10. Find the following limits by inspection.


(f) lim (x + x3)
x x
(a) lim
x 0 ln x x3
 3x 2
 1  x 1
x3 11. Evaluate lim  2 
(b) lim x   2 x  1 
x  e  x  

(c) lim (cos x)tan x 12. If f(x) = [x2 + 1][x + 1], where [.] denotes the greatest integer
x(  / 2) 
(d) lim (ln x) cot x
x0
function, find lim
x→1
f(x).

METHODS OF EVALUATING LIMITS of the given expression otherwise we repeat the process till
we get rid of the indeterminate form (0/0).
x3
1.6 Factorisation and Cancellation of Example 1: Find lim .
x2  9 x 3

Common Factors Solution: Here, the denominator tends to zero as
If f(x) and g(x) are polynomials such that f(a) = g(a) = 0, then x → 3 and the numerator also tends to zero. But since x2 – 9
= (x – 3) (x + 3), we have
f (x)
(x – a) is a factor of both f(x) and g(x). Now to solve lim ,
x →a g ( x ) x 3 x 3 1 1
lim  lim  lim 
we cancel the common factor (x – a) from both the numerator x 3 x  9 2 x  3 ( x  3)( x  3) x  3 x  3 6

and denominator, and again put x = a in the given expression. In the solution of this problem we cancel x – 3, and one may
If we get a meaningful number, then the number is the limit think that this is illegitimate since x → 3, and the division by

FREE BOOKS FOR JEE & NEET =>(@iitjeeadv)


1.24 Differential Calculus for JEE Main and Advanced

zero is not allowed. But this is not the case here : the functions x2  4
x 3 Example 4: Find lim  .
y 2 and y  1 coincide identically for all x ≠ 3, and x→2 cos x
x 9 x 3 4
Solution: We put x – 2 = z, i.e. x = z + 2, and as
the definition of a limit of a function for x → a, does not involve x → 2 we have z → 0.
the value of that function at the point x = a itself, and therefore z(z  4) z ( z + 4)
lim x  4
2

the limits of the above functions as x → 3 are equal to each lim lim
x→2  = z 0  = z→0 π
other. The essence of this transformation is that the limit of the cos x cos (z  2) − sin z
4 4 4
new function is found easier than that of the original function. π
x2 + x − 6 z ( z + 4)
z
Example 2: Evaluate lim π lim 4 16
x →−3 x+3 = zlim = – . lim (z + 4) = – .
Solution: We cannot apply direct substitution because →0
− sin z
π 4 z→0 sin π z z→0 π
the limit of the denominator is 0. 4 4

x2  x  6 x →−3
(
lim x 2 + x − 3 = 0 ) 1.
Note:
xn – an = (x – a)(xn–1 + xn–2a + xn–3a2 + xn–4a3 +...
x 3 lim ( x + 3) = 0


+ xan–2 + an–1)


x →−3

( )
where n is even or odd positive integer.
lim x 2 + x − 6 = 0
x  x   x →−3 2. xn + an = (x + a)(xn–1–xn–2a + xn–3a2 – xn–4a3 + ...
x lim ( x + 3) = 0 + (–1)n–1an–1)

x →−3 where n is odd positive integer. This formula is not applicable
Direct substitution fails here. when n is even.
ps
Because the limit of the numerator is also 0, the numerator 5
(1  x )3  1
Example 5: Find lim .
el
and denominator have a common factor of (x + 3). Thus, for x0 x
all x ≠ – 3, we can cancel this factor to obtain Solution: Let us put 1 + x = y5. As x → 0, y → 1.
eh

x  x   ( x  3)( x  2) Then we have


= x – 2, x ≠ – 3
je

=
x x 3 5
(1  x )3  1 = lim y  1
3
iit

It follows that lim y 1 y  1


5
x0 x
@

x2 + x − 6
lim = xlim ( x − 2) = −5. y2 + y + 1 3
x →−3 x+3 →−3 lim =
Although correct, the second equality in the preceding
y →1 y + y +y + y +1
4 3 2
5
computation needs some justification, since cancelling the factor x3  x 2  x  1
x + 3 alters the function by expanding its domain. However, Example 6: Find lim
x 1 x 3  x 2  x  1
the two functions are identical, except at x = – 3. From our previous Solution: Here we have an indeterminacy of the form
discussions, we know that this difference has no effect on the 0/0. Let us factorize the numerator and the denominator of
limit as x approaches – 3. the function
x 6  24 x  16
Example 3: Find lim x3  x 2  x  1 x 2 ( x  1)  ( x  1)
x→2
x 3  2 x  12 lim  lim
x 1 x 3  x2  x  1 x 1 x 2 ( x  1)  ( x  1)
lim x  24 x  16
6
Solution: x→2
x 3  2 x  12 ( x  1) ( x  1)  lim x  1  0  0
2
 lim x 1 x  1
.
x 1 ( x  1) ( x  1) 2 2
lim ( x  2)( x  2 x  4 x  8x  16 x  8)
5 4 3 2
= x→2
( x  2 x  6)( x  2)
2
x 3  1000
Example 7: Find lim
=
168
= 12 x 3  20 x 2  100 x
x 10

14 Solution: This is also an indeterminacy of the form 0/0.


Substitution for Limits x → a We have
Suppose that lim f(x) =  x 3  1000
x →a lim 3
Let x = a + t, then f(x) = f(a + t). x 10 x  20 x 2  100 x
If x → a then t → 0 and f(a + t) → , and we write
( x − 10)( x 2 + 10 x + 100)
lim f (a  t ) = . = lim
x a x →10 x (x − 10) 2

FREE BOOKS FOR JEE & NEET =>(@iitjeeadv)


Limits 1.25

x 2  10 x  100 1 1
= lim = – lim =– 2
x 10 x ( x  10) y 2x ( x  y  2) x
Here, y is a variable, so that it might be thought that we are
The numerator of the fraction tends to 300 and the denominator
dealing with functions of two variables. However, the fact that
tends to zero. Consequently, the fraction in question is an
x as a variable plays no role in this problem; for the moment, x
infinitely large quantity and
can be considered a constant.
x 3  1000
lim does not exist. 2 x  23 x  6
x 10 x 3  2 x 2  100 x Example 11: Evaluate lim
x 2 2 x / 2  21 x

lim
x 3  3x 2  x  3 2 x  23 x  6  0
Example 8: Find x→−3 Solution: lim x /2 1 x  form 0 
x x 6
2
x 2 2 2  
Solution: At the point x = – 3 both the numerator and 22 x  8  6.2 x
= lim
the denominator turn into zero. x 2 2x / 2  2
We have x2 + x – 6 = (x + 3) (x – 2) and
(2 x  2)(2 x  4)
x3 + 3x2 – x – 3 = (x + 3) (x2 – 1), and hence, on cancelling = lim
the factor (x + 3), we obtain
x 2 2x / 2  2
(2 x  2)(2 x / 2  2)(2 x / 2  2)
lim x  3x  x  3 = lim x − 1 = ( −3) − 1 = – 8
3 2 2 2
= lim
x→−3
x2  x  6 x→−3
x−2 −3 − 2 5 x 2 2x / 2  2
In the same manner we can find = 2 × 4 = 8.
ps
x 3  x 2 log x  log x  1
lim x  3x  x  3 dne, lim x  3x  x  3 = 0.
3 2 3 2
el
Example 12: Evaluate lim
x 1 x2 1
x→2 2
x x 6 x→1 2
x x 6
eh

Solution: The given limit = lim


 x  1   x  1 log x
3 2
je

Example 9: Evaluate x 1 x2 1
lim  1  2 (2 x  3)  ( x − 1) ( x )
iit

x→2 
2
+ x +1 − ( x − 1)( x +1) logx
 x  2 x 3  3x 2  2 x  = lim
@

x→1 ( x − 1)( x +1)


Solution: We have
 x  1  x 2 + x +1   x +1 logx 
lim  1  2 (2 x  3)  = lim
x→2   x1  x  1 x +1
 x  2 x 3  3x 2  2 x 
12 +1+1  1+1 log1
lim  1  2 (2 x  3) 
3
= = .
= x→2
 x  2 x ( x  1)( x  2)  1+1 2
3x 2  ax  a  3
lim  x ( x  1)  2(2 x  3) 
= x→2 Example 13: If the limit lim exists,
 x ( x  1)( x  2)  find a and the limit. x 2 x2  x  2
  Solution: We see that the denominator → 0. For
lim  x  5x  6 
2
= x→2 0
 x ( x  1)( x  2)  the limit to exist, we must have the   form. Hence the
0
numerator must → 0, i.e.
lim  ( x  2)( x  3)  lim 3x2 + ax + a + 3 = 0
= x→2   x2
 x ( x  1)( x  2) 
⇒ 12 – 2a + a + 3 = 0 ⇒ a = 15.
lim  x  3  = – 1 3x 2  ax  a  3 2 x 2  15x  18
= x→2 Now lim = lim
 x ( x  1)  2 x 2 x x2
2 x 2 x2  x  2

 1 
3( x  2)( x  3)
1 1  = lim  1 .
Example 10: Find lim     x 2 ( x  2)( x  1)
y 2  y  2  x  y  2 x  
f (x)
xx2y 1 Example 14: If lim  2 then evaluate the
lim
Solution: y · x2 x0
2 x ( x  y  2) y  2 following limits, giving explicit reasoning.

FREE BOOKS FOR JEE & NEET =>(@iitjeeadv)


1.26 Differential Calculus for JEE Main and Advanced

 f (x) 
(i) lim  f ( x )  , (ii) lim  ⇒ lim [A] = 0 and lim [A] = – 1
where [ . ] x0
x0  x 

x→ 0
x0
 ⇒ The given limit does not exist.
denotes the greatest integer function.
 f (x)  Example 15: Discuss the behaviour of
Solution: (i) Let l = lim f  x   = lim  2 · x 2 
x0  x 
x0
a 0 x m + a1x m +1 + .....a k x m + k
φ(x) =
Now argument of G.I.F is tending towards zero and b0 x n + b1x n +1 + .... + b x n + 
f (x)
from positive side as lim  2 and x2 → 0+ where a0 ≠ 0, b0 ≠ 0 as x tends to 0 by positive or negative
x0 x 2
f (x) 2 values.
we have · x → 0+ ⇒ l = 0.
x2 lim φ(x) = 0.
Solution: If m > n, x→0
 f (x)   f (x)  lim φ(x) = a /b .
If m = n, x→0
(ii) We write lim  = lim  2 · x 
x0  x 
0 0
 x 0  x 
If m < n and n – m is even, φ(x) → ∞ or φ(x) → –∞ according
and assume A = f ( x ) · x as a0/b0 > 0 or a0/b0 < 0.
x2 If m < n and n – m is odd, then φ(x) → ∞ as x → 0+ and φ(x)
Now x → 0 ⇒ A → 0+
+ → –∞ as x → 0–, or φ(x) → –∞ as x → 0 and φ(x) → ∞ as
and x → 0– ⇒ A → 0– x → 0–, according as a0/b0 > 0 or a0/b0 < 0.

Concept Problems E
ps

el
eh

1. Evaluate the following limits:


(c) lim f ( x ) (d) lim f ( x )
x →−3 x →0
lim x  3x  9 x  2 ( x  1) 2
3 2
je

(i) (ii) lim


x→2
x3  x  6 ( x  1) 2  1 [x ]  1
2
  
x 0
iit

4. Evaluate limit where [.] denotes the greatest


(x  h)  x 2 2 x1 x2 1
1 t
@

(iii) lim (iv) lim integer function.


h 0 1  t5

h t1
2. Evaluate the following limits:
lim
x 2  9 x  20
5. Evaluate x→ where [.] denotes
(i) | y 1 | y  1 5 x [ x ]
l im
y 1
| y 1 | y 1 the greatest integer function.

x 3  27 6. Evaluate the following limits where [.] denotes the greatest


(ii) lim integer function :
x 3 
| x  3 | ( x  3)
x  [x] x  [x]
(i) lim (ii) lim
Let f(x) = x + x − 2 find
2
3. x 2 
x2 x 2 
x2
x + 2x − 3
2

(iii) lim
[x ]  [x]
2 2
(iv) [x3 ]  x3
(a) lim f ( x ) (b) lim f (x) lim
x →−2 x →1 x 1 x2 1 x 10 [x]  x

Practice Problems D

7. Evaluate the following limits:
x 7 − 2x 5 + 1
(i) lim x − 3x + 2
4 (iii) lim
x →1 x 3 − 3x 2 + 2
x→1 x 5 − 4 x + 3

(iv) lim x − 6 x − 27
4 2
x 3 − 4 x 2 + 5x − 2
(ii) lim
x →1 ( x 2 − 1) 2 x →−3 x 3 + 3x 2 + x + 3

FREE BOOKS FOR JEE & NEET =>(@iitjeeadv)


Limits 1.27

8. Evaluate the following limits :


x 3 + cx 2 + 5x + 12
lim exists. Also find the corresponding
t t −t 2
x →3 x 2 − 7 x + 12
(i) lim − 4
t →1 t + 1 t −1 limit.
10. Evaluate the one-sided limits in the following :
(ii) 2 sin 2 x + sin x − 1
lim
x →π / 6 2 sin 2 x − 3 sin x + 1 lim x 2[x 2 ]
(i) x→ 2
(1+ | x − 2 |)
1 − cot 3 x
(iii) lim
x →π / 4 2 − cot x − cot 3 x lim x (e[ x ]+|x| − 2)
(ii) x→0
[ x ]+ | x |
9. Find a number c so that where [.] denotes the greatest integer function.


1.7 Rationalization  2  2
= lim   = 2 = 1.
x→0  1 + x + 1 − x 
If in any limit, the denominator or numerator involves the
radical sign then we can rationalize the irrational expression by
multiplying with their conjugates to remove the indeterminacy. (ii) We have lim
 (2 x − 3) x − 1

( ) 
x→1  
2x 2 + x − 3
4 − 15x + 1  
ps
Example 1: Evaluate lim .
2 − 3x + 1
( ) 
x→1
 (2 x − 3) x − 1
el

= lim 
lim 4 − 15x + 1 x→1 
(2 x + 3)( x − 1) 
eh
Solution: x→1
2 − 3x + 1  

( )
je

lim (4 − 15x + 1)(2 + 3x + 1)(4 + 15x + 1)


 (2 x − 3) x − 1 
=  
iit

= lim
(2 − 3x + 1)(4 + 15x + 1)(2 + 3x + 1)
( )( )
x→1
x→1 
(2 x + 3) x − 1 x +1 

 
@

(15 − 5x )
× 2 + 3x + 1 =
5
= lim
x→1
(3 − 3x )  
4 + 15x + 1 6
 2x − 3 
= lim
x→1 
(
(2 x + 3) x + 1  )

Example 2: Evaluate the following limits:
 
lim 1+ x − 1− x
(i) x→0 −1 −1
x = = .

(5)(2) 10

(ii) lim
(2 x − 3) ( x −1 ) 1− x 
Example 3: Evaluate lim sin −1  
2x + x − 3  1− x 
x→1 2
x →1
0
Solution: (i) The given limit takes the form when
0 
−1 1 − x

x → 0. Rationalizing the numerator, we get
Solution: lim sin  
1+ x − 1− x x →1  1 − x 
lim
x→0
x  1− x 
  = sin–1  lim 
x →1 1 − x 
lim  1 + x − 1 − x × 1 + x + 1 − x 
= x→0
 x 1 + x + 1 − x   1− x 
  = sin–1  lim 
 x →1 (1 − x )(1 + x ) 
lim  (1 + x ) − (1 − x )
= x→0 
 (
 x 1+ x + 1− x ) 


= sin–1  lim
1  1 π
= sin–1 = .
 x →1 1 + x  2 6
  x−2
lim  2x  Example 4: Evaluate xlim
( )
= x→0
x 1+ x + 1− x 
→2 +

x −4 + x−2
2
 

FREE BOOKS FOR JEE & NEET =>(@iitjeeadv)


1.28 Differential Calculus for JEE Main and Advanced

x−2 1 1
Solution: xlim = = – .
→2 +

x −4 − x−2
2 4+4+4 12
3
x2 − 23 x +1
x−2 . ( x − 4 + x − 2)
2 Example 6: Calculate lim
= lim x →1 ( x − 1) 2
x → 2+
x − 4 − x − 2 ( x − 4 + x − 2)
2 2
3
Solution: We substitute x = t
( x − 2)( x − 4 + x − 2 ) 2 Then, for the variable t, the expression under the limit sign
lim can be written in the form
=
x → 2+ ( x 2 − 4) − ( x − 2)
t 2 − 2t + 1
= lim ( x − 2)( x − 4 + x − 2 ) ( t 3 − 1) 2
2

x→2 +
x2 − x − 2 The number to which the new variable t tends, as
x → 1, can be found as the limit of the function
= lim ( x − 2)( x − 4 + x − 2 )
2

x→2 +
( x − 2)( x + 1) t(x) = 3 x as x → 1,
i.e. lim t ( x ) = lim 3 x = 1
= lim x 2 − 4 + x − 2 = 0. x →1 x →1

x→2 +
x +1 Thus we have
t 2 − 2t + 1 ( t − 1) 2
Note: lim = lim
t →1 ( t 3 − 1) 2 t →1 ( t − 1) 2 ( t 2 + t + 1) 2
n −1 n −2 1 n −3 2 n −1
ps
1 1
(x n − a n )( x n + x n an + x n an + ..... + a n ) = ( x − a) = lim
1
=
1
el
t →1 ( t 2 + t + 1) 2
9
1 1 n −1 n −2 1 n −3 2 .
eh

(x n + a n )( x n − x n an + x n an − ...) = ( x + a ) if n is odd x − 2a + x − 2 a
Example 7: Evaluate x lim
je

→ 2a +

3 (7 − x ) − 2 x 2 − 4a 2
iit

Example 5: Evaluate lim x − 2a + x − 2a 


x →−1 ( x + 1) 0
lim  form 
@

Solution: x → 2a +
x − 4a
2 2 0
Solution:
3 (7 − x ) – 2
x − 2a x − 2a
(7 − x ) − 8 = lim + lim
x → 2a + x → 2a +
= x − 4a
2 2
x 2 − 4a 2
(7 − x ) + (7 − x )1/3 .2 + 4
2/3
( x − 2a )( x + 2a )
= lim
( x + 1) x → 2a +
= ...(1) x 2 − 4a 2 ( x + 2a )
( x + 1) + ( x + 1)1/3 .2 + 4
2/3

( x − 2a )
+ lim
3 (7 − x ) − 2 ( x − 2a )( x + 2a )

x → 2a +

lim

x →−1 ( x + 1) x − 2a 1
= x lim +
( x + 1) → 2a +
( x − 2a )( x + 2a ).2 2a 4a
= – lim
x →−1 ( x + 1)(7 − x ) 2/3
+ (7 − x ) .2 + 4
1/ 3
x − 2a 1
= lim +
[from (1)] x → 2a +
x + 2a 2 2a 2 a
1 1 1
= lim
=0+ = .
x →−1 (7 − x ) 2 a 2 a
2/3
+ ( 7 − x ) .2 + 4 1/ 3

Concept Problems F

1. Calculate the value of the function at several points near x = 0 and hence estimate the limit
x of f(x) as x → 0.
f(x) =
x +1 −1

FREE BOOKS FOR JEE & NEET =>(@iitjeeadv)


Limits 1.29

2. Evaluate the following limits:


x 4 + x + x2 − 2
(i) lim 3. Evaluate lim
x→0
5− x − 5+ x x →−1 x +1

3− x 4. Evaluate lim x +1
(ii) lim x→ −1
x→9
4 − 2x − 2 6 x + 3 + 3x
2

3
x2 − 3 x 5. Find numbers a and b so that
(iii) lim
x→1 ( x − 1) ax + b − 1
lim = 1.
x →0 x
(iv) lim 5−x − 2
x→1
2−x −1

Practice Problems E

 x −3  x 2 − 2x + 6 − x 2 + 2x − 6
6. Evaluate lim log a  8. Evaluate lim .
x →3  x + 6 − 3 x →3 x 2 − 4x + 3
7. Evaluate the following limits :
9. Evaluate lim x + 7 − 3 2x − 3
1 + x + x 2 − 7 + 2x − x 2 x→ 2 3
x + 6 − 2 3 3x − 5
(i) lim
x 2 − 2x
ps
x→2
10. Evaluate the following limits :
1+ 2 + x − 3 x 2 + 8 − 10 − x 2
el
(ii) lim
(i) lim
x→ 2
x−2
eh

x →1
x2 + 3 − 5 − x2
(iii) lim x +1
8 + 3x − 2
je

3
x→ −1 4 x + 17 − 2 (ii) lim
iit

x → 0 4 16 + 5x − 2
1+ x − 1− x
(iv) lim
@

x →0 3 1 + x − 1− x 5
1 + x )3 − 1
(iii) lim .
x→0
(1 + x ) 3 (1 + x ) 2 − 1

1.8 Limit Using Expansion Series of x 3 2x 5


(vi) tan x = x + + + ........

Functions 3 15
x3 x5 x7
In this method basically we use the series expansion of sin (vii) tan–1x = x − + − + .......
x, cos x, tan x, log(1+x), ax, ex, etc. to evaluate the limit. 3 5 7
Following are some of the frequently used series expansions: 12 3 12.32 5 12.32.52 7
(viii) sin-1x = x + x + x + x + .......
3! 5! 7!
x 1n a x 2 1n 2a x 3 1n 3a
(i) ax = 1+ + + + .........a > 0 x 2 5x 4 61x 6
1! 2! 3! (ix) sec x = 1 + + + + ......
2! 4! 6!
x x 2 x3
(ii) ex = 1 + + + + ............ nx x2
1! 2! 3! (x) (1 + x)n = 1 + + n (n − 1) +.... for –1< x< 1, n ∈ Q.
1 2
x 2 x3 x 4  1 11 2 
(iii) ln (1+x) = x − + − + .........for − 1 < x ≤ 1 (xi) (1 + x)1/x = e 1 − x+ x + .....
2 3 4  2 24 
x3 x5 x7 Note: Using the above expansions, we can find other
(iv) sin x = x − + − + ....... expansion series. For example to find the expansion series of
3! 5! 7!
1 − cos 2 x
x2 x4 x6 sin2x, we write, sin2x = and use the expansion series
(v) cos x = 1 − + − + ...... 2
2! 4! 6! of cos x with x replaced by 2x.

FREE BOOKS FOR JEE & NEET =>(@iitjeeadv)


1.30 Differential Calculus for JEE Main and Advanced

Maclaurin's Theorem  x3   x3 
 x + 3 ........ −  x − 3 ! .......
x2 x3    
f(x) = f(0) + xf'(0) + f "(0) + f '" (0) + ... = xlim
→0
2! 3! x3
xn n 1 1 3
........ + f (0) + ........  +  x + ........
n! lim  3 3! =
1
+
1 1
= .
x→0 x3

3 6 2
Example 1: Expand sin x in powers of x.
1 − x
e x + ln 
 e 
Solution: Here f(x) = sin x, Hence f(0) = 0,
f'(x) = cos x, f'(0) = 1, Example 5: Evaluate lim
tan x − x

f"(x) = – sin x, f"(0)= 0, x→0

f'"(x) = – cos x, f"'(0) = – 1 1 − x
e x + ln 
 e 
.............

 nπ  Solution: lim
fn(x) = sin  x +  fn(0) = sin nπ x→0 tan x − x
 2 2
e x + ln(1 − x ) − ln e
nπ n = lim [form 0/0]
x3 x5
x sin x→0 tan x − x
Thus sin x = x – + − .... + 2 + ...
3! 5! n! x 2 x3  x 2 x3 
1+ x + + + ... +  − x − − − ... −1
Example 2: Expand ln(cos x) in powers of x. 2! 3!  2 3 
= lim
ps
Solution: Here f(x) = ln(cos x), x→0  x 3
2x 5

f'(x) = – tan x = – t, say,  x + 3 + 15  − x


 
el

f"(x) = – sec2 x = – (1 + t2),


eh

1
f'"(x) = – 2 tan x sec2 x = – 2t (1 + t2), − x 3 (1+ terms containing x and its higher powers)
f(4)(x) = – 2 (1 + 3t2) (1 + t2) = – 2 (1 + 4t2 + 3t4), 6
= xlim
je

→0 1 3
f(5)(x) = – 2 (8t + 12t3) (1 + t2) x (1+ terms containing x and its higher powers)
iit

= – 2(8t + 20t3 + 12t5), 3



f(6)(x) = – 2 (8 + 60t2 + 60t4) (1 + t2) 1
@

=– .
= – 2(8 + 68t2 + 120t4 + 60t6) 2

Hence, 5 sin x − 7 sin 2x + 3 sin 3x
Example 6: Evaluate lim
f(0) = 0, and x →0 x 2 sin x
f'(0) = f(3)(0) = f(5)(0) = ...= 0, also
5 sin x − 7 sin 2x + 3 sin 3x
f"(0) = – 1, f(4)(0) = –2, f(6)(0) = – 16. Solution: lim
x →0 x 2 sin x
x2 x4 x6
Hence ln(cos x) = – −2 − 16 .............  x3   ( 2 x )3   (3x )3 
2! 4! 6! 5 x − + .... − 7  2 x − + .... + 3  3x − + ....
 3!   3!   3!! 
= lim
ex − 1 − x x →0
2
x x −
x3
+ ....

Example 3: Evaluate lim
x→0 x2  3! 

e −1− xx 5x 3 56 x 3 81x 3
Solution: lim − + −
x→0 x2 3! 3! 3!
= lim
 
x →0  x 3 
x2 x 3 1 − + ...
 1 + x + 2 ! ....... − 1 − x  3! 
 
= lim
x→0 x2 −5 + 56 − 81
= = −5 .
1 x x2 1 3!
= xlim + +
→ 0 2! 3!
............ = . x2
4! 2 n (1 + x ) − sin x +
Example 7: Evaluate lim 2
Example 4: Evaluate tan x − sin x x→0 x tan x sin x
x3 x2
tan x − sin x n (1 + x ) − sin x +
Solution: lim Solution: lim 2
x→0 x3 x→0 x tan x sin x

FREE BOOKS FOR JEE & NEET =>(@iitjeeadv)


Limits 1.31

 x 2 x3   x3 x5  x2 x 7 − 2x 5 + 1
 x − 2 + 3 ..... −  x + 3! + 5! + ..... + 2 Example 11: Find lim
x→1 x 3 − 3x 2 + 2
.
   
= xlim
→0 tan x i
si n x 0
x3 . . Solution: This is of the form if we put x = 1.
x x 0
1 1 1 Therefore we put x = 1 + h and expand.
= + = .
3 6 2 x 7 − 2x 5 + 1 (1 + h )7 − 2(1 + h )5 + 1
lim = lim
cos x 3 − 1 + n (1 + x 6 ) x→1 x − 3x + 2
3 2
h→0 (1 + h )3 − 3(1 + h ) 2 + 2
Example 8: Evaluate lim
x 2 (e x − 1 − x 2 )
2
x →0 (1 + 7 h + 21h 2 + ...) − 2(1 + 5h + 19h 2 + ...) + 1
= lim
cos x − 1 + n (1 + x )
3 6 h→0 (1 + 3h + 3h 2 + ...) − 3(1 + 2h + h 2 ) + 2
Solution: lim
x 2 (e x − 1 − x 2 ) −3h + h 2 + ...
2
x →0
= lim
x6 x12 h→0 −3h + ....
1− + ........ − 1 + x 6 −
2 2 −3 + h + ...
= lim = lim = 1.
x →0  x2 x4  h→0 −3 + ....
x 2 1 + + ....... − 1 − x 2 
 1 2  a cos x + bx sin x − 5
Example 12: If lim exists, find a,
1 b and the limit.
x →0 x4
+ .....  0
2 Solution: As x → 0, x4 → 0 the limit must be in  
= lim =1.  0
x→0  1  form. Hence lim acos x + bx sin x – 5 = 0.
ps
 2 + ....... x→0
⇒ a – 5 = 0 ⇒ a = 5.
el

esin x − sin x − 1  x2 x4   x3 
5 1 − + ....... + bx  x − ...... − 5
eh

Example 9: Find lim


x →0 x2  2 2   3 
Limit = lim
je

x →0 x4
esin x − sin x − 1
 5 2  5 b 4
iit

Solution: lim
x →0 x2  b −  x +  −  x + ....
2 24 6 
@

 sin x sin 2 x sin 3 x  = lim


1 + 1 + + + ..... − sin x − 1 x →0 x4
 2 3  4
For limit to exist, x must cancell from the numerator. Hence
lim
= x→ 0 x2 we assume the coefficients of all powers of x
 sin x   1 sin x 
2
1 1 5 5
= lim   + + ... = = . less than 4 to be zero ⇒ b − = 0 ⇒ b = .
x→0  x    2 3  2 2 2 2
 5 b 4
(7 + x )1/3 − 2  −  x + ........
Example 10: Evaluate lim 24 6  5 b
x →1 x −1 Now the limit = lim = −
x →0 x4 24 6
Solution: Put x → 1 + h
5 5 −5
(8 + h )1/3 − 2 ∴ Limit = − = .
lim 24 12 24
h→0 h
Ae x − B cos x + Ce − x
 h
1/ 3 Example 13: If lim = 2 , find A,
x →0
2 . 1 +  −2 B and C. x sin x
 8
= lim Solution: The given limit is equal to
h→0 h
 x2   x2   x2 
 1 1   h
2  A 1 + x + ....... − B  1 − + ....... + C  1 − x + .......
 − 1     2   2   2 

 1 h 3 3   8  lim
x →0  x3 
2 1 + . + + ....... − 1 xx − .......
 3 8 1 . 2   3 

 

= lim  A + B + C 2
( A − B + C) + ( A − C) x +   x
h→0 h  2
= lim
1 1 x →0  x2 
= lim 2 × = . x 2 1 − ......
h→0 24 12  3 

FREE BOOKS FOR JEE & NEET =>(@iitjeeadv)


1.32 Differential Calculus for JEE Main and Advanced

For limit to exist, x2 must cancel from the numerator. Hence


 1 11 
we assume A – B + C = 0 ...(1) = e 1 − x + x 2 + .....
 


and A – C = 0
...(2) 2 24
A+ B+C 1
∴ Limit = = 2. ...(3) (1 + x )1/ x − e + ex
2

2 Now lim
On solving these three equations, we get A = 1, B = 2, C = 1. x →0 x2
1 11 2 1
Example 14: Show that e(1 −
x+ x + ....) − e + ex
2 24 2 11e
1
(1 + x )1/ x − e + ex 11e = lim 2 = .
x →0 x 24
lim 2 = .
x →0 x2 24 f (x)
Example 15: Let f(x) be a function such that lim = 1.
x →0 x
Solution: Let y = (1 + x)1/x Find the values of a and b such that
1 x (1 + a cos x ) − b sin x
∴ ln y = ln(1 + x ) lim =1.
x x →0 {f ( x )}3
1 1 2 1 3 
=  x − x + x − ... x (1 + a cos x ) − b sin x
x 2 3 Solution: Since, lim =1
x →0 {f ( x )}3
1 1
= 1− x + x 2 − ..... ,   x2 x4    x3 x5 
2 3 x 1 + a 1 + +  − b x − + − ....
1 1   2 ! 4 !    3! 5! 
1− x + x 2 −...... ⇒ lim =1
Now y = e
ps
2 2 x →0 {f ( x )}3
1 1  a b  a b
el
1− x + x 2 −......
2 2 x (1 + a − b) + x 3  +  + x 5  −  + ...
= e. e  2! 3!  4! 5!
eh

⇒ lim =1
  1 1  x →0 {f ( x )}3
= e 1 +  − x + x − ...... + .....
2
je

  2 3  (1 + a − b)  a b   a b
+  +  + x 2  −  + ...
iit


2  2 ! 3!  4! 5!
 x
2
1 1 1 2  ⇒ lim =1
+  − x + x − ...... + .... x →0  f (x) 
3
@

2 2 3 
  
 x 
 1  1 1 2 
= e 1 − x +  +  x + ..... Since the limit exists we must have 1 + a –b = 0 and –
a b
+
 2  3 8   =1 ⇒ –3a + b = 6 2 ! 3!
Solving these, we get a = –5/2 and b = –3/2.

Concept Problems G

1. Evaluate the following limits :
e x sin x − x (1 + x )
1 (iii) lim
3
1+ x −1− x x→0 x3
(i) lim 3
x →0 x2 3
1 + 3x − 1 + 2 x
e x sin x − x − x 2 (iv) lim
(ii) lim 2 x→0 x2
x →0 x + x ln(1 − x )
3 tan x − 3x − x 3
x2 3. Evaluate lim
cos x − 1 + x →0 x5
(iii) lim 2
x →0 x3
sin −1 x − tan −1 x
2 x − ln(1 + 2 x ) 4. Evaluate lim
(iv) lim x→ 0 x3
x →0 x2
2. Evaluate the following limits : (2 − x )(e x − x − 2)
5. Evaluate lim
2 x - sin -1 x x →0 x3
(i) lim
2 x + tan -1 x
x →0 6. For what values of constants C and D is it true that
3 sin x − sin 3x lim( x −3 sin 7 x + Cx −2 + D) = −2
(ii) lim x →0
x→0 x − sin x

FREE BOOKS FOR JEE & NEET =>(@iitjeeadv)


Limits 1.33

Practice Problems F


7. Evaluate the following limits :
sin x − x + x 3 / 6 − x 5 / 120
(1 + x 2 ) − (1 − x )
(ii) lim
(i) lim x →0 (sin x )7
x→0 (1 + x 3 ) − (1 + x )
e x cos x − 1 − x
(iii) lim
3
1+ x2 − 4 1 − x2 x→0 sin( x 2 )
(ii) lim
x→0 x + x2 x2
x cos3 x − ln(1 + x ) − sin −1
31+ 3x − 1 2
(iii) lim (iv) lim
x→0 (1 + x ) 50
− 1 − 50 x
x →0 x3
1 1+ x
ln 2 (1 + x ) − x 2 2 sin x + ln − 3x
(iv) lim 10. Evaluate lim 2 1− x
x →0 x3 x →0 x 5

8. Evaluate the following limits : x4 3


m
1 + ax − 1 + bx
n x 3
e4 − sin 2 x 2
(i) lim (m, n ∈ N) 11. Evaluate lim
x →0 x x →0 x7
12. Find the values of a and b so that
m
1 + ax n 1 + bx − 1
(ii) lim (m, n ∈ N) (1 + ax sin x ) − b cos x
ps
x →0 x lim
x→0 x4
el
3
1 + 2x − 4 1 + 9x may tend to a definite limit and also find limit.
(iii) lim
eh

x →0 x 13. For what values of the constants a, b is ,


1− 1−
2  sin 3x a 
je

lim  + 2 + b = 0 ?
x→0  x 3 x 
iit

4 4 1 − x 2 − 4e x
2

(iv) lim
x →0 tan −1 x − x 1  1 1 + ax 
@

14. If lim 3
−  exists and has the value
9. Evaluate the following limits : x→0 x  1 + x 1 + bx 
x3 x5 1 2 3
sin x (1 − cos x ) − + equal to , then find the value of − + .
(i) lim 2 8 a  b
x →0 x7

1.9 Standard Limits e x −1


In particular lim =1
Following are some basic limits which are used frequently x→0 x
in solving the limits. ln(1+ x )
sin x tan x (iii) lim =1
(i) lim = 1 = lim x→0 x
x→0 x x→0 x
log a (1+ x )
−1 −1 lim = loga e, ( a > 0, a ≠ 1)
lim tan x = lim sin x
= x→0 x→0 x
x→0 x
x
(1 + x ) n − 1
[ where x is measured in radians ] (iv) lim =n
x →0 x
1 − cos x (1 − cos x )(1 + cos x )
Also, lim = lim
2
x 2 (1 + cos x ) xn − an
x = n a n −1 .
x→0 x→0
lim
2 x →a x−a
(sin 2 x )  sin x  1
= lim 2 = lim  
x (1 + cos x )  x (1 + cos x)
x→0 x→0
Study Tip
1
= . (i) These limits help in finding limits in 0/0 form and most of
2
them are based on x approaching 0. In case, if x approaches
a x −1 a, we use the substitution x = a + t so that t approaches 0
(ii) lim = ln a (a > 0).
x→0 x when x approaches a.

FREE BOOKS FOR JEE & NEET =>(@iitjeeadv)


1.34 Differential Calculus for JEE Main and Advanced

(ii) These formulae are also applicable when a function, say πx


tan
180 . πx
f(x) has been replaced for x. But it must be ensured that
as x approaches a, f(x) must approach 0.
πx 180 .
sin f ( x )
180 π
For instance, lim
x →a f (x)
= 1, if f(x) approaches 0 as = lim =
x→0 x 180
x approaches a.
e3 x − 1
Example 2: Evaluate lim
Caution x→0 x/2

sin f ( x ) e −1
3x
e3 x − 1
Solution: lim = lim 2 × 3 = – 6.
But lim where f(x) and g(x) approach 0 as x x→0 x/2 x→0 3x
x →a g( x )
approaches a, should not be immediately taken as 1. sin 2 x
Example 3: Compute lim
sin 2x sin 2 x x→0 sin 3x
For example, lim = lim .2 =2 sin 2 x
x→0 x x→0 2x Solution: We have x→0lim
sin f ( x ) sin 3x
Further, lim should not be taken as 1 when sin
x →a f (x)  sin 2 x 2 x 3x 
= lim  . .
f(x) approaches 0 as x approaches a, but f(x) itself does not x→0  2 x 3x sin 3x 
approach 0.  sin 2 x  2  3x 
sin(π − x ) =  lim  . .  lim ,x≠0
≠ 1, since π – x does not approach  2 x→0 2 x  3 3x→0 sin 2 x 
ps
For example, lim
x→0 π−x −1
2  sin 3x  2
el
0 as x approaches 0. In fact, the limit is equal to 0. =1. . lim = ×1= .
3 3x→0 3x  3
eh

Why Radian Measure Is Used


x3 − 8
je

Throughout calculus, angles are measured in radians, as is Example 4: Evaluate lim


x→2 x 2 − 4
customary in calculus. If we measured angles in degrees instead,
iit

the formulas for the limits of the trigonometric functions would Solution: The given expression is of the form
@

be more complicated. Each formula would have an extra factor, x 3 − (2)3 x 3 − (2)3 x 2 − (2) 2
π/180, as we will now show. x − (2)
2 2 =
x−2
/
x−2
Earlier, it was shown that when angles are measured in radians,
x3 − 8 x 3 − ( 2) 3 x 2 − (2) 2
sin θ ⇒ lim = lim / lim
lim = 1. x→2 x2 − 4 x→2 x−2 x→2 x−2
θ→0 θ
When angles are measured in degrees, this limit is not 1. xn − an
Let sin θ denote the sine of an angle of θ degrees. The following = 3(22)/(2(21)) (using lim = nan–1 )
x →a x−a
table suggests that the limit is much smaller (angles measured = 12 / 4 = 3.
in degrees; data to four significant figures): 1 + x1/ 3
Example 5: Find xlim .
θ 10 5 1 0.1
→−1 1 + x1/ 5

(x )
5
sin θ 0.1736 0.08716 0.01745 0.001745 1/ 5
− ( −1)
Solution: We have lim = 5( −1) 4 = 5
sinθ x →−1 x1/ 5
− ( −1)
0.01736 0.01743 0.01745 0.01745
θ ( x1/3 )3 − ( −1)
and lim = 3 (–1)2 = 3
sin θ x →1 x1/ 3 − ( −1)
The data suggest that lim is about 0.01745. We can
θ

θ→0
x1/ 3 − ( −1) 5
find that the limit is precisely π/180. Hence the limit = lim = .
x →−1 x1/ 5 − ( −1) 3
tan x °
Example 1: lim Example 6: Evaluate
x→0 x
π a x + h + a x − h − 2a x
Solution: x ° = x radian lim , a>0
180 h→ 0 h2
πx  a h + a −h − 2 
tan x °
tan Solution: Limit = lim ax  
Now lim = lim 180 h→0  h2 
x→0 x x→0 x

FREE BOOKS FOR JEE & NEET =>(@iitjeeadv)


Limits 1.35

tan x − sin x
 a 2 h − 2a h + 1   2 
2 Example 12: Evaluate lim
x x a −1 x→0 x3
= lim a   = a  h 
h →0
 ahh2   
lim tan x − sin x
Solution: x→0
= ax ln2a. x3
a x − aa
Example 7: Evaluate lim , a > 0. tan x (1 − cos x )
x→ a x − a = lim
x→0 x3
Solution: Put x = a + h
2 x
a a+h − a a a a (a h − 1) lim tan x . 2 sin 2
Limit = lim = lim = aa lna = x→0
h →0 h h→0 h x3
2
 sin x   x
Example 8: Evaluate lim  . sin
x →0  x   tan x 1 2 = 1 .
= lim . 
x→0 x 2 x  2
sin x  sin x   
Solution: Since, 0 < <1,   = 0 in the 2 
neighbourhood of x = 0. x  x 
sin(π cos 2 x )
 sin x  Example 13: Evaluate lim
Hence lim 
x →0  x 
= 0. x →0 x2

 tan −1 x 
Solution: Here πcos x → π as x → 0.
2

With a similar reason, lim   = 0. We change πcos2x to πsin2x, so that it tends to 0.


x →0
 x  sin(π cos 2 x ) sin{π(1 − sin 2 x )}
lim = lim
ps
 tan x  x →0 x 2
x →0 x2
Example 9: Evaluate lim  .
x →0  x  
el

Solution: Since, 1 <


tan x
< 2 in the neighbourhood of sin(π − π sin 2 x ) sin(π sin 2 x )
= lim = lim
eh

x x →0 x2 x →0 x2
 tan x 
x = 0, 
tan x   sin(π sin 2 x ) π sin 2 x 
je

 = 1. Hence, xlim  x  = 1. = lim  × × 2 


 x  → 0   x →0  π sin 2 x x 
iit

 1
 sin −1x 
With a similar reason, lim   =1.
@

x →0 sin(π sin 2 x ) sin 2 x


 x  = lim × π × lim
x →0 π sin 2 x x →0 x 2
 −2x  = 1 × π × 1 = π.
Example 10: Evaluate lim 
x →0  tan x 
, where [.] denotes
greatest integer function.  x sin{x}
Example 14: Evaluate lim , where {x}
Solution: We know when x → 0 x→1 x −1
x −x denotes the fractional part of x.
⇒ <1 ⇒ > −1
tan x tan x Solution: As x → 1– , {x} can be replaced by 1– h,
−2 x where h is a small positive quantity.
⇒ –2 < < −1
tan x As x → 1+ , {x} can be replaced by h, where h is a small
positive quantity.
 −2x 
So, lim   = –2. x sin{x} 1− h
x →0  tan x 
∴ lim = lim sin(1 − h ) = – ∞
ln x − 1 x →1 −
x −1 h →0 − h +

Example 11: Evaluate lim .


x →e x − e x sin{x} sin h
lim = lim (1 + h )
ln x / e x →1+ x −1 h →0 +
h
Solution: Limit = lim
x →e  x  = 1×1=1
e  − 1

e  Since, L.H.L. ≠ R. H.L. the limit does not exist.
ln t cos x ln( x − a )
= lim (putting x/e = t) Example 15: Evaluate lim
t →1 ( t − 1) e x →a ln(e x − ea )
ln (1 + y ) 1 cos x ln( x − a )
Put t = 1 + y = lim = Solution: lim Put x = a + h
y→ 0 ey e x →a ln(e x − ea )

FREE BOOKS FOR JEE & NEET =>(@iitjeeadv)


1.36 Differential Calculus for JEE Main and Advanced

ln h 2
= cos a · lim   sin t  
h →0 a+h
ln(e − ea )  sin   2
 2.   2   . sin t   1
= lim .
ln h t 0  sin t  4.t 2 2
= cos a · lim  2 
h →0  a  e h − 1   
ln  e ·   · h
  h   Example 19: Find the limiting value of

ln h
tan 2 x − 2 sin x
= cos a · lim as x tends to zero.
h →0  e h − 1 x3
a + ln   + ln h sin 2 x − 2 sin x cos 2 x
 h  Solution: lim
x→0 x 3 cos 2 x
1
= cos a · lim = cos a. 2 sin x[cos x − cos 2 x ]
h →0 a ln(e h h )
+ +1 = lim
ln h ln h x→0 x 3 .1
3x
1 − cos 2 x 2 sin sin x
Example 16: Evaluate lim = 2 lim
2 2
x→0 x x→0 x2
1 − cos 2 x 3 sin
3x
sin x
Solution: f(x) = = 2 lim 2 . 2 = 3.
x x→0 3x x
ps
2 2
2
2 sin 2 x 2 | sin x |
= Example 20: Let a = min [x2 + 2x + 3, x ∈ R] and
el
= ,
x x

n
eh

 a r bn r .
sin x cos x
 sin x , if 0 < x < π / 2, b = lim , then find the value of
but |sin x| =  − sin x , if − π / 2 < x < 0 x 0 e x  e x r 0
je


iit

 sin x  Solution: a = minimum value of (x + 1)2 + 2


Hence f(0 ) = lim f ( x ) = lim  − 2

=− 2 ⇒
@

x →0 −
x →0  x  − a=2
sin 2 x 1
 sin x  b = lim = .
and f(0+) = lim f ( x ) = lim  2 x 0 2(e 2 x  1).2 x
x →0 
= 2. 2
x →0 +
x  +

2x
n cos x n n r n
Example 17: Evaluate lim 1 1
 a r bn r  22 r
.
x →0 4
1+ x2 −1 Now =  2r  2  =
r 0 2n r 0
n (1 + cos x − 1)(cos x − 1)
Solution: Limit = xlim
→0
(cos x − 1)
{(1 + x )
2 1/ 4
}x
−1 2 =
1 n
 4r =
1
[1 + 4 + 42 + ...... + 4n]
x 2 2 n
r 0 2n
1  1 1  4n1  1  4n 1  1
= 1     2 . =  = .
 2 2n  3  3.2n

4
e x  ln( x  e)
n cos(sin t ) Example 21: Evaluate lim .
Example 18: Evaluate lim 2
x 0 ex  1
t →0 t
  x 
n cos(sin t ) (e x  ln  e  1   
Solution: lim   e 
t →0 t2 Solution: lim
x 0  ex  1 
x  
 (sin t )   2  sin t    x 
n 1  2 sin 2   2 sin  
 2 
 lim 
2  
.  (e x  1) ln 1  ( x e)  
t 0  sin t  t2 = lim   
2 sin 2   x 0  x x 
 2 

FREE BOOKS FOR JEE & NEET =>(@iitjeeadv)


Limits 1.37

= 1 – lim
ln 1  ( x e) 
   cos 1 x ·    cos 1 x 
 x
x0 x = lim
e· x1
1 x 2
  cos 1
e

=1–
1
e
.
= lim
1    cos x 1

x1 2  1 x2
 n (cos 3x ) 2 sin x 
Example 22: Evaluate lim  . x x  . 1 cos 1 ( x )
x 0  x2 e e  = lim
x1 2  1 x2
Solution:
n (cos 3x ).2 sin x n (cos 3x ).2 sin x.e x put cos–1(–x) = θ ⇒ x = – cos θ



lim lim
x 0 x 2 (e x  e  x ) x 0 x 2 (e 2 x  1) 1  1 
= lim = lim
 n (1  cos 3x  1) 2  0 | sin  | 2  0 sin  

= lim   (cos 3x  1)
x→0  (cos 3x  1) 1
= .
2 sin x 1  2 π
  
x  e2 x  1  2  tan x  sin{tan 1 (tan x )}
  2 x  Example 25: Solve lim
 2x   x
 tan x  cos 2 (tan x )
2

  2 3x 
 2 sin 2 
ps
 Solution: Here, RHL
n  
 
(1 cos 3 x 1)
= lim 
tan x  sin{tan 1 (tan x )}
el
x→0  (cos 3x  1) 2x 2
= lim
tan x  cos 2 (tan x )
eh



2 sin x 1  x
   2
 e2 x  1  
je

x
 x   tan x  sin( x  )
iit

  = lim
tan x  cos 2 (tan x )


x

@

9 9 2
= 1 × (–1) × ×2×1=– . π
4 2 {∵ tan–1 (tan x) = x – π, when x > }

2
1− x
Example 23: Evaluate lim . sin x
x →1− (cos −1x) 2 1
tan x 1+ 0
= lim = = 1.
Solution: Put cos-1 x = y and x → 1– ⇒ y → 0+ x
 cos 2 (tan x )

1+ 0
2 1 
1− x 1 − cos y tan x
lim −1 = lim
x →1−
(cos x) y→0 y2 2 Now, LHL
Now rationalizing the numerator, we get tan x  sin{tan 1 (tan x )}
(1 − cos y) = lim
tan x  cos 2 (tan x )


= lim 2 x
y → 0 y (1 + cos y ) 2

1  cos y 1 1 1 1 tan x  sin( x )


= ylim . lim = . = . = lim
0 y 2 y 0 1  cos y 2 2 4 x
 tan x  cos 2 (tan x )
2

Example 24: Evaluate lim


sin 1    cos 1 x .  1 
as tan (tan x )  x , when x  
x 1
1 x 2
 2
     
Solution: sin x
sin 1    cos 1 x    cos 1 x  = lim
1
tan x =
1 0
1 0
=1
 x
lim  cos 2 (tan x )
x
  cos 1 1

x1
1 x2 2
tan x

FREE BOOKS FOR JEE & NEET =>(@iitjeeadv)


1.38 Differential Calculus for JEE Main and Advanced

tan x  sin{tan 1 (tan x )} 1


.
1
=
1
∴ lim = 1. =
2
.
x
 tan x  cos 2 (tan x ) 2 2
2
Hence, limit does not exist.
xh
(x  h) x x
Example 26: Evaluate lim Use of Substitution
h→0 h
Sometimes in solving limit problem we convert lim f(x) by
(x > 0). x →a

x  h  n (x  h) x n x substituting x = a + h or x = a – h as lim f(a + h) or lim
e e h→0 h→0
Solution: lim f(a – h) according to the need of the problem.
h→0 h
e e x − e3
 1
x  h  n (x  h)  x n x
x n x   Example 28: Compute lim
= lim e
 h 
x→3 x −3
h→0
Solution: Put y = x – 3. So, as x → 3, y → 0.
 es  1
= x
x
 lim e x − e3 e3 y  e3
s  0 s Thus lim = lim
x→3 x −3 y→0 y
x  h  n (x  h)  x n x
lim  e3 . e y − e3
h→0 h  = lim
y

y→0
where s = x  h  n (x  h)  x  n x
e y −1
  h  = e3 lim = e3 . 1 = e3 .
ps
x  h  nx  n 1     x nx y→0 y
x   x 
= x . hlim
el
→0 h ln tan x
Example 29: Find lim
eh

 x   / 4 1  cot x
x ( xh  x)
= x  lim  n x
h  0 Solution: Put x = t + π/4
je

h
x/h  ln tan( t   / 4)
iit

(x  h)  h lim
 lim  n 1    t 0 1  cot( t   / 4)
h0 x  x 
@

 1  tan t 

x  n x 1  ln  
= x   .  1  tan t 
 2 x x  = lim
t 0  cot t  1 
1  
1  cos x  cot t  1 
Example 27: Evaluate lim
x 0 x ln(1  tan t ) ln(1  tan t )
= lim + lim
t 0 2 tan t t 0 2 tan t
1  cos x
Solution: lim
x 0
1  tan t 1  tan t
x
1 2
1  cos x 1 = [1.1 + 1.1] = = 1.
= lim . 2 2
x 0 x 1  cos x
 π
cos  x + 
2 | sin( x / 2) | 1  6
= lim . Example 30: Evaluate lim
x 0 x → π / 3 (1 − 2 cos x ) 2 / 3
2( x / 2) 1 cos x
Now, we have 
1 sin( x / 2) 1 Solution: Putting x – z
LHL = lim . . 3
x 0 
2 x/2 1 cos x  
cos   z 
1 1 1  2 
= .  . = lim
2 2 2 z 0 (1  2 cos z  3 sin z ) 2 / 3

1 sin( x / 2) 1  sin z
and RHL = lim . . = lim
x 0 
2 x/2 1  cos x z 0 (1  cos z  3 sin z) 2 / 3

FREE BOOKS FOR JEE & NEET =>(@iitjeeadv)


Limits 1.39

z z xn x


− ex  n a
2 sin   cos   Solution: l = lim e
2 2 x→a x−a
= lim 2/3 2/3
z 0
  z   z  z  e x  n a e x (  n x   n a )  1 x ( n x −  n a )
 2 sin  2   sin  2   3 cos  2    
        = lim
x→a x ( n x   n a )
.
x−a
  z 1/ 3  z  h
21/ 3 sin    cos   (a + h) ln 1 + 
  2   2 −21/ 3 .0.1 a
= a . lim  a  = aa.
= zlim 2/3 = =0. h→0

( )
0
 h
 z 
2/3
z
 2 sin  2   3 cos  2   3
e x  n a − ea  n x
     m = lim
x→a x−a
(1  tan )
Example 31: Evaluate lim
(1  2 sin ) ea  n x e x  n a  a  n x  1
  / 4
= lim   . xn a − an x
x→a xn a  an x x−a
(1  tan )  0
Solution: Let P = lim  form 0 
x→0 (1  2 sin )   = aa . lim (a  h )  n a  a  n (a  h )
h→0
π h
Put θ = +h 
  
= aa (ln a − 1)
a/h
4 h
= aa lim
h→0
 n a   n 1  a 
 
 
1  tan   h  Now l = m ⇒ a = e2
ps
∴ P = lim  4 
h 0   Example 33: Let
1  2 sin   h 
el

4 
sin −1 (1 − {x}).cos −1 (1 − {x})
eh

 1  tan h 
f(x) = , then find lim f(x) and
1  2{x}.(1 − {x}) x0 

je

 1  tan h   0
lim f(x), where {x} denotes the fractional part of x.
= hlim  form 0 
iit

0  1 1    x0 
1 2  cos h  sin h 
 2 2 
@

Solution: We have f(x)


−2 tan h sin −1 (1 − {x}).cos −1 (1 − {x})
lim =
h →0 (1 − cosh) (1 + cos h ) 2{x}.(1 − {x})
(1 − tan h ) − sin h
=
(1 + cos h ) ∴ lim f(x) = lim f(0 + h)
x0  h→0
tan h
= –2 lim
h 0  sin 2 h  sin (1  {0  h}).cos 1 (1  {0  h})
1

(1  tan h )   sin h  = lim


h 0 2{0  h}.(1  {0  h})
 1  cos h 
sin h sin 1 (1  h ).cos 1 (1  h )
= –2 lim = lim
h 0  sin 2 h  h 0 2h .(1  h )
(1  cos h  sin h )   sin h 
 1  cos h  1
sin (1  h ) cos 1 (1  h )
= hlim . lim
1 0 (1  h ) h 0 2h
= –2 lim
h 0  sin h  (In second limit put cos–1 (1 – h) = θ
(cos h  sin h )   1

⇒ 1 – h = cos θ)
 1  cos h 

sin 1 (1  h ) cos 1 (cos )
1 = lim . lim
= –2 . = 2. h 0 (1  h ) 0 2(1  cos )
(1 − 0)(0 − 1)
sin 1 (1  h ) 
xx − ax = lim . lim (∵ θ > 0)
Example 32: Let lim = l, a > 0 and h 0 (1  h ) 0 2 sin( / 2)
x→a x − a
= sin–1 1 . 1 = π/2
a x − xa
lim = m, a > 0 . If l = m then find the value of 'a'. and lim f(x) = lim f(0 – h)
x→a x−a x→−0 h→0

FREE BOOKS FOR JEE & NEET =>(@iitjeeadv)


1.40 Differential Calculus for JEE Main and Advanced

We show that the limit P does not exist :


sin 1 (1  {0  h}).cos 1 (1  {0  h})
= lim | sin h| | sin(0  t |
h 0 2{0  h}.(1  {0  h}) L.H.L. = lim  lim
h 0 h t 0

(0  t ) 

1 1
sin (1  h  1).cos (1  h  1) sin t
= lim = lim  1
h 0 2( h  1).(1  h  1) t
t 0


−1 sin h | sin(0  t ) |
sin 1 h lim cos h and R.H.L. = lim  lim
= lim . h →0
2(1 − h ) h 0 h t 

0 (0  t ) 

h 0 h
sin t
π/2 π = lim 1
=1. = . t 0


t
2 2 2
∵ L.H.L. ≠ R.H.L. Hence P does not exist.
1  sin 2 x ∴ The limit L also does not exist.
Example 34: Evaluate lim
x  / 4 (  4x )
Special Limits
1  sin 2 x Let us evaluate some limits with the help of the following
Solution: Let L = lim
x  / 4 (  4x ) special limits :
x  sin x 1 ex  1  x 1
1 1  sin 2 x (i) lim  (ii) lim 
= lim x0 x 3
6 x 0 x2 2
4 x   / 4  
 4  x e x  e x  2x 1
  (iii) lim 
ps
x 0 3


x 3

h x  tan x
el
Put x = 1
4 (iv) lim 
x3 3
eh

x0

  ex  1  x3
3

1  sin 2   h 
je

Example 35: Evaluate lim


1 4  x 0 sin 6 2x
∴ L = lim
iit

4 h 0    e  1  x3
x3
  h
4  4
@

Solution: Let l = lim


 x 0 sin 6 2x
(1  (cos 2h ) ) ex  1  x3
3

1
=  lim = lim
4 h 0 h x 0 ( 2 x )6
3
Now put x = t
1 (1  (cos 2h ) ) (1  (cos 2h ) )
=  lim . 1 et  1  t
4 h 0 h (1  (cos 2h ) ) l= lim again put t = 2y
64 t 0 t 2

(form
0
) 1 e2 y  1  2 y
0 = lim

64 y 0 4y2
1 1  cos 2h
=  lim 1 (e y  1) 2  2e y  2  2 y
4 h  0
h (1  (cos 2h ) ) = lim
64 y 0 4y2
1 2 | sinh |   y 2 
=  . lim 1 1 e 1  1 ey  y  1
= lim    
4 h  0
h (1  (cos 2h ) ) 64 y 0  4  y  2 y 2 
 
1 1 | sin h| 1 1 1 
=  . 2 lim lim ∴  .64 
4 h 0 h 0 h l= 
64  4 2
(1+ (cos2h) ) 
1 1 1 l 1 1
=  . 2. P P ⇒ = l= .
4 2 4 2 4.64 128
| sin h| e x  e x  2x
where P = lim . Example 36: Evaluate lim
x 0 x  sin x

h →0 h

FREE BOOKS FOR JEE & NEET =>(@iitjeeadv)


Limits 1.41

Solution: lim
e x  e x  2x
= lim
 e  1  t
t

= lim
et  t  1

1
x 0 x  sin x t 0
 et  1  2
2 t 0 t2 2
  .t
e x  e x  2x l1
 t 
= lim = l (say), where
x 0 x  sin x 3 2
(using special limit(ii))
.x


x3
e x − e − x − 2x x-sinx Caution
l1 = lim and l2 = lim
x →0 x3 x →0 x3 1 ln (1  y) 1  1 1
Doing lim   ·     0 using
x − sin x 3t − sin 3t y0  y y y  y y
Now l2 = lim = lim Put x =3t
x→0 x3 t→0 27 t 3 ln (1  y)
lim  1 , is not correct.
3t − (3 sin t − 4 sin t ) 3 y0 y
= lim
t→0 27 t 3 (1  x )1/ x  e
Example 38: Evaluate lim
3( t  sin t ) 4 sin 3 t 1 4 x 0 x
= lim  lim = l2 +
t→0 27 t 3 t 0 27 t 3 9 27 (1  x )1/ x  e
Solution: lim
8l2 4 1 x 0 x
⇒ = ⇒ l2 = .
9 27 6  ln(1 x ) 1 
x e x  1 e
e ex
 2x  
ps
Now l1 = lim Put x = 3y = lim   , assume M = ln(1 x )
1
x 0 x3 x 0 x x
el

e3 y − e −3 y − 6 y eM  1 e M eM
eh

= lim 3 = lim . = lim


y→0 27 y x→0 x
x 0 M x
je

e   
3
y
 e y  3 e y  e y  6 y ln(1  x )  x e
=−
iit

= lim = e. lim
y→0 27 y 3
x 0 x 2 2
@

3 {using previous example}


 e2 y  1  1  e y  e y  2 y 

8
= lim    lim   1 1
y→0 27  2y  y 0 9
 y3  Example 39: Evaluate lim −1 2
− 2
x→0 (sin x) x
8 1
= + l1 1 x2 1
27 9 Solution: Solving lim 2
. −1 2
−.
x→0 x (sin x) x2

8l1
=
8 ⇒ l1 = 1 1 1
9 27 3 = lim 2
− = 0, is wrong.
1 6 x→0 x x2
∴ The required limit = . = 2 .
3 1 The correct way is to put x = sin θ
1 1
 1 ⇒ lim − 2
Example 37: Evaluate lim x  x 2 n 1   x→0 (sin −1 x ) 2 x
x  x

 1 1 sin 2   2
1  
x
 = lim = lim
Solution: lim x 1  n 1    Put x = 1/y 0  sin 
2 2 0 2 sin 2 
x    x  
(sin   ) (sin   )
 1 n (1  y)  = lim
= lim  
y0  y
 0 4
 y2 
sin     1 1
Put ln(1+y) = t ⇒ 1+y = et = 2. lim = 2     .
0 3  6 3
⇒ y = et –1 ⇒ as y→0, t→0 (using special limit(i))

 1 t sin x − x 2 − {x}.{− x}
= lim    Example 40: Evaluate lim
t 0  e t
 1 (e  1) 
t 2
x→0 x cos x − x 2 − {x}.{− x}

FREE BOOKS FOR JEE & NEET =>(@iitjeeadv)


1.42 Differential Calculus for JEE Main and Advanced

where {.} denotes the fractional part function.


2 t  n t  1  t2
sin h − h − {h}{−h}
2 = 8 t3
Solution: f(0+) = lim 2 t . (1  t 2 )3
h→0 h cos h − h 2 − {h}{−h} 2 t n t  1  t 2
= . 4 t2
sin h − h − h (1 − h )
2 (1  t )3 (1  t )3
= lim
h→0 h cos h − h 2 − h (1 − h ) 1 2 t n t  1  t 2
= . 4 t2
sin h − h 2 (1  t )3 (1  t )3
= lim as t → 1 , y → 0
h→0 h cos h − h Put t = 1 + y;
1 2 (1  y)  n (1  y)  1  (1  y) 2
h − sin h h3 1 1 Limit = lim
= lim · = ·2 = . 2 y→0  y3
h→0 h3 h (1 − cos h ) 6 3
Put 1 + y = ez ; as y → 0, z → 0
− sin h − h 2 − (1 − h )h 1 2 z ez  1  e2 z
Similarly f (0–) = lim =− lim
h→0 −h cos h − h 2 − (1 − h )h 2 z → 0  ez  1  . z3
 z 
(sin h  h )
= lim 1 e 2z
− 2 z ez − 1
h→0  h (1  cos h ) = lim
2 z → 0 z3
sin h
1 2 1 e (e − e − z − 2 z)
z z
1
h lim
ps
= lim = = 1. = = .l
h0 1  cos h 2 2 z→0 z3 2
el
Since f (0–) ≠ f (0+), the limit does not exist. e3 u − e − 3 u − 6 u

 
l = lim when z = 3 u
eh

x  n 1  x2  x u→0 27 u 3
Example 41: Evaluate lim (eu  e  u )3  3 (e u  e  u )  6 u
je

x→0 x3 = lim
27 u 3
iit

u→0
Solution: Let 1 + x2 − x = t
3
@

⇒ x → 0 , t → 1  e2 u  1  1
= lim   + l
1 − t2 u→0  2 u  9
Also 1 + x2 = t2 + x2 + 2 t x ⇒ x = .
2t
8 8 1
1  t2 ⇒ l = ⇒ l =
 n t 9 27 3
2t
The given expression = 8 t3 1
(1  t 2 )3 ∴ The required limit = .
6

Concept Problems H

1. Evaluate the following limits :
xn −1
sin x o
tan x 3 (ii) lim (m, n ∈ N)
(i) lim (ii) lim x→ 1 xm −1
x→0 x x→0 x
log 2 (1+ x )
2. Evaluate the following limits : (iii) lim
sin 2 x x→0 x
(i) lim (ii) lim sin 8x cot 3x
x→0 5 x x→0 ln (1 + 8x )
(iv) lim
sin 3x 1  cos 5x x→0 ln (1 + 7 x )
(iii) lim (iv) lim
x sin 2 x 2 4. Evaluate the following limits :
x 0 3x
3. Evaluate the following limits : ln (1  x ) ln sec x
(i) lim (ii) lim
e4 x  1 x  0 3x  1 x→0 x2
(i) lim
x  0 tan x 5. Evaluate the following limits :

FREE BOOKS FOR JEE & NEET =>(@iitjeeadv)


Limits 1.43

(1  x )1/ m  (1  x )1/ n sin 3x  sin x ln(1  3x sin x )


(i) lim (iii) lim (iv) lim
x→0 x x 0 ln( x  1) x 0 tan x 2
a x − bx 10. Evaluate the following limits :
(ii) lim 8x  7 x
x→0 x 1− x2 (i) lim x x
x 0 6  5
6. Evaluate the following limits :
(i) lim
sin 2 x sin 2 3x
(ii) lim
x→0 tan 3x x 0 ln 2 (1  2 x )
cos x  sec x
(ii) lim (5  1) (4  1)
x0 x2 (iii) lim
 0 (3  1) (6  1)
cos 7 x  cos 9 x
(iii) lim
x0 cos x  cos 5x 1  2x  1
(iv) lim
n (1  sin 4 x ) x→0 tan 3x
(iv) lim
x→0 e 1
sin 5 x
11. Evaluate the following limits :
7. Find whether the following limits exists: (1  cos ) 2
(i) lim
1 1  cos x  0 tan 3   sin 3 
(i) lim
x→0 x 1  cos x
x (1  1  x 2 )
1
cot 1 (ii) lim
ps
(ii) lim x x 0
1  x 2 (sin 1 x )3
el
x0 x 1  cos x
for x  0
 x 2
| x −1 |
eh

(iii) lim −1 12. Find lim f(x) if f(x) = 


x→1 tan ( x − 1)
 x
je

x→0
for x  0
(iv) lim x4 sin (2 x )  2 x  x 2
iit

x0
8. Evaluate the following limits : 13. Evaluate the following limits :
@

e tan x  e x sin x  cos x


(i) lim (i) lim
x  0 tan x  x
x  / 4   4x
n cos x sin(e x 1  1)
(ii) lim (ii) lim
x→0 4 1  x 1 ln x
x2 1
1  sin 3x  1 3
7  x3  3  x 2
(iii) lim (iii) lim
x 0 ln (1  tan 2 x ) x 1 x 1
cos x
1 x  x2 1 (iv) lim
(iv) lim x  / 2 3
x→0 sin 4 x (1  sin x ) 2

9. Evaluate the following limits : 14. Evaluate the following limits :


x ln x  1
1  x sin x 1 1  cos 5x (i) lim (1  x ) tan (ii) lim
(i) lim (ii) xlim x1 2 x  e x e
x 0 x 2 0 1  cos 3x
x3  x 2  2
(iii) lim (1  x ) log x 2 (iv) lim
x 1 x1 sin( x  1)

Practice Problems G

sin(1  x )  sin(1  x ) 1  tan x  1  sin x 1
15. Show that lim = 2cos 1. 16. Prove that lim 3
=
x 0 x x 0 x 4

FREE BOOKS FOR JEE & NEET =>(@iitjeeadv)


1.44 Differential Calculus for JEE Main and Advanced

cos( x e x )  cos( xe  x ) 26. Evaluate the following limits :


17. Evaluate lim
x 0 x 3 sin 2   sin 2 
(i) lim
   2  2
tan([2 ]x 2 )  tan([ 2 ]) x 2
18. Evaluate lim where [.]
x 0 sin 2 x  ya y 
(ii) lim  sin
2a 
. tan
denotes the greatest integer function. y a  2

sin( n ) 27. Evaluate the one-sided limits and find whether the limit
19. Evaluate lim (m and n positive integers) exists:
 0 (sin ) m
x sin( x − [ x ])
sin(a  2h )  2 sin(a  h )  sin a (i) lim
20. Evaluate lim x→1 x −1
h 0 h2
1 − cos 2( x − 1)
21. Evaluate the following limits : (ii) lim
x→1 x −1
n (1+ a sin x )
(i) lim {x}sin( x  2)
x→0 sin x 28. Evaluate lim , (where {.} denotes the
( x  2) 2
x  2
1 − cos x fractional part function.)
(ii) lim
x0 sin 4 3 x 29. Evaluate the following limits :
sin 3x 2 cos 1 (1  x )
(iii) lim (i) lim
x→0 ln cos( 2 x − x ) 2
x 0 x
ps
x
2 sin x  sin 2 x 1  sin
el
(iv) lim 2
x0 x3 (ii) lim
x x
eh

x  x
22. Evaluate the following limits : cos  cos  sin 
2 4 4
je

3 sin x  x 2  x 3
(i) xx  aa
iit

lim
x→0 tan x  2 sin 2 x  5x 4 (iii) lim (a  0)
x a x  a
@

1  cos x  2 sin 1 x  sin 3 x 30. Evaluate the following limits :


(ii) lim
x 0 3 tan 1 x  x 2  x 5 ln(1  x  3x 2  2 x 3 )
1 1 (i) lim
sin 2 x  (sin x )  (tan
2
x) 2
x 1 ln(1  3x  4 x 2  x 3 )
(iii) lim
x→0 3x
x x 1
23. Evaluate the following limits : (ii) lim
x 1 x ln x
sin 3
x ln (1  3x ) 31. Evaluate the following limits :
(i)
 
lim
x 0 1 2
(tan x) e 53 x
1
  cos 1 x
(i) lim
e x  33x
2
x  1 x 1
(ii) lim
x 0  x2  ln x
sin    sin x (ii) lim
x 1 x4 1
 2 
32. Evaluate the following limits :
logsec x / 2 cos x
(iii) lim 4 2  (cos x  sin x )5
x→0 logsec x cos x / 2 (i) lim
x/4 1  sin 2 x
1 − cos 7 (π − x ) sin( x   / 6)
24. Evaluate lim , n = 1, 2. (ii) lim
x →π 5 ( x − π)n x/6 3  2 cos x
25. Evaluate the following limits : 33. Evaluate the following limits :
1  3 tan x x 2x − x log10 (1  2 x  3x 2  4 x 3 )
(i) lim (ii) lim (i) lim
x 3 / 4 1  2 cos 2 x x → 0 1 − cos x x 0 log10 (1  x  2 x 2  7 x 3 )

FREE BOOKS FOR JEE & NEET =>(@iitjeeadv)


Limits 1.45

tan  (ii) Existent × Non-existent → no comment


(ii) lim For example, consider lim (x – 1).[x]
 0 3
(1  cos ) 2


x→1

L.H.L. = 0.(–1) = 0, RHL = 0.0 = 0.


cos x  3 cos x


(iii) lim 2 The limit exists even if one of the individual limits
x 0


sin x does not exist. However, this may not always hold.
1  cos x cos 2 x For example, the limit
(iv) lim
x 0 x2 lim (cosx).21/x does not exist.


x→0

1.10 Algebra of Limits In fact, all the limit questions in 0 × ∞ form cannot


be determined in advance.
Earlier, we had seen the theorem on limits. But we face some (iii) Non-existent × Non-existent → no comment
difficulty in applying these theorems in several situations. We
should take some precautions while applying the theorems. For example, consider lim [ x ].[ x ]
x 0


1. lim(f ( x )  g ( x )) = lim f ( x )  lim g ( x ) L.H.L. = (–1).0 = 0, RHL = 0.(–1) = 0.


x a x a x a The limit exists even if both the individual limits do not


(i) Existent + Existent → Existent exist. However, this may not always hold.
 sin x  tan x  For example, the limit lim [ x ].[ x ] does not exist.
lim   x 1

x 0  x 
L.H.L. = 0.(–1) = 0, RHL = 1.(–2) = –2.

 sin x   ta n x 

= lim    lim  x   lim f (x) 
ps
x 0  x  x 0  

lim  f ( x ) / g ( x )   x a

 lim g(x) 
3.
el
= 1 + 1 = 2. x a

x a
(ii) Existent + Non-existent → Non-existent
eh

 sin 3 x  tan x  Existent


 Existent
je

lim   Existent (non  zero)


x 0  x3
 
iit

In all other cases, we need to examine the limit carefully.



3
@

 sin x   ta n x 
= xlim    xlim   4. If lim f(x) = , then
0  x  0  x 3  x →a

= 1 + ∞ = ∞.
 
1 1
lim  f ( x ) 
1/ n
 lim f ( x )

n
= λn .
 sin x 1 x a x a
Also, lim   sin  is non-existent.
x 0  x x

If n is an odd integer, then there is no concern. If n is even and 
(iii) Non-existent + Non-existent → No comment is positive, again there is no concern. If n is even and  is zero,
For example, consider lim ([x] + [–x]) we need to examine the domain of the function.

x→0
lim (sin x)1/3 = 01/3 = 0
L.H.L. = –1 + 0 = –1, RHL = 0 + (–1) = –1. x→0

The limit exists even if both the individual limits lim (cos x)1/2 = 11/2 = 1

do not exist. However, this may not always hold. x→0

For example, consider lim (2[x] + [–x]) lim (sinx)1/2 = 01/2 = 0. Note that the function is defined only

x→0 x→0
L.H.L. = 2(–1) + 0 = –2, RHL = 2. 0 + (–1) = –1. in the right neighbourhood of x = 0. Thus, the limit evaluated

We see that the limit does not exist. is actually the right hand limit lim (sin x )1/ 2 = 0.
  

x 0
2. lim(f ( x ) . g ( x )) = lim f ( x ) . lim g ( x )
x →a x a x a
However, lim( sin x ) 2 1/ 2
is not equal to 01/2 i.e. 0, since the
(i) Existent × Existent → Existent x 0

 sin x. tan 1 x  function (− sin 2 x )1/ 2 is not defined either in the left or the
lim  
x 0  x2 right neighbourhood of x = 0.
 

 

m
 ta n 1 x 
m
 sin x  lim  f ( x )  n  lim f ( x ) n
, where m and n are integers.
= lim    lim  
x 0  x  x 0 
x a x a
 x 

= 1 × 1 = 1. Here also we need to examine the domain of the function.

FREE BOOKS FOR JEE & NEET =>(@iitjeeadv)
1.46 Differential Calculus for JEE Main and Advanced

Note: wrong. We should not calculate the limit of some term which
is not as a separate factor to the problem.
A. If lim[f ( x )  g ( x )] exists then we can have the following
x c 1 − cos(1 − cos x ) 1
cases: (ii) We have lim = by proceeding with
x→0 x4 8
(i) If lim f ( x ) exists, then lim g ( x ) must exist.
x →c x →c  x
1  cos  2 sin 2 
Since g = (f + g) – f, we have by the theorem on  2.

limits 4
lim g ( x )  lim(f ( x )  g ( x ))  lim f ( x ) and this exists.
x
x c x c x 0  1  cos x 2 

1  cos  .x 
(ii) Both lim f ( x ) and lim g ( x ) do not exist.  x2 
x →c x →c However, solving as lim 4
x→0 x
Consider lim[ x ] and lim{x} .
1 − cos( x 2 / 2)

x →1 x →1
1  cos x 1
where [.] and {.} represent greatest integer and = lim [since lim  ]
x4 2

fractional part function, respectively. Here both the
x→0 x0 x 2
limits do not exist but lim([ x ]  {x})  lim x  1 x2 x2
exists.
x 1 x 1 2 sin .sin
= 4 4 = 1 is wrong, although the answer
B. If lim f ( x )g ( x ) exists, then we can have the following 2
x x 2 8
x →c
16 . .
cases: 4 4
(i) Both lim f ( x ) and lim g ( x ) exist. Obviously, then may be correct.
x →c x →c
ps
e x  cos x  1  1
2

e x  cos x
2

lim f ( x )g ( x ) exists.
x →c (iii) lim = lim
el
x0 x2 x 0 x2
(ii) lim f ( x ) exists and lim g ( x ) does not exist.
eh

x →c x →c
(e x  1) 1  cos x
2

1 3
1 = lim  lim  1 
x 0 x2 x 0 x2
je

Consider f(x) = x; g(x) = . 2 2



sin x
e x − cos x
2
iit

lim f ( x ) = 0 exists but lim g ( x ) does not exist. However, solving as lim
x →0 x →0

x2
@

x→0
Here lim f ( x ) . g ( x ) exists and equals 1 but this may
x →0

not be always true. = lim = 1 (as cos x → 1 as x → 0) is wrong.
x→0
(iii) Both lim f ( x ) and lim g ( x ) do not exist.
x →c x →c esin 2 x  esin x
1 if x  0 (iv) lim
x0 x
Let f(x) =  and
2 if x  0
esin 2 x sin x  1 sin 2 x  sin x
2 if x  0 lim
x 0 sin 2 x  sin x
· = 1·[2 – 1] = 1
g(x) =  . Here lim f ( x ) and lim g ( x ) x
1 if x  0 x →0 x →0

However, solving as
do not exist while lim f ( x ).g ( x ) exists and sin 2 x sin x
x →0
esin 2 x − esin x 2x
.2x
−e x
.x
equals 2. But this may not be always true. lim = lim e
x→0 x x→0
x
Caution
e2 x − ex e x (e x −1)
= lim = lim =1
1  cos x cos 2 x
3 x→0 x x→0 x
(i) To prove lim 
x 0 x2 2 is wrong although the end result may be correct.
we rationalize the numerator first and proceed. 5. Composition Law
1 − cos x cos 2x It is tempting to write
However, solving as lim 2
x→0 x lim x2  9  lim ( x 2  9) ...(1)
1 − cos x x  4 x  4
= lim using lim cos 2x = 1 is
x→0 x2 x→0  (4) 2  9  25  5 .

FREE BOOKS FOR JEE & NEET =>(@iitjeeadv)


Limits 1.47

But can we simply "move the limit inside the radical" in (1) ?  1 x 
To analyze this question, let us write Now consider lim n  
x 1  1 x
g(x) = x and f(x) = x2 + 9.  
1 x 1 1
Then the function that appears in (1) is the composite function lim  and y = λnx is continuous at x = .
x 1 1  x 2 2
g (f ( x ))  f ( x )  x 2  9 .  1 x  1
∴ lim n   n .
Hence our question is whether or not x 1  1  x  2
 
lim g (f ( x ))  g ( lim f ( x )) .
x a x a But one should note that even if limit of f(x) does not exist,
The next limit law answers this question in the affirmative, the limit of g(f(x)) may exist.
provided that the "outside" function" g meets a certain For example, let f(x) = sgn(x), whose limit does not exist at
condition; if so, then the limit of the composite function g(f(x)) x = 0, and g(x) = x2.
as x → a may be found by substituting into the function g the We see that lim g (f ( x )) = lim (sgn( x )) 2 = 1.
x →0
limit of f(x) as x → a. x →0
A. If lim f ( x )  b and g(x) is continuous at x = b, then Also, the limit of g(f(x)) may exist even if g(x) is discontinuous.
x a

 
For example, let f(x) = x2 + 1, whose limit at x = 0 is 1, and
lim g (f ( x ))  g lim f ( x ) = g(b). g(x) = [x]. We know that g(x) is discontinuous at x = 1.
x a x a
Proof: Since g is continuous at b, for every neighbourhood However, lim g (f ( x )) = lim[ x 2  1] = 1.
x →0 x 0
N of g(b) there exists a neighbourhood C of b such that g(y)
is in N for every y in C.  x2 1 
ps
Example 1: Evaluate lim   , [.] is G.I.F.
x 1  4( x  1) 
In turn, since l im f(x)=b there exists a deleted neighbourhood  
el
x →a
D of a such that f(x) is in C for every x in D. Combining x2 1 1

eh

these remarks, we see that for every neighbourhood N of Solution: lim


x 1 4( x  1) 2
g(b) there exists a deleted neighbourhood D of a such that
je

f(x) is in C, and therefore g(f(x)) is in N, for every x in D. 1


y = [x] is continuous at x = .
iit

This proves the result. 2


1
1
@

A function such as y = 2 , is a composition of the Hence the limit is   = 0.


x +3 2
polynomial function f(x) = x2 + 3 and the reciprocal function
g(x) =
1 Study Tip
x
1 1 We know that the function |x| is continuous everywhere; thus,
i.e., g(f(x)) = = 2 . it follows that if lim g ( x ) exists then
f (x) x +3 x →a
Thus, y = g(f(x)). Therefore, since lim f(x) = f(a) and g is
x →a lim g(x)  lim g(x)
x a x →a
1 That is, a limit symbol can be moved through an absolute
continuous at f(a), lim y = .
x →a a2 + 3 value sign, provided the limit of the expression inside the
 x2 1 absolute value signs exists. For example,
 , x 1
Let f(x) =  x  1 , g(x) = sin–1 x .
3 ,x 1 4 lim 5 − x 2 = lim(5 − x 2 ) = | − 4 | = 4 .
 x →3 x →3

To find lim g (f(x)) , we ask two questions. An useful application of composition law is the following:
x →1
Let L  lim  f ( x ) 
g(x)

Firstly, whether lim f ( x ) exists . The answer is yes : lim f ( x ) = 2. x a


x →1 x →1
⇒ ln L  lim g ( x ).lnf ( x )
Secondly, whether g(x) is continuous at x = 2. x a
x
We know that sin–1 is continuous. ⇒ L = elim g ( x ).nf ( x )
x →a

4
Thus, using the above theorem, B. If lim f ( x )  b , with f(x) ≠ b for every x in some
π x a
lim g (f ( x )) = g(2) = . neighbourhood of 'a' and if lim g ( x )  c , then
x →1 6
x b

FREE BOOKS FOR JEE & NEET =>(@iitjeeadv)


1.48 Differential Calculus for JEE Main and Advanced

lim g  f ( x )   c . (This theorem is sometimes useful if = lim g  1  2h 


x a h 0
g is discontinuous at b.) = lim 2(–1 + 2h) + 3 = 1.
h→0
x 1  2
sin x  , x 1 Thus, lim g  f ( x )  = 1.
Let f(x) = , g(x) =  x  1 x 2
x 
3 ,x 1 6. Domination Law
To find lim g (f ( x )) , we ask two questions. Firstly, whether
x →0 If two function f(x) and g(x) satisfy the inequality


lim f ( x ) exists. The answer is yes: lim f ( x ) = 2. Secondly, f(x) < g(x) ...(1)


x →0 x →0
x2 −1 for all the values of x belonging to a neighbourhood of a point
whether lim g( x ) exists. We have lim g ( x ) = lim = = 2. a except possibly at a, then
x →1 x →1 x →1 x −1
Thus, using the above theorem, lim g (f ( x )) = lim g ( x ) = 2. lim f ( x )  lim g ( x ) ...(2)
x →0 x →1 x a x a


Note that the above limit has been evaluated even if g(x) is provided that the limits of both functions, as x → a, exist.
discontinuous. Further, lim g (f ( x ))  g lim f ( x )
x 0
 x 0
 Note that a strict inequality connecting functions may lead to
a non-strict inequality for their limits. For example, if 0 < |x|
 x2 1 < 1 then x2 < |x|. But nevertheless,
 , x 1
Now let f(x) = 1, g(x) =  x  1 . lim x 2  lim | x |  0 .
3 ,x 1 x 0 x 0

Find lim g (f ( x )) . sinx tanx
ps
x →0 Now consider f(x) = and g(x) = .
x x
The theorem is not applicable since the function f(x) attains
el

the value 1 in the neighbourhood of '0'. We have f(x) < g(x) in the neighbourhood of x = 0,
eh

We see that g(f(x)) = 3. Hence lim g (f ( x )) = 3. sin x tanx


x →0
x ≠ 0. Since lim = lim = 1,
x →0
x x →0 x
je

1  x ,  1  x  2
Example 2: If f(x) =  we have lim f ( x )  lim g ( x ) .
iit

 2 x  5, 2  x  3 x 0 x 0

 x  1 ,  2  x  1 Thus, we see the necessity of the equality sign in the


@

 2
g(x) =  domination law.
2x  3 ,  1  x  1

The theorem remains of course true if the strict inequality
find (i) lim g  f ( x )  and (ii) lim g  f ( x )  .
x 1 x 2 f(x) < g(x) entering into its formulation is replaced by the
Solution: (i) lim f ( x ) = lim 1  x = 0. non-strict inequality f(x) ≤ g(x).
x →1 x 1 In the case x → ∞ we must require that inequality (1) should
Also, g(x) is continuous at x = 0. hold for x > N where N is a fixed number.
 

Hence lim g  f ( x )   g lim f ( x ) = g(0) = 3. An important particular case of the theorem arises when one of
x 1 x 1
the two functions g(x) and f(x) is a constant. For definiteness,
(ii) lim f ( x ) does not exist. The theorem is not let g(x) = constant. Then the theorem reads :
x →2

applicable. We evaluate the one sided limits : If f(x) < M (or f(x) ≤ M) for 0 < |x – c| < δ where δ is a
L.H.L. = lim g  f ( x )  fixed number then lim f ( x )  M provided that the limit of
x  2 x c
= lim g  f (2  h )  the function f(x) exists.

h 0

= lim g 1  (2  h ) 
h 0
1.11 Limits when x → ∞

= lim g (1  h )
h 0

Sometimes we will not be concerned with the behaviour of g(x)
= lim 2(1  h )  3  1 . near a specific value of x, but rather with how the values of f(x)
h 0

behave as x increases without bound or decreases without bound.
R.H.L. = lim g  f ( x )  This is sometimes called the end behaviour of the function
x  2
= lim g  f (2  h )  because it describes how the function behaves for values of x

h 0 that are far from the origin. To investigate the end behaviour of
= lim g  2(2  h )  5  = lim g  4  2h  5  a function we find limits of the function as x → –∞ or as x → ∞.

h 0 h 0

FREE BOOKS FOR JEE & NEET =>(@iitjeeadv)


Limits 1.49

To discuss the end behaviour of a function we assume that Solution: (a) Divide numerator and denominator of
the function is defined on an infinite interval. x 1
by x2 (the highest power of x in the denom-
Let the independent variable x of a function y = f(x) increase ( x  3) ( x  2)
indefinitely. This means that x is made to take on the values 1 / x  1 / x2
becoming greater than any given positive number. In such a inator), obtaining
(1  3 / x ) (1  2 / x )
case we say that x becomes infinitely large or that x approaches
infinity and write x → ∞. If x decreases indefinitely, that is, Hence, as x → ∞,
becomes less than any given negative number, we say that x
0−0 0
becomes negatively infinite or that it approaches minus infinity y→ = =0
and write x → –∞. (1 + 0) (1 − 0) 1
Definition A number λ is said to be the limit of the function As x → − ∞, the factors x – 1, x + 3, and x – 2 are negative,
y = f(x) as x → ∞ if for all sufficiently large values of x the and, therefore, y → 0–. As x → ∞, those factors are positive,
corresponding values of the function f(x) become arbitrarily and, therefore, y → 0+.
close to the number λ. ( x  2) (1  x )
(b)
x 3
Limits as x Tends to – ∞
We can easily understand the meaning of the statement: f(x) − x 2 − x + 2 − x − 1 + 2 /x
= = , after dividing
approaches λ as x → – ∞ . x −3 1 − 3/x
In fact, if x = – t and f(x) = f(– t) = g(t) then t tends to ∞ numerator and denominator by x (the highest power of x in
as x tends to – ∞, and the question of the behaviour of f(x) the denominator).
ps
as x tends to – ∞ is the same as that of the behaviour of g(t) As x → ∞, 2/x and 3/x approach 0, and –x – 1 approaches
as t tends to ∞. ∞. Thus, the denominator approaches 1 and the numerator
el

2x  1 approaches ∞.
eh

Example 1: Evaluate lim .


x x 1 As x → –∞, x + 2 and x – 3 are negative and 1 – x is positive ;
je

Solution: Note that both the numerator and the denom- so, y → ∞. As x + 2 and x – 3 are positive and 1 – x is negative;
so, y → –∞. It should be emphasized that the symbols ∞ and
iit

inator approach infinity as x approaches infinity.


–∞ are not real numbers. The phrase “f(x) approaches ∞” is
@

lim (2 x − 1) → ∞ equivalent to saying that “f(x) approaches the unapproachable”


x →∞
2x − 1 or “f(x) increases without bound”.
lim
x →∞ x + 1 “The symbols ∞ and –∞ are used here to encapsulate a
lim ( x + 1) → ∞ particular way in which limits fail to exist. It is important, to
x →∞
keep in mind that “∞” is not a number. The limit does not exist.
To resolve this problem, we divide both the numerator and the To say, for example, that f(x) → ∞ as x → a+ is to indicate
denominator by x. After dividing, the limit may be evaluated that lim f(x) does not exist and that it fails to exist because
as follows: x a 
the values of f(x) increase without bound as x approaches a
2x  1
2x  1
from the right. Furthermore, since ∞ and –∞ are not numbers,
lim  lim x
it is inappropriate to manipulate these symbols using rules
x  x  1 x  x  1
of algebra. For example, it is not correct to equate (∞ – ∞)
x
with 0.
1
lim 2  lim
= lim
2 x  (1 / x )
=
x  x 
x How Limits at Infinity Can Fail to Exist?
x  x  (1 / x ) 1
lim 1  lim Limits at infinity can fail to exist for various reasons. One
x  x  x possibility is that the values of f(x) may increase or decrease
20 without bound as x → ∞ or as x → –∞. For example,
= 2.
1 0 the values of f(x) = x3 increase without bound as x → ∞
and decrease without bound as x → –∞. We denote this by
Example 2: Determine what happens to y as x → –∞ writing
and x → ∞.
lim x3 = ∞, lim x3 = – ∞,
x 1 ( x  2) (1  x ) x x
(a) y = (b)
( x  3) ( x  2) x 3 lim (–x3) = –∞, lim (–x3) = ∞
x x

FREE BOOKS FOR JEE & NEET =>(@iitjeeadv)


1.50 Differential Calculus for JEE Main and Advanced

If the values of f(x) increases without bound as x → ∞ or as because both x and x – 1 become arbitrarily large and so their
x → – ∞, then we write product does too.
lim f(x) = ∞ or lim f(x) = ∞ Example 3: Discuss the behaviour of 2x3 – 11x2 + 12x
x x
when x is large.
and if the values of f(x) decreases without bound as x → ∞
or as x → – ∞, then we write Solution: First consider x positive and large. The three
terms, 2x3, –11x2 , and 12x, all become large in absolute
lim f(x) = –∞ or lim f(x) = – ∞
x x value. To see how 2x3 – 11x2 + 12x behaves for large positive
Limits at infinity can also fail to exist because the graph of the x, factor out x3.
function oscillates indefinitely in such a way that the values  11 12 
of the function do not approach a fixed number and do not 2x3 – 11x2 + 12x = x 3  2   2  ...(1)
 x x 
increase or decrease without bound; the trigonometric functions
sin x and cos x have this property. In such cases we say that Now, since 11/x and 12/x2 → 0 as x→ ∞
the limit fails to exist because of oscillation.  11 12 
lim  2   2   2
Note: If f(x) → l as x → ∞, and l is not zero, then f(x)cosxπ x   x x 
and f(x)sinxπ oscillate finitely as x → ∞. If f(x) → ∞ or
f(x) → –∞, then they oscillate infinitely. The graph of either Moreover, as x → ∞ , x3 → ∞,
function is a wavy curve oscillating between the curves y =  11 12 
f(x) and y = –f(x). Thus lim x 3  2   2   
x   x x 
End Behaviour of a Polynomial Hence lim (2x3 – 11x2 + 12x) = ∞
ps
x
The end behaviour of a polynomial matches the end behaviour
Now consider x negative and of large absolute value. The
el
of its highest degree term.
More precisely, if cn ≠ 0 then argument is similar. Using (1), and the fact that
eh

 11 12 
lim (c0 + c1x + ... + cnxn) = lim cnxn lim x3 = – ∞ and lim  2   2   2
je

x x x x   x x 


iit

lim (c0 + c1 x + .... + cnxn) = lim cnxn it follows that lim (2x3–11x2 + 12x) = – ∞
x x
x
@

We can get these results by factoring out the highest power This completes the discussion. It is interesting to graph f(x) =
of x from the polynomial and examining the limit of the
2x3–11x2 + 12x and see what is happening for |x| large. The
factored expression.
figure is shown below.
 c0 c  Since lim (2x3–11x 2 + 12x) = ∞ the graph rises arbitrarily
Thus, c0 + c1x + .... + cnxn = xn   n11  ...  c n 
x  x
n
x
high as x → ∞.
As x → –∞ or x → ∞, it follows that all of the terms with
positive powers of x in the denominator approach 0, so the Since lim (2x3–11x2 + 12x) = − ∞ , the graph goes arbitrarily
x
above limits are certainly plausible. far down as x → – ∞ ,
For example lim (7x5 – 4x3 + 2x – 9)
x

= lim 7x5 = – ∞
x

and lim (–4x8 + 17x3 – 5x + 1)


x

= lim – 4x8 = –∞
x

Let us find lim (x2 – x).


x
Note that we cannot write
lim (x2 – x) = lim x2 – lim x = ∞ – ∞.
x x x
The above example generalizes to any polynomial function
The limit laws can't be applied to infinite limits because f(x) = anxn + an–1xn–1 + ..... + a0.
(∞ – ∞) is indeterminate. However, we can write Let f(x) be a polynomial of degree atleast 1 and with the
lim (x2 – x) = lim x (x – 1) = ∞ leading coefficient an positive. Then
x x

FREE BOOKS FOR JEE & NEET =>(@iitjeeadv)


Limits 1.51

lim f ( x )   We obtain
x 
5x 3  2 x 2  1 5x 2  2 x  1 / x
If the degree of f is even, then lim f ( x )   lim = lim =∞
x   x  3x  5 x  35/ x
But if the degree of f is odd, then lim f ( x )   where the final step is justified by the fact that
x 
1 5
5x2 – 2x → ∞, → 0, and 3 + →3
End Behaviour of a Rational Function x x
x 3 + 6 x 2 + 10 x + 2 as x → –∞.
Let us determine how f(x) = Let f(x) be a polynomial and let axn be its term of highest
2x 3 + x 2 + 5 degree. Let g(x) be another polynomial and let bxm be its term
behaves for arbitrarily large positive number x. of highest degree.
As x gets large, the numerator x3 + 6x2 + 10x + 2 grows large, f (x) ax n
influencing the quotient to become large. On the other hand, Then lim  lim
x  g ( x ) x  bx m
the denominator also grows large, influencing the quotient to
become small. An algebraic step will help reveal what happens f (x) ax n
and lim  lim
to the quotient. We have x  g ( x ) x  bx m

 6 10 2  The proofs of these facts are similar to the argument used in


x 3 1+ + 2 + 3 
x 3 + 6x 2 +10x + 2  x x x  the previous example. In short, when working with the limit
f(x) = = of a quotient of two polynomials as x → ∞ or as x → −∞ ,
2x 3 + x 2 + 5 3 1 5
x 2+ + 3  disregard all terms except the one of highest degree in each of
 x x 
ps
the polynomials. The next example illustrates this technique.
6 10 2 Alternatively, divide each term of the numerator and
1  
el

 x x 2 x 3 for x ≠ 0 denominator by x k where k is the highest power of


eh

1 5 x in numerator and denominator. Then use the result


2  3
x x c
 0 where c is a constant and k > 0.
je

lim
Now we can see what happens to f(x) when x is large. x  x k
iit

As x increases, 6/x → 0, 10/x2 → 0, 2/x3 → 0, 1/x → 0 and (i) First of all simplify the given expression.
5/x3 → 0.
@

(ii) Then divide each term of the numerator and denominator


1+ 0 + 0 + 0 1 by xm, where x is the independent variable and m is the
Thus, f(x) → = .
2+0+0 2 highest power of x.
c
So, as x gets arbitrarily large through positive values, (iii) Then expression of the form k tends to 0, where c is a
x
x 3 + 6 x 2 + 10 x + 2 1 constant and k is a positive number since
the quotient approaches . c
2x 3 + x 2 + 5 2 when x → ∞ or x → – ∞, → 0, k > 0.
The technique used in this example applies to any function xk
that can be written as the quotient of two polynomials i.e. a Example 5: Examine the following limits:
rational function. 3x 4  5x 2
A useful technique for determining the end behaviour of (a) lim
x   x 4  10 x  5
rational function f(x) = p(x)/q(x) is to factor and cancel
x 3  16 x
the highest power of x that occurs in the denominator q(x) (b) lim 4
x  5x  x 3  5x
from both p(x) and q(x). The denominator of the resulting
fraction then has a (nonzero) limit equal to the leading x4  x
(c) lim
coefficient of q(x), so the limit of the resulting fraction can x  6 x 3  x 2
be quickly determined. The following examples illustrate Solution: By the preceding observations,
this technique. 3x 4  5x 2 3x 4
(a) lim  lim  lim (3) = –3
3
5x - 2x +12 x   x  10 x  5
4 x  x 4 x 
Example 4: lim
x  3x + 5 x 3  16 x x3 1
(b) lim = lim  lim 0
Solution: Divide the numerator and denominator x 5x  x  5x
4 3 x 
5x 4 x  5x
by the highest power of x that occurs in the denominator, x4  x x4 x
(c) lim = lim  lim  
namely x. x  6 x 3  x 2 x  6 x 3 x  6

FREE BOOKS FOR JEE & NEET =>(@iitjeeadv)


1.52 Differential Calculus for JEE Main and Advanced

In short, let f(x) = a0xm + a1xm–1 + .... + am –1x + am, lim f(x) = 3 and lim f(x) = 3.
and g(x) = b0xn + b1xn–1 + .... + bn –1x + bn be two polynomial x x

functions. Then We see that the line y = 3 is a horizontal asymptote of the


f (x) a 0 3x 2
(i) lim  if m = n graph of f(x) = .
x2 +1

x  g ( x ) b0
f (x)
0  ∞
(ii) lim if m < n Limits of Form  ∞ 

x  g(x )
f (x) f (x)
(iii) lim   if m > n. Consider the limit lim where f(x) → ∞ and g(x) → ∞

x  g(x ) x →a
g( x )
(It is ∞ if and only if a0 and b0 have the same signs. as x → a. Here a may be finite or infinite.
f (x) 
(iv) lim   if m > n. This is called   form.
x   g ( x ) 
a One way to handle this indeterminate form is to write the
(The correct sign is the sign of (–1)m–n 0 .
b0 1
a 0 x m  a1x m 1  ....
∴ l im limit lim
f (x)
as lim
g( x )
which enters
x  b0 x n  b1x n 1  ... x →a g ( x ) x →a 1

 a0 f (x)
 b , when m  n
ps
 0 0
into   form which can be solved by methods
 0, when m  n 0
el


=  discussed earlier.
eh

a0
 , when m  n and
b0
0 cot x 
 For example, lim 
je

x 0 cot 2 x  
 a0
iit

 , when m  n and 0 tan x 0


 b0 = xlim 0
@

0 tan 2 x  

Thus, the limit of the ratio of two polynomials is one of the
which can be evaluated using standard limits to get 1/2.
following : the ratio of leading coefficients of the polynomials,
if their degrees coincide; infinity, if the degree of the numerator Another way is to find the largest term occurring in the
is greater than that of the denominator; zero, if the degree of numerator and denominator and divide them by this term and
the numerator is less than that of the denominator. evaluate the limit using the basic result:
1
Definition of Horizontal Asymptote If a term p(x) → ∞ as x → a then its reciprocal →∞
p( x )
The line y = L is a horizontal asymptote of the graph of f if as x → a.
lim f(x) = L or lim f(x) = L. x  2 cot x   
x x For example, lim  
x 0 cos x  cot x   
3x 2 x
Consider the graph of f(x) = . 2
x2 +1 cot x x tan x  2
= lim
x 0 cos x
= lim
x 0 sin x  1
= 2.

1
cot x
x 2  4x  5
Example 6: Evaluate lim
x x 3  3x 2  2 x
Solution: Dividing the numerator and denominator by
x3 which is the highest power of x.
The limit of f(x) as x approaches – ∞ or ∞ is 3. 1 4 5
 2 3
As shown in the figure, we see that the value of f(x) approaches = lim
x x x
3 as x increases without bound (x → ∞). Similarly, f(x) x 3 2
1  2
approaches 3 as x → – ∞. These limits at infinity are denoted by x x

FREE BOOKS FOR JEE & NEET =>(@iitjeeadv)


Limits 1.53

000 Solution: In both parts it would be helpful to manip-


= 0
1 0  0 ulate the function so that the powers of x are transformed to
powers of 1/x. This can be achieved in both cases by dividing
x 3  4x 2  5 the numerator and denominator by |x| and using the fact that
Example 7: Evaluate lim
x 2  3x  2
x
x 2 = |x|.
Solution: Dividing the numerator and denominator by (a) As x → ∞, the values of x under consideration are positive,
x3 which is the highest power of x. so we can replace |x| by x where helpful. We obtain
4 5
1  3 x2  2 x2  2 / | x |
x x lim = lim
= lim x  3x  6 x  (3x  6)/ | x |
x 1 3 2
 
x x 2 x3
x2  2 / x2
1 0  0 = lim
=  x  (3x  6) / x



000
Hence, the limit does not exist.
lim 1  2 / x 2
1 2 / x2
x  2 x  3x  4
3 2
= lim = x 
Example 8: Find lim x  3  6 / x lim (3  6 / x )
x 4 x 3  3x 2  2 x  1 x 

Solution: This is an indeterminacy of the form ∞/∞. lim (1  2 / x ) 2


1 0 1
x 
Let us divide the numerator and the denominator of the func- = = 
lim (3  6 / x ) 30 3
ps
tion by the leading power of x, i.e. by x3. x 
x 3  2 x 2  3x  4 (b) As x → –∞, the values of x under consideration are
el
lim
x   4 x 3  3x 2  2 x  1 negative, so we can replace |x| by –x where helpful. We
eh

obtain
1  2 / x  3 / x2  4 / x2 1
je

 lim  . x2  2 x2  2 / | x |
x 4  3 / x  2 / x 2  1 / x3 4 lim = lim
x  (3x  6)/ | x |
iit

x  3x  6

3x 4  2
@

Example 9: lim . x2  2 / x2
x = lim
x  3x  4
8
x  (3x  6) / ( x )
Solution: Let us divide the numerator and the denom- 2
1
inator by x4. = x2 1
lim 
3x 4  2 x  3  6 / x 3  
lim 
x

x
x 8  3x  4  3
x 
 x  
3  2 / x4 3 Example 12: Evaluate lim 
3
x 
 lim   3. x  x
x  x3 x 
1  3/ x  4/ x
7 8 1
 
2 x 1  ....inf inity 
 x 2  2x  1  2 x 1  
Example 10: Find the value of lim  2 
x  2 x  3x  2 
x
  Solution: Let y =
3
x
x 2  2x  1 1 x+ 3
Solution: Since, lim  and
x+
x
x  2 x  3x  2
2
2
x+3 x
2 x 1
2x  1  x 2  2 x  1  2 x 1 1 ....inf inity
lim  1 we have lim  2  = .
x  2 x  1 x   2 x  3x  2  x
  2 Then y 
y
x
Example 11: Find x 2/3
x2  2 x2  2 x 5/ 3
(a) lim (b) lim ⇒ y=
x  3x  6 x 3x  6 x 5/ 3
+y

FREE BOOKS FOR JEE & NEET =>(@iitjeeadv)


1.54 Differential Calculus for JEE Main and Advanced

⇒ y2 + (x5/3)y – x5/3 = 0 1
3  2t 2 .
x 5/ 3
 x 10 / 3
 4x 5/ 3
t2
∴ y = lim
1  2t

t 0
2
t
 x 5 / 3  x10 / 3  4 x 5 / 3
= y (∵ y > 0) 3  2t 2 t
2 = lim
t 0 (1  2 t ) | t |
4x 5/ 3
= 3
2( x10 / 3 + 4 x 5 / 3 + x 5 / 3 ) = =– 3.
−1
e1 x  1
2

2
= Example 15: Evaluate lim
 4  x  2 arc tan x 2  
1  5/ 3   1
 x  e1 x  1
2

0
Solution: lim  
2 2 x  2 arc tan x 2    0 
∴ lim y    1.
x  1 0 1 2 e1 x  1
2

e1 x − 1
2

= lim = lim
Study Tip   x −2 cot −1 x 2
2   tan 1 x 2 
x

2 
1
As x → ∞ we get → 0. Hence in many problems 1 e1 x − 1
2
ps
x = – lim
x 2 1
tan −1 2
el
1 1
we write the expression in terms of and apply → 0.
x
eh

x x
To present this in a simple form we use the substitution x = 1/t. 1
As x → ∞, t → 0+
je

Hence, Put x =
t
iit

lim f(x) = lim f(1/t) and 1 et − 1 t 1


x t0 = – lim . −1
=– .
@

t →0 2 t+
tan t 2
lim f(x) = lim f(1/t), if these limits exist.
  1  2x 
 2 x
x t0 7

Example 16: Find xlim  


For the latter limit we may also write   (1  8x 3 )1/ 3
 
lim f(x) = lim f(– 1/t) using the 1 
x t0  x  2
substitution x = – 1/ t. Solution: lim    1 = −2 = –1.
7
1/ 3
2x (8)1/ 3
x 

1 1/ x 0  1 
Example 13: Evaluate lim x sin  3  8
x  x x 
1 x  cos x
Solution: lim x sin Example 17: Find lim
x  x  x  sin x
x
1 Solution: Here sin x occurs in the given expression and
Put x = As x → ∞, t → 0+ when x → ∞, sin x oscillates between –1 and 1.
t
1 1
sin t Let x = ∴t= and when x → ∞, t → 0
= lim = 1. t x


t0 t
x  cos x
3x 2  2 Now lim
x  x  sin x
Example 14: Find lim
x   x2
1 1
cos 1  t cos
3x 2  2 1 t t
Solution: lim 
x   x2 = lim t 1  lim t
t 0 1 t 0 1
−1 sin 1  t sin
As x → – ∞, t → 0+ 1 t t
Put x =
t 
t 1 t

FREE BOOKS FOR JEE & NEET =>(@iitjeeadv)


Limits 1.55

1 Study Tip
1  t cos
t  1  0 1
= lim
1 1 0 For sufficiently large values of x we have
t 0
1  t sin logax << xp << ax << x! where
t a>1 p > 0 a > 1 x ∈ N.
1 1 What we mean by this is that the function xp is much larger than
∵ when t → 0, t sin → 0 and tcos → 0
t t logax for large values of x when p > 0 and a>1.
sin x cos x As x→ ∞, both logax (a > 1)and xp approach ∞, but xp (p > 0)
Note: lim  lim 0 . is much larger than logax.
x 
x x  x
log a x
Note: The graphs of y = ex and y = ln x suggest that as Hence, lim = 0 , for a > 1.
x → ∞, ex → ∞ much more rapidly than ln x → ∞. Thus, x x
when x is large positive, ex is so much larger than ln x that (ln x )
the quotient ex/(ln x) is large positive. Specially, lim 0
x  x
ex
Hence lim = ∞. np an
x  ln x Also, lim = 0 for p > 0 and a > 1, and lim 0.
n  a n n  n !
The exponential function is notable for its very rapid rate of
n
increase with increasing x. In fact, ex increases more rapidly lim 2 = 0 .
For example, n
as x → ∞ than any fixed power of x. n!
x2 4 x  2nx
ps
Thus, lim x
=0. Example 18: Evaluate lim
xe x  x 2  22 x 1
el

In fact, if n is a positive integer then nx


1 2
eh

ex
 , lim x e n x
0. 4 x  1  2.0   1
lim x  Solution Limit = lim .
02
je

x  xn x  x 2 2
 2
iit

4x
@

Concept Problems I

1. Evaluate the following limits: 3. Evaluate the following limits:
x 4  5x 3  7 tan x tan 1 x
(i) lim (i) lim (ii) lim
x  2 x 5  3x 4  1 

x tan 3x x  x
(2 x  3) (3x  5) (4 x  6) 2
(ii) lim
x  3x 3  x  1 x  sin x
(iii) lim tan–1(x2 – x4) (iv) lim
x x x  cos 2 x
(2 x 3  4 x  5) ( x 2  x  1)
(iii) lim 1
x  ( x  2) ( x 4  2 x 3  7 x 2  x 1) 4. Show that lim x tan 1 = lim x cot 1 x = 1
x x x
2 x 4  3x 2  5 x  6
(iv) lim 5. Evaluate the following limits:
x  x 3  3x 2  7 x  1
cos x 9x 6 − x
2. Evaluate the following limits: (i) lim lim
(ii) x →
2x x 
x2 1
−∞ x3 + 1
(i) lim
x 5 x+ 3

  x 1   x x x x2 1
(ii) lim cos log   (iii) lim (iv) lim
x    x  x  x 1 x 3x  6
[ x ] 1 6. Show that lim xn e–x = 0 for all positive integers. Hence
(iii) lim x 
x  x2 ( 6 x 1)/( 3 x  2 )
 3x  2 x  1  x1000
(iv) lim  2  find lim .
x 
 x x2  ex
x

FREE BOOKS FOR JEE & NEET =>(@iitjeeadv)


1.56 Differential Calculus for JEE Main and Advanced

Practice Problems H


2x 2  3 2x 2 + 3 x ex
7. Evaluate lim and lim (i) lim 2
(ii) lim
x  x5
x 
x  4 x  2 x →−∞ 4 x + 2 ln x
8. Assuming that m and n are positive integers, find ln x ln x
(iii) lim (iv) lim
2  3x n x  3
x x 0 1 / x


lim
1 xm .
x  13. Evaluate the following limits :
9. Evaluate the following limits: x  sin 2 x
(i) lim
x  x
2 x x x 3 5
(i) lim x (2  sin x )
x 3x  2  3 2 x  3 (ii) lim
x  x2 1
 1 1 
 cos x  1  2  x (2  sin 2 x )
4
(ii) lim x
x  2x  (iii) lim
x  x 1
( x  3) 40 (5x  1)10
(iii) lim ln100 x
x  (3x 2  2) 25 (iv) lim
x  x5
5
x 7  3  4 2x 3  1
(iv) lim ( x  1)10  ( x  2)10  ....  ( x  100)10
x  14. Find lim
6
x  x 1  x
8 7
x  x10  1010
ps
10. Find lim logx–1 x . logx (x + 1).logx+1 (x + 2).
x 15. Find the limit of the sequence {xn} which is defined by
el

log x −1 x5. the recursive relation:


logx+2 (x + 3) ........
eh

xn+1 = xn(2 – xn) ∀ n ≥ 1, where x1 is an arbitrary number



2 x  3 3 x  4 4 x  .......  n n x satisfying the inequalities 0 < x1 < 1.
je

11. Find lim


x  (2 x  3)  3 (2 x  3)  ....  n (2 x  3) 1
(3x 4  2 x 2 ) sin  | x |3  5
iit

12. Evaluate the following limits: 16. Evaluate lim x .


| x |3  | x |2  | x |  1
@

x 

1.12 Asymptotes We can also write lim


1
  but there is no corresponding
x 0 | x|
Infinite Limits
1
Consider the function f(x) = 1/x. As x approaches 0 from the statement for lim .
x →0 x
right, 1/x, which is positive, becomes arbitrarily large. The
notation for this is Let f be the function given by f(x) = 3/(x–2). From figure we
can see that f(x) decreases without bound as x approaches 2
1 from the left, and f(x) increases without bound as x approaches
lim .
x 0 x 2 from the right. This behaviour is denoted as
3 3
As x approaches 0 from the left, 1/x, which is negative, has lim = – ∞ and lim = ∞
x 2 x  2 x 2 x  2 

arbitrarily large absolute values. The notation for this is


1
lim   .
x 0 x 

The behaviour of 1/x, is quite different from that of


1/x2. Since x2 is positive whether x is positive or negative and
since 1/x2 is large when x is near 0, we have
1 1
lim   and lim 
x 0 x2 x 0 x2

1
In this case we may write lim   , meaning that
x2 x 0

“as x → 0, both from the right and from the left, 1/x2 becomes
arbitrarily large through positive values.”

FREE BOOKS FOR JEE & NEET =>(@iitjeeadv)


Limits 1.57

If it were possible to extend the graphs toward infinity, we would If the function is a quotient of two functions then the vertical
see that the graph becomes arbitrarily close to the vertical line asymptote occurs at a number where the denominator is 0 and
x = 2. This line is a vertical asymptote of the graph of f. the numerator is not 0.
Example 1: Determine the points x = a for which each Theorem Let f and g be continuous on an open interval
denominator is zero. Then see what happens to y as x → a– containing a. If f(a) ≠ 0, g(a) = 0. and there exists an open
and as x → a+. interval containing a such that g(x) ≠ 0 for all x ≠ a in the
x 1 f (x)
(a) y = interval, then the graph of the function given by h(x) =
( x  2) ( x  2) g( x )
has a vertical asymptote at x = a.
( x  2) ( x  1)
(b) y =
( x  3) 2 Example 2: Determine all vertical asymptotes of the
graphs of the functions:
( x  2) (1  x )
(c) y =
(b) f(x) = x  1 (c) f(x) = cot x
2
x3 (a) f(x) =
1
2( x + 1) x2 1
Solution:
(a) The denominator is zero for x = –2 and x = 1. Solution:
As x → –2–, y → –∞ ; as x → –2+, y → ∞. (a) When x = – 1, the denominator is 0 and the numerator is
As x → 1–, y → ∞ ; as x ∞ 1+, y → –∞. not 0. Hence, we can conclude that x = – 1 is a vertical
(b) The denominator is zero for x = 3 asymptote.
As x → 3–, y → ∞ ; as x → 3+, y → ∞.
ps
(b) By factoring the denominator as
(c) The denominator is zero for x = 3.
(x2 – 1) = (x – 1) (x + 1)
As x → 3–, y → ∞; as x → 3+, y → –∞.
el

we can see that the denominator is zero at x = – 1


eh

Vertical Asymptotes and x = 1. Moreover, because the numerator is not 0 at


If f(x) approaches infinity (or negative infinity) as x these two points, we conclude that the graph of f has two
je

approaches a from the right or the left, then the line x = a is vertical asymptotes.
iit

a vertical asymptote of the graph of f. (c) By writing the function in the form
@

The line x = a is called a vertical asymptote of the curve cos x


f(x) = cot x =
y = f(x) if atleast one of the following statements is true : sin x
lim f(x) = ∞ lim f(x) = ∞ lim f(x) = ∞ we conclude that vertical asymptotes occur at all
x →a x →a − x a  values of x such that cos x ≠ 0 and sin x = 0. Hence,
lim f(x) = – ∞ lim f(x) = – ∞ lim f(x) = – ∞ the graph of this function has infinitely many vertical
x →a x →a − x a  asymptotes. These occur when x = nπ, where n is an
integer.
Note: If a function f has a vertical asymptote as
x = a, then f is discontinuous at a. The theorem requires that the value of the numerator at x = a
be nonzero. If both the numerator and the denominator are 0
at x = a, then we obtain the indeterminate form 0/0, and we
cannot determine the whether a vertical asymptote exists at
x = a or not, without further investigation.

Example 3: Determine all vertical asymptotes of the


x 2  2x  8
graph f(x) = .
x2  4
Solution: We simplify the expression, as follows :

x 2  2x  8 ( x  4)( x  2) x+4
f(x) = = = , x ≠ 0.
x 4
2 ( x  2)( x  2) x+2

At all values of x other than x = 2, the graph of f coincides


with the graph of g(x) = (x + 4)/(x + 2). Thus, we conclude
that there is a vertical asymptote at x = – 2, as shown in the
figure. From the graph, we can see that

FREE BOOKS FOR JEE & NEET =>(@iitjeeadv)


1.58 Differential Calculus for JEE Main and Advanced

x 2 + 2x − 8 x 2  2x  8
lim = − ∞ and lim 
x→2 −
x −4
2 x 2

x2  4

Note: From the definition it follows that the graph of a


function of x can have atmost two horizontal asymptotes – one
Note that x = 2 is not a vertical asymptote.
to the right and one to the left.
We can see in the figure that the function f(x) = 1/(x2 + 1)
Horizontal Asymptotes
approaches the same horizontal asymptote to the right and to
Bearing in mind the geometrical interpretation of the limit the left. This is always true of rational functions. Functions that
of a function, as x → ∞, we can say that if are not rational, however, may approach different horizontal
lim f(x) = , the curve y = f(x) has the straight line asymptotes to the right and to the left.
x 
ps
y =  as its horizontal asymptote for x → ∞.
el
The line y =  is called a horizontal asymptote of the curve
eh

y = f(x) if either
je

lim f(x) = L or lim f(x) = L


x x
iit
@

In a number of cases the function f(x) behaves in a different


manner as x → ∞ and as x → –∞ .

9x 2  1
For instance, for the function f(x) = defined for
x 1
all x ≠ 1, we have

9x 2  1 9x 2  1
lim  3 , lim 3
x  x 1 x  x 1

Therefore, when investigating for horizontal asymptotes of


functions, we usually consider both lim f ( x ) and lim f ( x ) .
x  x 

Example 4: Determine the limits


3x  2 3x − 2
(a) lim (b) lim
x  x →−∞
2x  12
2x 2 + 1
For example, the line y = 2 is a horizontal asymptote to
Solution
2x  1
the right in the curve f(x) = since lim f(x) = 2. (a) For x > 0, we can write x = x 2 . Thus, dividing both
x 1 x
the numerator and the denominator by x produces
By taking the limit as x → – ∞, we see that y = 2 is also a
horizontal asymptote to the left. The graph of the function is 3x  2
lim
shown in the figure. x 
2x 2  1

FREE BOOKS FOR JEE & NEET =>(@iitjeeadv)


Limits 1.59

2
3
x 30 3
= lim =  .
x 1 20 2
2 2
x
2
(b) For x < 0, you can write x = – x . Thus,
dividing both the numerator and the denominator by x
produces Fig. 1
3x − 2 2 2
3− 3−
3x − 2 x x x
= = =
2x 2 + 1 2x 2 + 1 2x 2 + 1 − 2 + 1

− x2 x2 x2
and we can take the limit as follows.
2
3
3x − 2 x Fig. 2
lim
x →−∞
= lim
2x + 1
2 x→ −∞ 1
 2
x2
3−0 3
= = .
− 2+0 − 2 ps
el
eh
je
iit

Fig. 3
@

Use of Asymptotes in Graphing


Some examples of graphing rational functions will show the
usefulness of asymptotes.
1
Example 5: Draw the graph of f(x) = .
x ( x − 1)
Fig. 4
Solution: Note that when x = 0 or when x =1 the denom-
inator x(x–1) is 0. Thus 0 and 1 are not in the domain of the
function. More important, when x is near 0 or near 1 the quo-
tient 1/[x(x–1)] has large absolute value. Thus the lines x = 0 and
x = 1 are vertical asymptotes for the graph. To decide how the
graph approaches these asymptotes, it is necessary to exam-
ine the sign of f(x) for x near 0 and for x near 1. Consider x
near 0. If x is a very small positive number, then
1 1
f (x) 
x x 1

is the product of 1/x, which is a large positive number, and 1/
Fig. 5
(x–1), which is near 1/(–1) = –1. So, for x small and positive, f(x)
is negative and of large absolute value; that is, limx→0+ f(x) = –∞. If x is near 0, but negative, then 1/x is negative and of large
This fact is recorded in Fig. 1 absolute value. Again, 1/(x–1) is near –1. Thus,

FREE BOOKS FOR JEE & NEET =>(@iitjeeadv)


1.60 Differential Calculus for JEE Main and Advanced

1 1
lim f ( x )  lim 
x 0 
x 0 
x x 1
This fact is recorded in Fig.2.
Next, how does f(x) behave near x = 1? 4x 2
y=
Consider first x near 1 but larger than 1. x2 +1
1 1
Then f ( x ) = is a large positive number, since 1/x is
x x −1 The notion of an asymptote can be extended to include curves
near 1 and x – 1 is a small positive number. Thus, as well as lines. Specifically, we say that f(x) is asymptotic
to g(x) as x → ∞ if
lim f ( x )  
x 1 lim [f(x) – g(x)] = 0
x
Similarly, lim f ( x )   and that f(x) is asymptotic to g(x) as x → –∞ if
x 1
lim [f(x) – g(x)] = 0
These two facts are recorded in Fig. 3. Piecing together these x
three figures suggests that the graph of f for x in or near the Informally stated, if f(x) is asymptotic to g(x) as x → ∞, then
interval (0,1) looks something like Fig. 4. How high the curve the graph of y = f(x) gets closer and closer to the graph of
goes for x in (0,1) is the type of question answered in a later y = g(x) as x → ∞, and if f(x) is asymptotic to g(x) as x →
chapter with the aid of derivative. –∞, then the graph of y = f(x) gets closer and closer to the
How does f(x) behave when |x| is large? graph of y = g(x) as x → –∞.
For example, if
ps
1 1
Since lim  0 and lim 0 f(x) = x2 +
2
and g(x) = x2
x  x ( x  1) x  x ( x  1)
x −1
el

then g(x) is asymptotic to g(x) as x → ∞ and as x → –∞ since


eh

the x axis is a horizontal asymptote (both for x positive and


for x negative). In both cases the graph approaches the x axis 1
lim [f(x) – g(x)] = lim =0
je

from above, not from below, since the function is positive when x x x − 1
|x| is large. The graph of f must look something like Fig. 5.
iit

1
lim [f(x) – g(x)] = lim =0
Example 6: Discuss and sketch the graph of the x  x  1
@

x
4x 2 This asymptotic behaviour is illustrated in the following figure,
equation y = 2 .
x +1 which also shows the vertical asymptote of f(x) at x = 1.
Solution: The graph is symmetric w.r.t. the y axis, since
x occurs with even powers only.
x2
Since, 0 ≤ < 1, 0 ≤ y < 4 for every number x.
x +1
2

Thus the graph lies between the lines y = 0 and y = 4.


4x 2 4
Since, lim = lim = 4,
x 1 x  1  1 / x
2 2
x 
the line y = 4 is an asymptote of the graph of the function. The
graph may be drawn as shown in the figure.

Concept Problems J

1. Determine the vertical and horizontal asymptotes of the
x2 −1 x2 1
graph of the given functions : (i) f(x) = (ii) f(x) =
x2 − 4 x2  4
2x 2
2x 2 3. Use asymptotes to sketch the graphs:
(i) f(x) = (ii) f(x) =
( x + 2) 2
x +9
2 1 1 x ( x  1)
(i) y = (ii) y = (iii) y =
x −x
2
x −x
4 2
x2 1
2. Sketch the graph of each of the following functions,
indicating the vertical and horizontal asymptotes. 4. Sketch the graph of (i) y = xe–x (ii) y = x ln x.

FREE BOOKS FOR JEE & NEET =>(@iitjeeadv)


Limits 1.61
3(n  1)
1.13 Limit of a sequence Example 3: Evaluate lim ,n∈N
n  (n  1)3 3n
Here we consider limit of a sequence f(n) as n tends to ∞. (3n  1)(3n  2)(3n  3)
Solution: Limit = lim
We say that the sequence approaches the limit λ if for all n  (n  1)3
sufficiently large integral values of n the corresponding values
of f(n) become arbitrarily close to the number λ.  1  2  3
3  n 3  n 3  n 
If f(n) → ∞ we may say that f(n) diverges to ∞ . If f(n) = lim       = 27
does not tend to a limit or to ∞ , then it oscillates finitely n 
 1
3

or infinitely: in this case we say that the sequence oscillates 1  n 


 
finitely or infinitely.
The only difference between the ‘tending of n to ∞’ and Example 4: Find
‘tending of x to ∞’, is that x assumes all values as it tends to 2n n 1 n n (1) n
∞ , i.e. that the point x coincides in turn with every point of lim cos  
n  2n 2  1 2n  1 1  2n n 2  1
the number line to the right of its initial position, whereas n
tended to ∞ by a series of jumps. We can express this distinction  2 1  11/ n 
by saying that x tends continuously to ∞. Solution: lim   n cos  2  1 / n 
n   2  1 / n 2   
There is a very close correspondence between functions of x and
functions of n. Every function of n may be regarded as a selection 1 (1) n 1
from the values of a function of x.    
(1 / n  2) (1  1 / n ) n 
2
We can easily see that [x] → ∞ , x–[x] oscillates finitely, and
     
   
[x] + √{x–[x]} → ∞ as x → ∞. 1  2   11/ n 
ps

n  2  1 / n 2 
One simple remark may be inserted here. The function f(x) = x = lim cos  
n   2 1 / n 


– [x] oscillates between 0 and 1, as is obvious from its graph. It
el
is equal to zero whenever x is an integer, so that the function f(n) 1 (1) n 
  
eh

derived from it, is always zero and so it tends to the limit zero. (1 / n  2) (1  1 / n 2 ) 
The same is true if f(x) = sin xπ.
je

2 1 1 1 
f(n) = sin nπ = 0. = 0    cos    = 0.
iit

It is evident that f(x) → l or f(x) → –∞ involves the corresponding  2 2 2 1 


@

property for f(n), but that the converse in often untrue. Example 5: Evaluate
Note: If f(n) is a sequence such that lim f (n )  a , then 1.n + 2 .(n − 1) + 3(n − 2)..... + n .1
n  lim
n →∞ n3
we have lim f (n  1)  a .
n 
n

n2 1  n
 r(n  r  1)
Example 1: Find lim r 1
Solution: Limit = lim
n  4 n3
n 1  n
3 n 

n n
n  11 / n2  1 / n  (n  1) r   r 2
 
r 1 r 1
 
Solution: lim = lim
n  3/ 4 4
n 11/ n  1/ n
3 4
n  n3
(n  1)n (n  1) n (n  1)(2n  1)

n1/ 4
 11/ n 2
 1/ n  . = lim
2

6

 11/ n 1/ n 
= nlim

n  n3
4 3
4
1 1 1
=   .
n! 2 3 6
Example 2: Find lim
n  ( n  1)!  n !
Example 6: If the value of
n!  3  3  3  3
Solution: lim lim n 3  1   1   1   .... 1  
n  (n  1)!  n ! n   7  8  9   n 
n! 1 is k then find the sum of digits of k.
(n  1)! n 1 n 3
= lim  lim 0 . Solution: lim n3
4 5 6 7
    ......
n  n! n  1
1 1 n 7 8 9 10 n
(n  1)! n 1

FREE BOOKS FOR JEE & NEET =>(@iitjeeadv)


1.62 Differential Calculus for JEE Main and Advanced
2n 1 n

n 1 n 1 0 1
4  5  6n 3 Solution: Limit = lim  1.
= nlim = 120 = k. n  2n 1
 (n  2)(n  1)n 1

n 1
Thus the sum of digits of k = 3.
Example 10: Find

 1 
Example 7: Find the value of  ln 1  2  . lim  1  1   1  1   1  1  ....  1  1 
n       
n 2  n   22   32   42   n2 

 1  Solution: We have limit
Solution: We have  ln 1  n 2  n 1 n n 1
n
n2 1 n
n 2
= lim  n2
= lim  n · n
 n2 1 
 
  n 1   n 1   n  n  2 n  n 2 n 2
=  ln  2  =
   ln   n   n    1 2 3 n 1   3 4 5 n 1 
n 2  n  n 2  
= lim  · · ...   2 · 3 · 4 ..... n 
n   2 3 4 n   

  n 1   n 1  
=   ln  n 
 ln 
 n 
 1 n +1 1
n 2  = lim · = .
n  n 2 2

  n 1   n 
=   ln    ln   Example 11: Calculate lim sin( n 2  1) .
n 2   n   n 1   n 

Solution: We have
 1 2  2 3
ps
=  ln  ln    ln  ln  + .....  1   1 1
1/ 2

 2 3  3 4 n 2  1  n 1  2   n 1  . 2  ........ 
 n   2 n 
el

1  n 
eh

= ln – lim ln   1
2 n   n 1  = n + 2n + .........
je

1   
= ln – ln 1 = – ln 2 .
Hence sin(  n  1)  sin  n   ....... 
2
iit

2
 2n 
@

xn
Example 8: Evaluate lim .   
n  n ! = (–1)n sin   ....... 
 

Solution: Since x is fixed number, there will be a pos-
2n
itive integer N such that |x| < N.    
|x| The sequence {(–1)n} is bounded and sin   .......   is
Let
N
= q ( 0 < q < 1). Now, we have   2n 
n infinitesimally small, and therefore the product of these two
x x x x x x x x sequences is an infinitely small sequence. Thus,
 . . ....... . . ...
n! 1 2 3 N 1 N N 1 n
lim sin( n 2  1)  0
n  .
|x| |x| |x| |x| n · 3n 1
< . . ........ . q. q.....q 
N −1 Example 12: If lim

1 2 3 n  n ( x  2)  n · 3n 1  3n
n
3
|x|
[since ≤ q for n ≥ N] where n ∈ N, then find the number of integral values of 'x'.

N Solution: We have
| x |N 1 n  N 1
= q
( N  1) ! n · 3n 1
lim  .
n  n ( x  2)  n · 3n 1  3n
n
3
| x |N−1
where is a fixed quantity independent of n, while Dividing both numerator and denominator by n. 3n,
( N − 1) !
1 1
qn–N+1 approaches 0 as n approaches ∞. lim n

n 
xn  x2 1 3
Hence, we have lim = 0.  3  3 n
n  n !  
Clearly for the limit to exist, we must have
2n 1  n n
Example 9: Evaluate lim x−2
n  n  1  2n –1< <1 ⇒ –1<x<5
3

FREE BOOKS FOR JEE & NEET =>(@iitjeeadv)


Limits 1.63

∴ The possible integral values of 'x' are 0, 1, 2, 3, 4 5. If f(n) and g(n) both oscillate finitely as n tends to infinity,
Hence there are 5 integral values of 'x'. then f(n) + g(n) must tend to a limit or oscillate finitely
as n tends to infinity.
Behaviour of Two Functions as n Tends to Infinity
For example
Results (i) f(n) = (–1)n, g(n) = (–1)n+1,
1. If f(n) → ∞ as n tends to infinity and g(n) ≥ f(n) for all (ii) f(n) = g(n) = (–1)n.
values of n, then g(n) → ∞ as n tends to infinity. 6. If f(n) oscillates finitely, and g(n) infinitely as n tends to
2. If f(n) → 0, as n tends to infinity and |g(n) | ≤ |f(n) | for infinity, then f(n) + g(n) oscillates infinitely as n tends to
all values of n, then g(n) → 0 as n tends to infinity. infinity.
3. If lim |f(n)| = 0, then lim f(n) = 0. 7. If both f(n) and g(n) oscillate infinitely as n tends to infinity,
n n
then f(n) + g(n) may tend to a limit, or to ∞ , or to –∞ , or
4. If f(n) tends to a limit or oscillates finitely as n tends to oscillate either finitely or infinitely as n tends to infinity.
infinity , and |g(n)| ≤ |f(n)| when n ≥ N, then g(n) tends Theorem If lim f(n) = a and lim g(n) = b, then
to a limit or oscillates finitely as n tends to infinity. n n

5. If f(n) tends to ∞ or to –∞ or oscillates infinitely as n tends lim f(n)g(n) = ab.


n
to infinity , and |g(n)| ≥ |f(n)| when n ≥ N, then g(n) tends
to ∞ or to –∞ or oscillates infinitely as n tends to infinity. Theorem If lim f(n) = a and lim g(n) = b and b is not
n n
Behaviour of Sum of Two Functions as n Tends f (n ) a
zero, then lim = .
to Infinity n g(n ) b
ps
Theorem If f(n) and g(n) tends to limits a, b, then f(n) + Results
g(n) tends to the limit a + b.
el
1. If f(n) → ∞ and g(n) oscillates finitely as n tends to
The argument is roughly like this : ‘when n is large, f(n) is
infinity, then f(n)g(n) must tend to ∞ and –∞ or oscillate
eh

nearly equal to a and g(n) to b, and therefore their sum is


infinitely as n tends to infinity.
nearly equal to a + b’.
je

Examples of these three possibilities may be obtained by


Results
iit

taking f(n) to be n and g(n) to be one of the three functions


1. If f(n) tends to a limit, but g(n) tends to ∞ or to – ∞ or 2 + (–1)n, –2–(–1)n, (–1)n
@

oscillates finitely or infinitely as n tends to infinity, then 2. If f(n) and g(n) oscillate finitely as n tends to infinity,
f(n) + g(n) behaves like g(n) as n tends to infinity. then f(n)g(n) must tend to a limit (which may be zero) or
2. If f(n) → ∞ and g(n) → ∞ or oscillates finitely as n tends oscillate finitely as n tends to infinity.
to infinity, then f(n) + g(n) → ∞ as n tends to infinity. In this For example,
statement we may obviously change ∞ into –∞ throughout. (i) f(n) = g(n) = (–1)n,

3. If f(n) → ∞ and g(n) → –∞ as n tends to infinity, then f(n) (ii) f(n) = 1 + (–1)n, g(n) = 1 – (–1)n,
+ g(n) may tend either to a limit or to ∞ or to –∞ or may 1 1
oscillate either finitely or infinitely as n tends to infinity. (iii) f(n) = cos nπ , g(n) = sin nπ
3 3
These five possibilities are illustrated in order by 3. A particular case of the theorem which is important is
(i) f(n) = n, g(n) = –n, that in which g(n) is constant. The theorem then asserts
(ii) f(n) = n2, g(n) = –n,
(iii) f(n) = n, g(n) = –n2, simply that lim kf(n) = ka if lim f(n) = a.
n n
(iv) f(n) = n + (–1)n , g(n) = –n, 4. If f(n) → ∞ as n tends to infinity then kf(n) → ∞ or kf(n)
(v) f(n) = n2 + (–1)nn , g(n) = –n2. → –∞ , according as k is positive or negative, unless k = 0,
4. If f(n) → ∞ and g(n) oscillates infinitely as n tends when of course kf(n) = 0
to infinity, then f(n) + g(n) may tend to ∞ or oscillate for all values of n and lim kf(n) = 0. And if f(n)
infinitely as n tends to infinity, but cannot tend to a limit, n

or to –∞, or oscillate finitely. oscillates finitely or infinitely as n tends to infinity, then


For g(n) ={f(n) + g(n)} – f(n); and, if f(n) + g(n) behaved so does kf(n), unless k = 0.
in any of the three last ways, it would follow, from the Note:
previous results, that g(n) → –∞ , which is not the case.
1. If f(n) is positive and f(n + 1) ≥ K.f(n), where K > 1, for
As examples of the two cases which are possible, consider
all values of n, then f(n) → ∞ as n tends to infinity.
(i) f(n) = n2, g(n) = (–1)n n, (ii) f(n) = n, g(n)= (–1)nn2. Here For f(n) ≥ K.f(n–1) ≥ K2f(n–2) ≥ ... ≥ Kn–1f(1) from which
again the signs of ∞ and –∞ may be permuted throughout. the conclusion follows at once, since Kn → ∞

FREE BOOKS FOR JEE & NEET =>(@iitjeeadv)


1.64 Differential Calculus for JEE Main and Advanced

Note that the same result is true if the conditions are f(n) oscillates infinitely if x ≤ –1
satisfied only when n ≥ N. and f(n) → 0 if –1 < x < 0.
2. If f(n) is positive and f(n + 1) < Kf(n), where 0 < Example 14: Prove that if x is positive then
K < 1, then lim f (n ) = 0. This result is also true if the n
x → 1 as n → ∞
n 
conditions are satisfied only when n ≥ N. Solution: Suppose, for example, x < 1.
3. If |f(n + 1)| ≤ K |f(n)| when n ≥ N and 0 < K < 1, then Then x , x , 3 x ,..... is a decreasing sequence, and n x > 1
lim f (n ) = 0. for all values of n. Thus n x → l , where l ≥ 1. But if l > 1
n 
f (n 1) we can find values of n, as large s we please, for which n x
4. If f(n) is positive and lim = l > 1, then f(n) → → l or x > ln, and since ln → ∞ as n → ∞, this is impossible.
n  f ( n )

∞ as n tends to infinity. Example 15: Prove that n


n → 1as n → ∞.
f (n +1) Solution:  1)  n
n 1 ( n n
For we can determine N so that >K>1
f (n ) if (n+1)n < nn+1 or (1 + n–1)n < n, which is certainly satisfied
when n ≥ N ; we may, for example take K half way if n ≥ 3 . Thus n n decreases as n increases from 3 onwards
between 1 and l and apply 1. and since it is always greater than unity, it tends to a limit
f (n 1) which is greater than or equal to unity. But if n n → l , where
5. If lim = l, –1 < l < 1,
n  f ( n ) l > 1, then n > ln , which is certainly untrue for sufficiently
then lim f (n ) = 0. large values of n, since ln /n → ∞ with n.
n 
ps
Example 16: The sequence of real numbers x1, x2,
Example 13: Determine lim nrxn, where r is any x3, ..... satisfies lim (x2n + x2n+1) = 315 and lim (x2n +
el
n n  n 
positive integer.
Evaluate lim (x2n/x2n+1).
eh

x2n–1) = 2003.
Solution: Let f(x) = nrxn. n 
Solution: We have lim (x2n+1 – x2n–1)
je

If x = 0 then f(n) = 0 for all values of n, and lim n x = 0. Inr n


n n 
all other cases
iit

= lim [(x2n + x2n+1) – (x2n + x2n–1)]


f (n  1)  n 1  n 
 lim  x  x
@

lim
n  f (n ) n   n  = 315 – 2003 = – 1688 ; it follows that x2n+1 → ∞ as

n → ∞.
First suppose x is positive. x x  x 2 n 1
Then f(n) → ∞ if x > 1 and f(n) → 0 if x < 1. Then lim 2 n  lim 2 n  1  1
n  x 2 n 1 n  x 2 n 1
If x = 1, then f(n) = nr → ∞.
since x2n + x2n+1 → 315 while x2n+1 → –∞.
Secondly, suppose x is negative. 1. Evaluate the following limits :
Then |f(n)| = nr |x|n tends to ∞ if |x| ≥ 1 and to 0 if |x| < 1.

Concept Problems K

2n  5.6n 12 + 22 + 32 + .... + n 2
(i) lim (ii) lim
n  3n  6n n n3

(n  1) 4  (n  1) 4 5n 1  3n  22 n
(ii) lim (iii) lim
n  3n 3  5 n 5n  2n  3n  3
1
n 3  3n 2  n  3
(iii) lim 2n
n n2  n  6 (iv) lim
n  1
2n 1
( n 2  1  n )2
(iv) lim 
n  3
n6  1 3. Using the set x n  (2n  1) , show that the sequence
2
2. Evaluate the following limits: {sin xn} diverges. Hence prove that the function sin x
 1 does not have a limit as x → ∞.
(i) lim n 2 1  cos  4. Prove that the following sequences tend to zero as n
n  n
tends to infinity:

FREE BOOKS FOR JEE & NEET =>(@iitjeeadv)


Limits 1.65

n 3n 5. Prove that n (n !)   as n → ∞.
(a) x n = n
, (b) xn = , (c) x n  n 51
2 n!

Practice Problems I


6. Evaluate the following limits:  1 1 1 
14  24  34  .....  n 4 (iii) lim    ......  
(i) lim n   1.2.3 2.3.4 n (n  1)(n  2) 
n  n5
9. For x > 0 and ≠ 1 and n ∈ N, evaluate,
(n  1)  (n  1)3
3
 1 1
(ii) lim   ......
n  ( n  1) 2  ( n  1) 2 lim 
n  log x 2 . log x 4 log x 4 . log x 8
4
n5  2  3 n 2  1 1 
(iii) lim  
. log x 2 
n  5
n 4  2  n3  1 log x 2 n 1 n

(iv) lim (n  2)! (n  1)!



1 9 
n  ( n  2)! ( n  1)! 10. If f(n + 1) = f ( n )   , n ∈ N and f(n) > 0 for
2 f (n ) 
n 3  2n 2  1  3 n 4  1 all n ∈N, then find lim f (n )
7. Find lim n 
n  4
n 6  6n 5  2  5 n 7  3n 3  1
ps
8. Evaluate the following limits: 11. Evaluate the following limits:
1 1 1  1 2 n 
1    .....  n (i) lim   ...... 
el

2 4 n 1  n 2
1 n 2 
1 n2 
lim 2
eh

(i)
n  1 1 1  1  2  3  4  .........  2n 
1    .....  n
3 9 (ii) lim  

je

3 n  n2 1 
 1 1 1 
iit

(ii) lim    .......    x x x 


n   1.3 3.5 (2n  1)(2n  1)  (iii) lim  cos cos .......cos n 
 2 
@

n 2 4

1.14 Limits of Forms (0 x ∞) and (∞ – ∞) 1


Example 1: Evaluate lim x tan
x  x
(0 x ∞) Form Solution: This is an ∞. 0 indeterminate form, but we
Consider the limit lim f(x).g(x) where f(x) → 0 and convert it into a 0/0 form by using the substitution u = 1/x
x →a
and noting that u → 0 as x → ∞
g(x) → ∞ as x → a. Here a may be finite or infinite.
This is called (0 x ∞) form. 1 1 tan u
lim x tan  lim tan u  lim = 1.
One way to handle this indeterminate form is to write the limit x  x u 0 u u 0 u

f (x) 0
lim f(x).g(x) as lim which enters into   form or Example 2: Evaluate lim x (nx  n ( x  2))
x →a x →a 1 0 x 

g( x ) x 1
g( x ) Solution: Limit = lim x.n Put x =
lim which is in    form. x  x2 t
x →a 1 
  n
1
f (x)
= lim 1  2 t = lim n (1  2 t ) .2 = – 2.
These can be solved by methods discussed earlier. t 0 t t 0 
2t
For example, lim sin2x. cotx is in (0 x ∞) form.
x→0  b 
sin 2 x 0 Example 3: Evaluate lim a x sin  x , a > 1
lim s i n 2 x . c o t x = lim  0  which can be x  a 
x→0 x  0 tan x  
evaluated using standard limits to get 2.   b 
 sin  x  
1
Solution: Limit = lim   a   .b  b .
1
(e x  1) x   b 
and lim x (e x  1) = lim = 1.  x 
x  x  1/ x  a 

FREE BOOKS FOR JEE & NEET =>(@iitjeeadv)


1.66 Differential Calculus for JEE Main and Advanced

 x 1    1
3
Example 4: Evaluate lim x  tan 1  .
x   x  2 4   n t 
Limit = l im  2 
 x 1  t  t
Solution: Limit = lim x  tan 1  tan 1 1
x   x2    nt 
3

= l im
 x 1  t  t2
 1 
= lim x.tan 1  x  2
3
  nt 
x 
 1  x  1  =  lim  2 / 3  = 0.
t   t 
  n 1
Example 9: Evaluate lim n2 ( n a  a ) , a > 0.
 1  1  
n
 tan  
 2x  3     1  1 n 1
= lim x  , lim n2 ( n a 
  2 x  3 
x   1 Solution: Put n = a)
y n
 2x  3 
 
ay  a
y
y 1

= lim
1 1 y→0 y2
= lim  .
x  3 2
2 1  a
y
y 1 1
= lim a  
x y
y→0 y2
Note: lim xnx  0
 
x 0
ay 1  a
y2
ps
y 1

nt 1 = lim
Proof: We have lim 0. Put t =
el
y→0 2
t  t x y
eh

y2 y2

n
1 a y1
1 a y1
1
je

x 0 = lim = lim
⇒ .  y  1
lim y→0 2 y→0 y2
x 0 1 y
y 1
iit

x = ln a.
@

Example 10: Evaluate:


⇒  lim xnx  0
x 0  
  1  1  1 
lim n 2   1  cos   1  cos   1  cos  .... 
⇒ lim xnx  0 . n 
  n  n  n 
x 0  
  
Example 5: Evaluate lim sin xn (tan x ) 1
x 0
Solution: Put =x
n
Solution: Limit = l i m cos x (tan x . n (tan x ))
x0  
  1  1  1 
= 1 × 0 = 0. lim n 2  1  cos  1  cos  1  cos  .... 

x 2 n ( x  x 2 ) n 
  n  n  n 
Example 6: Evaluate l i m 
.  
x0 tan x
x x.( x  x 2 ) (1  cos x ) (1  cos x ) (1  cos x ).....
Solution: Limit = l i m . n ( x  x 2 )
x0 tan x ( x  x 2 )

= lim
x 0 x2
= lm
x 0
x
.
1
tan x 1  x

( x  x 2 ) n ( x  x 2 )  (1  cos x ) 2
1

1 1
 _.....
2 2 23 1  cos x
= 1 × 1 × 0 = 0. = lim = lim
x 0 x2 x 0 x2
Example 7: Evaluate l i m x 3 (nx ) 2
x0
Solution: Limit = l i m x.( xnx )2 = 0 × 0 = 0. (1 − cosx ) . (1 + cos x )
= lim
x0
x →0 x2 (1 + cos x )
Example 8: Evaluate l i m x 2 (nx )3
x0 2
 sin x 
= (1) ⋅
1 2 1 1
1 = xlim   ⋅ xlim = .
Solution: Put x = . As x→ 0, t → ∞. →0  x  → 0 (1 + cosx ) 1+1 2
t

FREE BOOKS FOR JEE & NEET =>(@iitjeeadv)


Limits 1.67

One should transform the difference f1(x)–f2(x) into the product


x 2 (1  a )  x (a  b)  (1  b)
 f (x)  = lim 2
f1(x) 1  2  and first evaluate the indeterminate form x  ( x  1)
 f1 ( x ) 
Since, limit of the above expression is a finite nonzero number,
f2 (x)   degree of numerator = degree of denominator
 the form   ;
f1 ( x )   ⇒ 1–a=0 ⇒ a=1
∴ Putting a = 1 in the limit we get,
f2 (x)
if lim = 1, then we reduce the expression to the  x (1  b)  (1  b)
x → a f1 ( x ) lim 2
x  x 1
1
f2 (x) ⇒ –(1 + b) = 2 ⇒ b = –3
f1 ( x )  0 Hence, a = 1 and b = –3 .
form  the form 0  . Example 14: Find a and b if
1  
f1 ( x ) lim
x 
 (x 2

 x  1)  ax  b  0

Example 11: Evaluate lim 


1

1 

x x 1 x  x 0  x 

Solution: We have lim ( x 2  x  1)  ax  b  0 
1
Put x = −
0
Solution: Limit = lim 1  x  1 0
t
x 0 x 1  x     1 1  a 

 1 x 1   1  x  1 lim   2   1   b   0
t 0
  t t  t 
ps
= lim
x 0 x 1  x  1  x  1
 (1  t  t 2 )  a  bt 
el

⇒  
1 x 1
1 lim  0
eh

t 0 t
= lim
x 0 x 1  x .2
= . 
 

2
As t → 0, numerator → 1 + a and denominator → 0
je

Example 12: Find Since limit is finite, numerator must tend to 0.


iit

lim ( x 2  8x  3  x 2  4x  3 ) . ∴ 1 + a = 0 ⇒ a = –1
@

x
Substituting a = –1 in (1), we get
Solution: Here we have an indeterminacy of the form  (1  t  t 2 )  1  bt 
 
∞–∞. Multiply and divide the given expression by lim  0
t 0 t

 

x 2 + 8x + 3 + x 2 + 4x + 3 .
 (1  t  t 2 )  1 
lim x 2  8x  3  x 2  4x  3 ⇒ lim  b  0
x t 0  t 
 
 lim ( x  8x  3  x  4x  3 ) ( x  8x  3  x  4x  3 )
2 2 2 2

x
( (1  t  t 2 )  1)( (1  t  t 2 )  1)
x 2  8x  3  x 2  4 x  3 ⇒ lim b
t 0
t ( (1  t  t 2  1)
x 2  8x  3  x 2  4 x  3
 lim  
x   1 t  t2 1 
x 2  8x  3  x 2  4 x  3 ⇒ lim  b
t 0

 t ( (1  t  t 2
)  1) 

4x
 lim 1
x 
x  8x  3  x 2  4 x  3
2 ⇒ = b.
2 1
4x Hence a = –1 and b = .
 lim =
4
= 2. 2
x 
1  8 / x  3x  1  4 / x  3 / x 2
2
2 Example 15: Find lim ( x  4 x  x  4 x )
2 2
x 

 x2 1 
Example 13: If lim   ax  b   2 , find the Solution: lim ( x 2  4 x  x 2  4 x )
x   x  1  x 
values of a and b.  
 x 1
2   x 2  4x  x 2  4x  x 2  4x  x 2  4x 
 
Solution: We have lim   ax  b  = xlim
x  x  1  
x  4x  x  4x
2 2
 

FREE BOOKS FOR JEE & NEET =>(@iitjeeadv)


1.68 Differential Calculus for JEE Main and Advanced

On considering the degree of the leading term, it follows that


( x 2  4x )  ( x 2  4x ) 1 5
= lim –p – = –2. So p = .
x 
x 2  4x  x 2  4x 3 3
 1 
Example 18: Evaluate lim  cos ec3 x  3 
8x x 0  x 
= xlim

x 2  4x  x 2  4x  1 
Solution: We have lim  cos ec x  3  , [form∞-∞]
3
8 x 0  x 
lim
x 3  sin 3 x
= lim 
= x 
x  4x x 2  4x
2 1 1 
    lim
x2 x2 x  0  sin 3 x 3  x  0 x 2 sin 3 x
[When a negative number is to be written as square root of  x 3  sin 3 x   x 3 
a number, negative sign is put before the sign of square root, = lim   . 
x0 
 
for example – 4 = – 16 ].  x6   sin x  
8 8
1im  4. x 3  sin 3 x
x 4 4 2  lim
1  1 x0 x6
x x
( x  x sin x  sin 2 x ) ( x  sin x )
2
= lim .
Example 16: Find xlim ( 3 x3  x 2  3 x3  x 2 ) . x 0 x2 x4

 sin x   sin x    x  sin x 
2

Solution: xlim = lim  1   .


x 0   x   x    
3 2 1/3 3 2 1/3
  ((x + x ) – (x – x ) ) x4 

ps
(x3  x 2 )  (x3  x 2 )  
= xlim x3 x5
 (x  x )
3 2 2/3
 ( x 3  x 2 )1/ 3( x 3  x 2 )1/ 3 ( x 3  x 2 ) 2 / 3  x  x   ....... 
el
3 ! 5 !
= lim 3  
2x 2
eh

x 0 x4
= xlim
  2 1 1 2
 1 x 
 1  1 3 1 3  1 3  1 3 
je

 1   1    1   3   ....... 
 x   x  x  x  =
lim 
3! 5! =∞.
iit

  x 0 x
 1 1 
@

2 Example 19: Evaluate lim   .


= . x1  ln x x 1 
3
Example 17: Find p so that  1 1 
Solution: lim   
x1  ln x x 1 
lim xp (3 x  1  3 x  1  2 x) 3
x   1 1
is some nonzero real number. = lim   
h0  ln (1  h ) h 
1
Solution: Make the substitution t = . Then the limit h  ln(1  h ) h
equals to x = lim lim
h 0 h2 h 0 ln (1  h )
 1 1 1
lim t  p  3  1  3  1  2 3  Put ln (1 + h) = t
t 0  t 
 t t ⇒ 1 + h = et
 lim t  p 
t 0
1
3
 31  t  31  t  2 .   = lim
et  1  t
2
= lim
et  1  t
...(1)
t 0
 et  1  2 t 0 t2
We need the degree of the first nonzero term in the Maclaurin's   ·t
 t 
expansion of 31  t  3 1  t  2 . We have e t  1  t
Also,  = lim ...(2)
1 1 t2

t 0
31  t 1 t  t 2  ......... ,
3 9 Adding (1) and (2)
3 1 t  1 
1 1 et  e t  2 e 2 t  2e t  1
t  t 2  ......... 2 = lim = lim
3 9 t 0 2
t t 0 t2
2 2 2
It follows that  t  31  t  2   t  .........  et  1 
⇒ = 1
31
. = lim   = 2
t 0 
9
 t  2

FREE BOOKS FOR JEE & NEET =>(@iitjeeadv)


Limits 1.69

Example 20: Evaluate xlim



(x − n cosh x)
x

= lim (1) n cos n   n 2  n
n 

e e
 
x
where cosh x = . = lim (1) n cos   n  n 2  n 
2 n  
 n 
  ex  e x  = (1) nlim
n
cos  
Solution: xlim

 x   n     
n n n
2 

 2    
 
= xlim
  [ n e − n (e + e ) + n 2 ]
x x −x
 

ex = (1) lim cos 
n  =0
lim n + n 2
n   1 
= x
e  e x
x  1 1 
 n 
1   Example 23: Evaluate
= xlim  
  n  1  e  2 x  + n 2
   1 
 n 4
 
= 0 + n 2 = n 2. lim  4 (r  1)(r  2)(r  3)   n 
n   
  r 1  
Example 21: Find  
Solution: Limit
lim  n 2  n  1   n 2  n  1  
n      1 
  (n  1)(n  2)(n  3)(n  4)  4 
where [.] denotes the greatest integer function. = lim  4.   n
n   4 
ps
 
Solution: n < n2 + n +1 < n + 1
el
1
Put n =
 n2  n 1  = n t
eh

Hence
  1
je

= lim {(1  t )(1  2 t )(1  3t )(1  4 t )} 4  1


∴  = lim  n2  n 1  n 
iit

t 0 t
n
1
 
@

n 1
 = 1 = lim (1  10 t  .......)  1
4
= lim 
n  
 n  n 1  n 
2 2 t 0 t
10
1  t  .......  1
Example 22: Evaluate lim cos  n  n , when n is
an integer.
n 
2
  =
lim
t 0
4
t

 
10 5
Solution:  = lim cos  n  n
2 = = .
n  4 2

Concept Problems L

1. Evaluate the following limits: lim x.ln(x + x2)
(i) 
x0

1/10 7
(i) lim x . sin (ii) lim sin x .ln (tan x )
x →0 x x →0

x (iii) xlim ln x ln( x  1)


(ii) lim (a  x ) sec 1
x a
  
2a
3. Evaluate the following limits:
 ax x 
(iii) lim  sin
2a 
tan
x a  2 (i) lim  1  12 
x 2  x2 
x x  x3  8 
(iv) lim 3 sin(3 )
x 
(ii) xlim ( 3 x 1  3 x )
2. Evaluate the following limits: 

FREE BOOKS FOR JEE & NEET =>(@iitjeeadv)


1.70 Differential Calculus for JEE Main and Advanced

which clearly behaves like 3x + 2 for large x. Thus the


(iii) lim ( 4 x 2  9  2 x ) limit in question is 2, not 0.” Settle the argument.
x 

(iv) l im ( x  1  x ) x 7. Evaluate the following limits:


x 
lim n (
(i) n
4. Evaluate the following limits: n 2 + 1 – n)

(i) lim (cosec x  cot x )  1  2  3  ......  n n 


(ii) lim   
x0
n   n2 2
(ii) lim (cosec x  cot x )
x   1  3  5  7  ...  (2n  1) 2n  1 
(iii) lim   
  n   n 1 2 
(iii) lim (sin x  cos x ) tan   x
x  / 4  4  (iv) lim
3
n3  n  3 n3  1
n 
lim 1 1
(iv) x→0 – 8. Evaluate the following limits:
sin x tan x
5. Evaluate the following limits: (i) lim ( x − ln( x 2 + 1))
x →∞
(i) lim ( 3x 2  1  2 x )
(ii) lim x [( x  1)  ( x  1) ]
1/ 3 2/3 2/3
x 
x 

(ii) lim ( 3 x 3  2 x 2  5  x )  e x  e x 1
x  (iii) lim   x tan 
 x
x 
 e e
x  x

(iii) lim ( 5 x 5  3x 4  17  x ) Evaluate the following limits:


ps
x  9.
lim t ( t a 1) a 0
el
(i)
(iv) lim ( x 2  ax  b  x 2  cx  d ) t 
x 
eh

6. Two citizens are arguing about (ii) lim x n sin a , n  0


x  x
je

 3x 2  2 x 
lim   3x   a 
iit

  x
2 tan  x 
x 
 x5  lim 2 
The first claims, “For large x, 3x is small in comparison (iii) x
@

1
to 3x2, and 5 is small in comparison to x. So (3x2 + 3x)/ x sin  
x
(x + 5) behaves like 3x2/x = 3x. Hence the limit in question 1
is 0.” His companion replies, “ Nonsense. After all, (iv) lim x −2 (1.001) − x

x0
3x 2  2 x 3x  2
 ,
n m
10. Show that lim x (nx ) = 0 , m, n ∈ N
x5 1  (5 / x ) x 0

Practice Problems J

11. Evaluate the following limits : 12. Evaluate the following limits :
 x 3
x  2
 1 2 
(i) lim  2   (i) lim  2  4 
  x 1    1 

x 3 x 2
 3x 4   x 1 x

(ii) lim
x 
 x 2  2x  1  x 2  7 x  3  (ii) lim
x 
 3
x 3  3x 2  x 2  2 x 
 3x (2 x  1)(3x  x  2) 
2 2
(iii) lim     
  

x   2 x
  1 4x 2  (iii) lim   
x 0  4 x
 2 x (e x  1) 

 1 1 

(iv) xlim
2

 x ( x  2)
2 
x  3x  2 
2 lim
(iv) x  3 ( x  a ) ( x  b ) ( x  c)  x 

FREE BOOKS FOR JEE & NEET =>(@iitjeeadv)


Limits 1.71

lim ( 2
x  x  1 + ax – b) = 0, then find the constants then lim  f ( x ) 
g (x)
13. If x  AB .
xa
a & b.
Also lim  f ( x ) 
g (x) lim g ( x ) n ( f ( x ))
14. Evaluate e x a

xa
lim (n6 + 6n5 + 12n4 + 1)1/3 – (n4 + 4n3 + 6n + 1)1/2 Consider the calculation of the limit, as x → a, of the
n
exponential power function [f(x)]g(x), where the functions
 x 1
2 
15. If xlim   ax  b   0 find a and b. f(x) and g(x) are defined in a neighbourhood of the point a,
  x  1 
  and f(x) > 0.
16. For a certain value of B, the limit lim ( x  5x  3)  x
4 3 B
x  The following cases are possible here :
is finite and nonzero. Find B and then compute the limit. 1. If lim f(x) = b > 0, lim g ( x )  c , then lim f ( x )  bc .
g(x)
x →a x a x a
17. Evaluate the following limits :
2. If xlim f ( x )  0 , lim g( x )  c  0 (or  ), then
a x a
  
lim  2 x tan x 
(i)
x
 cos x  lim f ( x )g ( x ) = 0.
x →a

 
2
(ii) lim ln x  ln x 2  1  x  3. lim f ( x )  0 , lim g ( x )  c  0 (or  )
x a x a
x   
g(x)
      then xlim
→a
f (x) = ∞.
(iii) lim n.cos   .sin  4n 
n  4n    lim f ( x )  0, lim g ( x )  0 , t h e n lim f ( x )g ( x ) i s a n
4. x a x a x →a
 
ps
2
(iv) lim n sin  ln cos  indeterminate form of type 00
n  n

el
5. If lim f ( x )  , lim g ( x )  c  0 (or ) , then
x a x a
18. Evaluate the following limits:
eh

lim x3 lim f ( x )g ( x ) = ∞.
x →a
je

(i) x { x 2  1  x 4  x 2}
f ( x )  , lim g ( x )  c  0 (or  ) , then
iit

lim
(ii) x e x ( e x +1 – e x −1 )
6. If xlim
a x a
@

lim x3/2 ( x +1 + x −1 – 2 x )
(iii) x lim f ( x )g ( x ) = 0.
x →a

19. Evaluate the following limits : 7. If xlim f ( x )  , lim g ( x )  0 , then lim f ( x )g ( x ) is an


a x a x →a
(i) lim (sin x  1  sin x ) indeterminate form of type ∞0.
x 

8. If xlim f ( x )  1, lim g ( x )   , then lim f ( x )g ( x ) is an
 x 1 x  a x a x →a
(ii) lim x  tan 1  tan 1
x   x2 x  2  indeterminate form of type 1∞.
 1 1 1 
(iii) xlim x 5 sin     3  Note: If h > 0, then lim (1 – h)n = 0 and
   x  x 6x  n
lim (1 + h)n → ∞.
1.15 Limits of Forms 0 and ∞
n
0 0
1 x
 sin 2x 
Example 1: Find lim 
Calculating the Limits of Exponential Power x 0  x 
Functions Solution: Here,
We first note that  sin 2 x 
lim   2 and xlim
x 
(1 + x) = 1.
x 0  →0
 0, 0  a  1
 1, a 1 1 x
 Hence, lim 
sin 2x 
 21  2 .
lim a x  
x 
 , a 1
x  x 0 

dne a0 x2
 x 1 
If lim f ( x ) = A > 0 and lim g ( x ) = B, a finite quantity Example 2: Find lim  
x→a x   2 x  1 
x→a

FREE BOOKS FOR JEE & NEET =>(@iitjeeadv)


1.72 Differential Calculus for JEE Main and Advanced

Solution: We have n x
1 lim n f ( x )  lim =0
1 x  x  x
x 1 x 1
lim  lim  Therefore,
x  2 x  1 x  1 2
2 lim x1/ x  lim .f ( x )  lim en f ( x )  e0  1 .
x x  x  x 
n 2
and lim x   .
2
x 
Example 6: Evaluate lim x 1 nx .
x 
x2 n 2
Therefore, lim  x  1  0 Solution: lim x 1 nx  0
form 
x   2 x  1  x 
n 2
= lim .nx
x  1 nx
Limits of Form 00 e
n 2
Example 3: Evaluate lim ( x  x ) .
2
2 x lim
x 0  = x  1
1
e nx
(x  x 2 )x
2

Solution: xlim
0  = en2 = 2.
= lim x n ( x  x )
2 2
Example 7: Evaluate x lim (2x + 3x)1/x.

 
x  0
e
x
lim
Solution: x (2x + 3x)1/x  0 form
= lim ( x  x 2 ) n ( x  x 2 )  (using lim x n x = 0)
e 1 x x 0
x0 
We choose the largest term in the base and divide the base
= e0 = 1. with it. Here 3x is the largest term.
1
ps
1/ x
Example 4: Evaluate lim (1  x 2 ) n (1 x )   x 
x 1  Limit = lim  3x   2   1 
el
x    3  
1   
eh

1/ x
Solution: lim (1  x 2 ) n (1 x )  x 
x 1 = lim 3 1   2  
je

x    3  
lim
1
n (1 x 2 ) 
=
iit

x 1 n (1 x ) x
as x → ∞,   → 0 since 0 <
e 2 2
 n (1 x ) n (1 x ) 
< 1.
@

lim    3 3
= x 1
 n (1 x ) n (1 x ) 
e = 3(1 + 0)0 = 3.
 n  
1/ x
= e1 + 0 = e.
Example 8: Evaluate lim
x
 2nx , n ∈ N.
x 
Caution Solution: For n = 1, 2, 3, we have 2 > n . n

1/ x
  x

lim(sin x ) 2 x2  x
=1 Limit = lim 2n   n   1 = 2n.
x    2n  
x 0  
Note that this is not in 00 indeterminate form. The exponent For n ≥ 4, we have 2n < n .
is exactly 0. 1/ x
 n 
x 
lim ( x 2  x )sin
2
x
=0 2
x 0 Hence limit = xlim n    1 = .
   n  
This is not in 00 indeterminate form. The base is exactly 0.  
The limit lim (sin 2 x  cos 2 x  1) x  x cannot be evaluated
2
Example 9: Find
x 0
 
sin 2 x
since the function is undefined as both base and exponent are lim 1cos ec x  2cos ec x  ...  n cos ec x
2 2 2

x→0
exactly 0. Solution: Let y = cosec2x. As x → 0, y → ∞
Limits of Form ∞0 Limit = lim (1y + 2y + ... ny)1/y ( ∞ 0 form)
y
1
1/ y
 1  y  2  y y 
   n 1 
Example 5: Find the lim x x . 1/ y
x   lim n y
      ...     1
Solution: The limit leads to the indeterminate form ∞ 0 . y    
n n  n  
1/ y
1
We let f(x) = x and find xlim n f ( x )  1  y  2  y  n 1 
y 
x  lim n       ...     1
y   n  n  n  
1
n x 
Since n =
f ( x ) =
nxx 0
= n. 1 = n.
x

FREE BOOKS FOR JEE & NEET =>(@iitjeeadv)
Limits 1.73

Concept Problems M


1 x 2x
1. Evaluate lim  1  x  1 x  1  x 1
x→1   4. Evaluate xlim  
 2x    x 2 

2n 
2. Evaluate lim sin n  
5. Evaluate lim (4n + 5n)1/n
n
n   3n  1  1
2
x
3. Evaluate lim 
x2  6. Evaluate xlim (cot x ) ln x
 0 

x   2 x  1 

Practice Problems K

x 1
 x 1  10. Evaluate lim (1  cos ec3 x ) tan x
7. Evaluate lim  2  x 0
x 1  x  1 
 
11. Evaluate
x

( 2 x 1) lim x ( x )
x 0
lim  x  2 x  1   x 1
ps
2
8. Evaluate x
 2 x 2  3x  2  12. Evaluate  (e 
1/ n
lim  n
  )  ( e ) n
el
n
1/ln n 1
  
eh

9. Evaluate lim  cosh  1 13. Evaluate lim (tan x)tan 2x


n   n  x /2
je

x
where coshx = e  e .
x
iit

14. Evaluate lim (1x + 2x + 3x + ...... 99x)1/x


x
2
@

1.16 Limits of Form 1∞ Let xlim f ( x )  1 and lim g ( x )   then


a xa
1 x
 1 g ( x ) f ( x ) 1
lim  f ( x ) 
lim
l im (1  x ) x  l im  1    e ,
g (x)
e x a

x 0 x   x xa

Let  = lim 1  f ( x )  1
1 g(x)
(1  x ) x  e . Proof:
To prove xlim
0
xa
f ( x ) 1
The limit is in the indeterminate form 1∞.  g (x) 
1 = lim 1   f ( x )  1 f ( x ) 1 
We let f(x) = (1 + x ) x and find x
Lim n f ( x ) .
0 
xa 
 
1 1 Put f (x) – 1 = y, as x → a, y → 0
Since n f ( x ) = n (1 + x ) x = n (1 + x ) lim g ( x ) [ f ( x ) 1]
 1 x a

 =  lim 1  y  y 
x ∴
n (1  x )  0   y 0 
lim n f ( x )  lim 0 = 1
x 0 x 0 x   = e
lim
x a

g ( x ) f ( x ) 1


1
This is an important formula for solving 1∞ form.
lim (1  x) x  lim f ( x )  lim en f ( x )  e .
x 0 x 0 x 0
Example 1: Calculate lim (1 + sin 4x)cosx.
Generally, it is useful to remember that x0
x ax Solution: First notice that as x → 0+, we have
 a  1
lim 1 +  = lim (1 + x ) = lim 1 +  1 + sin 4x → 1 and cot x → ∞, so the given limit is
a/x
= ea .
x →∞  x x →0 x →∞  x indeterminate.

FREE BOOKS FOR JEE & NEET =>(@iitjeeadv)


1.74 Differential Calculus for JEE Main and Advanced

Let y = (1 + sin 4x)cosx tan


x

Then n y = n[1 + sin 4x)cotx] = cot x n (1 + sin 4x) Example 4: Evaluate xlim  a 2a .
→a  2  
 x
lim n y = lim n (1 + sin 4x )
x0 
x0  x
tan x tan
 a 2a
Solution: xlim
→a  2  
= lim n (1 + sin 4 x ) · sin 4 x · 4 x  x
Put x = a + h

x0 sin 4 x 4 x tan x
So far we have computed the limit of n y, but what we want   h 
tan   
is the limit of y. To find this we use the fact that y = en y lim 
= h→0 h   2 2a 
1  
lim (1 + sin 4x)cosx = lim en y = e4  (a  h ) 
x0 x0
 h 
 cot  
x lim  h   2a 
 x 2  5x  4  = h→0
1  a  h 
Example 2: Evaluate lim  2   
x   x  3x  7 
   cot
h 
. 1
h 
 1
= lim
2 a  a  h
Solution: By means of division of the numerator of e
h 0

the fraction by the denominator, we isolate the integral part:  h  2a
  
= lim   2a . = e–2/π .
x  5x  4
2
8x  3 h 0
 tan h  a  h
1  2 . e  2a 
x 2  3x  7 x  3x  7  a x  bx  cx 
2/ x

Thus, when x → ∞ the given function is a power whose base Example 5: Evaluate lim 
x 0  
tends to unity and the exponent, to infinity (an indeterminacy where a, b, c > 0.  3 
ps
x x 2/ x
 x 2  5x  4   8x  3  a b c 
x x x

  (form 1∞ )

Solution: We have lim 

of the form 1 ). lim  2  = lim 1
el
x   x  3x  7  x   x 0 
 x  3x  7 
2
   3 
eh

x (8x 3)
 a x  bx  cx  2
 x2  3 x  7
 x2  3 x  7
=  1.
 8x  3 
8x 3 lim
x 0  x
je

3
 lim 1  2  e

x   x  3x  7  
iit

2  a x  b x  c x 3 

lim  
 =
e 3 x 0  x 
@

8  3/ x

 x2  3 x  7
 1  3/ x  7 / x 2 2  a x 1 b x 1 c x 1 
 
 8x  3  lim 
8x 3
=
 lim 1  2  3 x 0  x x x 
  e
x  
 x  3x  7   2  a x 1 b x 1 c x 1 
  = lim  lim  lim 
3  x 0 x x 0
x x 0
x 
e
8x  3
Since  0 as x → ∞ and we have = 3(2/3) ln (abc) = eln( abc ) = (abc)2/3.
2/3

x 2  3x  7
x2  3 x  7
Example 6: Find the value of
 8x  3 
8x 3

lim  1  2  e . 1
x   x  3x  7      ln x
 lim  tan   ln x  .
8  3/ x x 1  4 
Taking into account that xlim 8,
 1  3 / x  7 / x2 1
x     ln x
 x 2  5x  4  Solution: lim  tan   ln x   (1∞ form)
we get lim  2  e
8
x 1  4 
x  x  3x  7 
  1

Example 3: lim (tan x)tan 2x = lim 1  2 tan(ln x )  ln x


 
x
4
x 1  1  tan(ln x ) 
lim (tan x 1) tan 2 x
Solution: Limit = x (using formula) 2 tan(ln x ) 1
e 4
= x 1 {1 tan(ln x )}
lim
.
ln x
2 tan x e
= lim(tan x 1)
x

1 tan 2 x tan(ln x ) 1
e 4
= 2 lim ln x
.
1 tan(ln x )
tan  / 4 1 e x 1

= 2 = e–1 =
1(1 tan  / 4 )
e e = e2 . (1) = e2

FREE BOOKS FOR JEE & NEET =>(@iitjeeadv)


Limits 1.75

 a 1 n b 
n
l n ({sin x}·{cos x} + 1)
Example 7: Find the value of lim  f(x) = where {x} denotes fractional

n   a {sin x}{cos x}
(a , b > 0) .  
part function, prove that

 a 1 n b 
n
     
(ii) f (0–) = f 
 2 
(i) f 
 2 
Solution: lim   (1∞ form) = f (0+)
n 
 a     
n lim
 n b 1  and show that f(x) does not exist.
= lim 1  
x→0
n   a  ln(sin h  cos h  1)

Solution: f (0+) = lim
lim  b 1 
n h0 sin h  cos h
= e n 
  . n ln(sin h ·cos h  1)
 a  lim
= h0 sin h
lim 1  b1/ n  1  · h cos h
= e n 
  h
a  1 / n 
= lim ln(sin h ·cos h  1)
1h
h 0
1
= log e b = b1/a.
ea =e log e b1/ a
1
= lim (sin h ·cos h  1  1) = 1.
n (1  x  x 2 4
) h0h
Example 8: Evaluate xlim   
→0
(e x
 1)x ln(cos h ·sin h  1)
f 
 2 
= lim = f (0+).
ps
h0 cos h ·sin h
 n (1  x 2  x 4 )  
f (0 ) = lim ln  (1  sin h ) cos h  1 = ln 2
el
lim
Solution: x→0  e 1  x –

(1  sin h ) cos h
eh

2
x   h0
 x 
⇒ lim f ( x ) does not exist.
je

 
1
= xlim
→0
l n 1  x 2  x 4 x2 x→0
iit

  
As before f 
 2 
is same as f (0–)
(1  x )
@

1
= l n xlim  x 1
2 4

→0 e x2   n
 3 n 
= xlim x +x 2 4
= xlim (1 + x2) = 1
Example 11: Evaluate L = lim  3
n  n
 r  2

→0 →0  r 1 
x2 n
3
Example 9: Evaluate Solution: We first note that lim
n  n 3
 r2
r 1
   3 n (n  1)(2n  1)
ln  tan   ax   = lim . = 1.
  4  . n  n 3 6
lim ( b  0)
x0 sin bx Hence the limit L is in 1∞ form
lim  ( n  1)( 2n  1) 
   ∴ L= e  n 
 1 n
ln  tan   ax    2n 2 
 4 
Solution: lim
= elim (2n  3n  1  2n )
2 2
x0 sin bx n 
bx
bx 2n
1 3n 1 3
= lim = .
= lim ln  tan    ax   e2
n 
bx e 2n
 4 
x 0   
Example 12: Evaluate

1       
= lim  tan   ax   tan  sec 2  
x0 bx  4  4 lim     2  2  bx 
x→0 sin  
ax tan ax     2a   2  ax   .
lim
= x . 1  tan   ax  .tan   sec 2 
bx ax   4  4  b  
lim sin 2 
0

2  bx

Solution: L = x→0 
Example 10: For a function   2  a x  

FREE BOOKS FOR JEE & NEET =>(@iitjeeadv)


1.76 Differential Calculus for JEE Main and Advanced

 cos 2  So, the term of least degree in f(x) is of degree 2.


=  lim sec 2
2  bx 2  ax = e −2 ,
e x 0
i.e., f(x) = a2x2 + a3x3 + ......
cos 
where  = x→0
lim 2a x  x 2  f (x) 
1/ x

cos  Now, lim 1   = e2


2bx x 0  x2
 
sin     
lim 2 2  a x   x2 f (x )  1 x2 f (x )
= x→0 ⇒ x 0 
lim  = e2 ⇒ lim = e2
sin     e  x x
x 0

2
e x3
2 2  b x 
   ∴ x 2  f (x) = 2
lim 2 2  a x = lim − a x 2 − b x
= x→0
lim
x 0 x3
  x→0 2 − a x − bx
2 2  bx
− a2
x 2  a 2 x 2  a 3 x 3  .....
= a. ∴L = e b2
⇒ lim =2
b
x 0 x3
Example 13: Find a polynomial of the least degree, ⇒ a2 = –1, a3 = 2 and a4, a5 ....... are any arbitrary constants.
Since, we want a polynomial of the least degree, we choose
1/ x
 x 2  f (x) 
lim 1  
such that x 0  x2  = e2. f(x) = – x2 + 2x3.
1/ x
 x 2  f (x) 
Solution: lim 1   exists only when it is
x 0
 x2 
ps
x 2  f (x)
in 1∞ form i.e. lim = 0.
el
x 0 x2
eh
je

Concept Problems N
iit

@

1. Show that
(iii) xlim
→0
(ax + x)1/x
1/ 2 if x  0
1 x  1
lim  1  x  1 x   2 / 3 if x 1 lim  1  tan x  sin x
 2x  (iv) x→0  
    1  sin x 
1 if x 
4. Evaluate the following limits :
2. Evaluate the following limits : x4
(i) lim  x6
 x 
x
x  x 1 
(i) lim    
x  1 x  3
x 1 (ii) lim x 4  ln x

(ii) lim  3x  4  3 x 1
 
x   3x  2 
x
(iii) lim 1 
1 
 ( n  0)
x
 1 1
(iii) lim  sin  cos  x   xn 
x   x x
1/( e x
1 x )
  (iv) lim(cos x )
tan 2 x
(iv) lim 1  3 cot 2 x x 0
x 2

3. Evaluate the following limits :


(i) lim (1 + sin πx)cot πx
x→1

(ii) lim (2  cos )cos ec 


2

 0

FREE BOOKS FOR JEE & NEET =>(@iitjeeadv)


Limits 1.77

Practice Problems L


5. Evaluate the following limits: 1/ x
(i) lim  x  1  cos x 
n x→0  
(i) lim  cos m x  x2 , m, n ∈ N  x 
x0 1/ln( x 1)
(ii) lim (1  ln(1  x ))
2

x2 x 0
(ii) lim  x sin 1  1   1
x   x   (iii) xlim  xn  an  x a
→a 
 ( x  a )na n 1 
x  
(iii) lim cos n x
n  n lim  e   1  x  
(iv) x   
 
nx  1  e   e 1  x  
(iv) lim  1
1x
 21 x  31 x  ..........  n1 x
 Evaluate the following limits :
x   n 9.
 
 e/ n  e / n 
6. Evaluate (i) lim  1
n
n 2 

1/ x n
 1 a x 1   n  1
lim  .  where a > 0, a ≠ 1. (ii) n 
lim  sin 
x   x a  1  
   n  1  n 
ps
7. Evaluate the following limits:
lim {cos(n(n–1) – n(n + 1))} ( n+1)
(iii) n 2
el
1/ x
 tan x   a 
(i) lim  lim tan n  π − 4 + 1 − 1  
x 
eh

x  0 (iv) n  
 4 n 
je

2
cot x
(ii) lim (cos x ) 2x
x →0 lim 1  l  m 
10. Find λ & µ if x  e2 .
iit

 2 
x  x 
cos ec x
(iii) lim  1  tan x 
@

11. For A and B positive constants, define


x  0  1  sin x  f(x) = (ex + Ax)B/x.
1/ x 2 (a) Compute L1 = xlim f ( x ) and L = lim f ( x )
 1  →0 2 x 
(iv) lim  tan x 
x 0   (b) What is the largest value of A for which the equation
 x  L1 = B L2 has a solution ? What are L1 and L2 in this
8. Evaluate the following limits: case ?

1.17 Sandwich Theorem/Squeeze Play


Theorem
Suppose that g(x) ≤ f(x) ≤ h(x) for all x in some open interval
containing a, except possibly at x = a itself. If

lim g(x) = lim h(c) =  then lim f(x) = .
x →a
x →a x →a

Note: x2 x2
For example, if 1 – ≤ f(x) ≤ 1 + for all x ≠ 0,
(i) This is true also when one or both of the above inequalities 4 2
are strict. we can find xlim f(x), even if we donot have the
→0
(ii) This theorem is also applicable to one sided limits and
limits at infinity. formula of f(x). We can see in the figure that the graph of f(x)
(iii) The theorem holds even if  is finite or infinite. is squeezed between the graphs of the other two functions.

FREE BOOKS FOR JEE & NEET =>(@iitjeeadv)


1.78 Differential Calculus for JEE Main and Advanced

1
Let f(x) = x cos3 . As x → 0, f(x) → 0.
x
1
Also |f(x)| = x cos3 approaches zero because
x
1
– |x| ≤ x cos3 ≤ |x| and
x
– |x| and |x| approach 0 as x → 0.
Since xlim 2 lim 2
→0 (1 – (x /4)) = 1 and x →0 (1 + (x /2)) = 1, 1
Example 1: Show that lim x 2 sin  0.
x 0 x
Sandwich Theorem implies that xlim →0
f(x) = 1. Solution: First notice that we cannot use
sin x
We shall now prove the standard limit xlim =1 1 x 2 · lim sin 1
→0
x lim x 2 sin = xlim
→0
x →0 x x →0 x
geometrically, with the help of Sandwich Theorem.
We take a circle of unit radius and suppose that the central because limx → 0 sin (1/x) does not exist.
1
angle x expressed in radians is contained within the limits 0 However, since –1 ≤ sin ≤ 1.
x
and π/2. It will be enough to consider only x > 0, since
sin x is an even function. We have, as illustrated by the figure,
1
x – x2 ≤ x2 sin ≤ x2
Proof: We shall compare the areas of three regions in the x
figure : two triangles and a sector.
ps
el
eh
je

We know that
iit

On inspecting the figure we find that, area of ∆OAP <


@

lim x 2  0 and lim( − x 2 ) = 0


area of sector OAP < area of ∆OAT x 0 x →0
sin x x tan x Taking g(x) = – x2, f(x) = x2 sin (1/x), and h(x) = x2 in the
< < ⇒ sin x < x < tan x
2 2 2 Sandwich theorem, we obtain
1
1<
x
<
1 lim x 2 sin  0.
⇒ (0 < x < π/2) x 0 x
sin x cos x
Example 2: Determine the limit x lim sin x .
sin x x
⇒ cos x < <1
x Solution: Since – 1 ≤ sin x ≤ 1, it follows that
Since, xlim cos x  1 , and lim 1 = 1

– 1 ≤ sin x ≤ 1 , x > 0,
0 x0 x x x
Using Sandwich Theorem we have lim sin x = 1 where x
lim (– 1/x) = 0 and lim (1/x) = 0. Therefore, by
x 0 x
x sin x
sin y Sandwich Theorem, we can obtain lim
Put x = – y, then lim = 1. x
= 0.
y 0 y
x
Hence, lim sin x = 1.
x→0 x
Zero Limit Theorem
A function f approaches the limit 0 as x tends to a if and only
if |f | approaches the limit 0 as x tends to a.
lim |f(x)| = 0 ⇔ lim f(x) = 0.
x →a x →a

Proof: Since –|f(x)| ≤ f(x) ≤ |f(x)|, and –|f(x)| and |f(x)| both
have limit 0 as x approaches a, xlim →a
f(x) = 0 by Sandwich
Theorem.

FREE BOOKS FOR JEE & NEET =>(@iitjeeadv)


Limits 1.79

Definition of e
Example 3: Evaluate l im [ x ] .
x  x  1
n

Solution: We have x – 1 < [x] ≤ x, The number e is defined as the limit lim 1   .
n   n
⇒ 1  1  [ x ]  1 for x > 0. The number e is irrational and therefore cannot be expressed
x x precisely by a finite decimal. Its approximate value is e ≈
 1 2.718
Now lim  1    1. Therefore by Sandwich theorem x
x   x  1
It can be shown that the function y  1   has
l im [x]
=1 .  x
x  x a limit not only when its argument runs through natural numbers
(that is x = n where n = 1, 2, 3, ....) but also when it varies
Example 4: Let a function f(x) be such that | f(x) | ≤ M continuously and approaches – ∞ or ∞. In all the cases the
for any x ≠ 0. Prove that lim x f(x) = 0. limit is the same number e.
x→0
Solution: We have | x f(x) | ≤ | x | . M Proof: Let n ≤ x < n + 1
⇒ – M | x | ≤ x f(x) ≤ M | x | for any x ≠ 0. ⇒ 1+
1
<1+
1
≤1+
1
lim M | x | = 0 and lim – M | x | = 0,
Since x→0 n +1 x n
x→0 n x n 1
 1   1  1
by Sandwich theorem, x→0
lim x f(x) = 0. ⇒ 1    1   ≤ 1  
 n 1   x  n
x  7 sin x n 1 −1
Example 5: Evaluate lim using ⇒  1   1 
< 1  x  x
1
x  2 x  13 1  n  1   1 + 
ps
Sandwich Theorem.   n + 1
el
n
Solution: We have –1 ≤ sin x ≤ 1 for all x.  1  1
≤ 1   1  
eh

⇒ –7 ≤ 7 sinx ≤ 7    

n n
⇒ x – 7 ≤ x + 7 sinx ≤ x + 7 Now as x → ∞, n also tends to ∞ and in such a case both the
je

Dividing throughout by –2x + 13 (a negative quantity for right hand and left hand side of the above inequality tend to e.
iit

large values of x), we get Thus by Sandwich theorem.


@

x 7 x7 x7  1
x
  . lim  1   = e.
2 x  13 2 x  13 2 x  13 x   x
We evaluate the limits of the extreme functions as x tends to
infinity. Since both the limits are equal to –1/2, by Sandwich Example 8: Evaluate
theorem the limit of the desired function is also –1/2.  n n n n 
lim   2  2  ....  2 
n  n 2  1 n 2 n 3 n n
x 2 (2  sin 2 x )
Example 6: Evaluate lim
x  x  10 Solution: Let f (n)
2x 2 x 2 (2  sin 2 x ) 3x 2 n n n n
Solution:   = 2 + 2 + 2 + .... + 2
x  10 x  10 x  10 n +1 n + 2 n + 3 n +n
2x 2 3x 2 Note that f(n) has n terms which are decreasing Suppose h (n)
lim   and lim 
x  x  10 x  x  10
=  n  n  n  ....  n 
x 2 (2  sin 2 x )  n2 1 n2 1 n2 1 n2 1 
Hence, by Sandwich Theorem lim .
x  x  10 n2
h (n) = (obviously f (n) < h (n) ) and g (n)
1  2 x 2 , x Q n2 +1
Example 7: Find x→0 lim f(x) if f(x) = 
= lim 
 1  x
4
, xQ n n n n 
   ....  
Solution: Consider | x | ≤ 1. n  n2  n n2  n n2  n n2  n 
x4 ≤ x2 ≤ 2x2.
n2

Now 1 ≤ f(x) ≤ 1 + 2x2 ∀ x. = (obviously g (n) < f (n) )
lim 1 + 2x2 = 1. By Sandwich theorem, lim f(x) = 1 n2 + n
x→1 x→0 Hence g (n) < f (n) < h (n)

FREE BOOKS FOR JEE & NEET =>(@iitjeeadv)


1.80 Differential Calculus for JEE Main and Advanced

lim g(n) = 1 = lim h (n) Thus, we have


Since n n n
1 1.3.5....(2n  1)  n2  2
Hence using Sandwich Theorem n
lim f (n) = 1.    n 1 
2 
2.4.8...2n 2.4.8....2n  
n
Example 9: Let xn = 
1 lim x .
. Calculate n Now, we have
k 1 n2  k n n
Solution: We have 1  n2  2
lim  0 and lim   n 1 
n
1 n
1 n
1 n  2.4.8.....2 n n  2 
∑ 2
<∑
2
<∑ Hence, by Sandwich theorem, we have
k =1 n +n k =1 n +k k =1 n2 1.3.5....(2n  1)
lim 0
n n  2.4.8....2n
Thus < xn < 1
n2 + n [ x ] + [2 x ] + [3x ] + .... + [nx ]
Example 11: Find nlim 
n 1 n2
Since nlim  lim  1, where [ . ] denotes the greatest integer function.
 n 
n n2
1
1
n Solution: We know that, x – 1 < |x| ≤ x
we have lim x n  1 in accordance with Sandwich theorem. ⇒ 2x – 1 < [2x] ≤ 2x
n  ⇒ 3x – 1 < [3x] ≤ 3x
....................
Caution ....................
⇒ nx – 1 < [nx] ≤ nx
ps
On the other hand, the general term in the expression for xn ∴ (x + 2x + 3x + .... + nx) – n
el
1 < [x] + [2x] + ..... +[nx] ≤ (x + 2x + .... + nx)
is equal to (k = 1, 2,...., n).
eh

n +k
2
n
xn (n +1) x.n (n +1)
1 ⇒ –n<  [r x ] ≤
je

Since lim  0 , if follows that 2 r 1 2


n 
n k
2
iit

⇒ lim x  1  1  – 1 ≤ lim [ x ] + [2 x ] + .... + [nx ]


  n  
@

1 1 1 2  n  n n n2
lim    ......  
n   
 n 1 n 2 n n lim x  1  1 
2 2 2
 ≤ n  
1 1 2  n
= lim  lim +.....
n 
n 1
2 n 
n 2
2

x lim [ x ] + [2 x ] + .... + [nx ] ≤ x
≤ n
1 2 n2 2
+ nlim = 0 + 0 + ... + 0 = 0

n2  n lim [ x ] + [2 x ] + .... + [nx ] = x

Thus, n
The latter method has error in the reasoning. When we add n2 2
infinite number of small quantities, the sum may not always Example 12: Evaluate
tend to zero.  [12 x x ]  [22 x x ]  .....  [n 2 x x ] 
Example 10: Using Sandwich Theorem, show that lim  lim 
x 0  n 

n3
 
1.3.5....(2n  1)
lim  0. where [ ] denotes the greatest integer function.
n  2.4.8......2 n

Solution: We can see that Solution: We have 12xx – 1 < [12xx] ≤ 12xx
1.3.5....(2n − 1) 1 22xx – 1 < [22xx] ≤ 22xx
> ............
2.4.8......2n 2.4.8.....2n

............

n
 n (1  2n  1)  n2xx – 1 < [n2xx] ≤ n2xx
1.3.5....(2n  1)  2 n 
 Adding the above inequations,

2.4.8....2n 21 2 .....n x x n 2  n [n 2 x x ] x x n 2
Using AM. – GM. inequality in the numerator.  
n3 n3 n3
1.3.5....(2n  1) n n
⇒  n (n  1)(2n  1) 1  2 x x ] n (n  1)(2n  1)
2.4.8....2 n n ( n 1) xx 3
 2
  xx
2 2 6n n n3 6n 3

FREE BOOKS FOR JEE & NEET =>(@iitjeeadv)


Limits 1.81

Now, as n tends to infinity, we have  [n 2 x x ]  1


  lim x
x
xx [n 2 x x ] x x lim  lim
 lim  x 0  n 
 3
3  

3 n  n 3 3  n  x 0

ln x
Hence, by Sandwich Theorem, we have =
1
lim e 1/ x (since 1/x is much larger than lnx)
[n 2 x x ] x x 3 x 0 

lim 
n  n3 3 = 1 e0 = 1 .
Now, the required limit 3 3

Concept Problems O

1. If 5 − 2x 2 < f(x) < 5 − x2 for x ∈ [–1, 1], find g(x) ≤
1
sin (50π 3 x ) ≤ h(x) for all values of
x2 1
lim (x)
x→0 f x except zero. What can you say about lim sin
x  x2
2. It can be shown that the inequalities
(50π 3
x) ?
x2 x sin x
1   1 hold for all values of x close
6 2  2 cos x 6. If b – |x – a| ≤ f(x) ≤ b + |x – a| in the neighbourhood of
x sin x x = a then find lim f(x).
to zero. What does this tell you about xlim  0 2  2 cos x
? x →a
ps
3. If x4 ≤ f(x) ≤ x2 for x in [–1, 1] and x2 ≤ f(x) ≤ x4 for  x if x is rational
 2
Let f ( x )  
el
7.
x < –1 and x > 1, at what point c do you automatically  0 if x is irrational

eh

know xlim→ c f(x) ? What can you say about the value of the
limit at these points? Use Sandwich theorem to prove that xlim f (x)  0 .
je

4. By considering the inequality 0


iit

x2 x2 x4 1 1
1  cos x  1   x 2 cos x 2 sin
@

2! 2! 4! 8. Show that lim x = lim x


sin x
lim 1  cos x  0 . x→0 sin x x→0
prove that x→0
x
lim f(x) if 4 x  1  f ( x )  4 x  3x for all x > 5.
2
5. Find formulae for function g and h such that 9. Find x
g(x) → 0 and h → 0 as x → ∞ and such that x x2

Practice Problems M

10. Evaluate the following limits: 2 1 − cos x 1
1
11. Show that – x < < for all x close to
 2 2 24 x 2
2
lim x2  cos 2 x  x 2 cos 
(i) x→0  x  lim 1 − cos x .
x = 0 and hence find x→0
x2
12. Prove that
(ii) lim n cos(e n )
n  2n  1 2x  3 2 x 2  5x  3
lim  lim
x  x 3
x x  1/ 2 4 x 2 18x 10
2e x + e − x − 3x
lim
(iii) x where [.] is G.I.F. x
[e x + x 2 ] sin 2
 lim 2
1 x 0 x2
xsin
(iv) x
lim
x0 sinx
13. Evaluate the following limits:

FREE BOOKS FOR JEE & NEET =>(@iitjeeadv)


1.82 Differential Calculus for JEE Main and Advanced

derivatives, provided that the given conditions are


(i) lim x + sin x
x
x + cos x satisfied. In other words, suppose that lim
f '( x )
= A, then
x sin x ! x →a g '( x )
(ii) lim f (x)
x 1  x 2
lim
x →a
= A where A may be a number, or ∞ or – ∞.
(iii) lim x 3  x 2 sin 
g( x )
x→0 x It is especially important to verify the conditions regarding
1 the limits of f and g before using L'Hospital's Rule.
(iv) lim x cos
x0

e x L'Hospital's rule is also valid for one sided limits and for limits
at infinity or negative infinity : that is, "x → a" can be replaced
lim [1 x ] + [2 x ] + [3 x ] + .... + [n x ] where
2 2 2 2
14. Evaluate n by any of the symbols x → a+, x → a, x → ∞ or x → – ∞.
n3
[.] denotes the greatest integer function and x ∈ R Special Case Proof
For the special case in which f(a) = g(a) = 0, f' and g' are
15. Let the inequality g(x) < f(x) < 1 – cos x + g(x) hold for
continuous, and g'(a) ≠ 0, it is easy to see why L'Hospital's
g( x ) rule is true. It is more difficult to prove the general version
all x in the neighborhood of x = 0. If lim  1, find
x 0 x of L'Hospital's rule.
f (x)
lim .
x →0 sin x
x b b If f and g are differentiable and f(a) = g(a) = 0 and g'(a)
16. If a and b are positive, then prove that lim    ,
x 0 a  x 

a
b x f ( x ) f '(a )

lim    0 . How do the functions behave when ≠ 0, then lim
x a
.
ps
x 0 x  a 
 g ( x ) g '(a )
x → 0– ? Proof : Using the definition of the derivative we have
el

x3 , x0
 f (a + h ) − f (a )
eh

17. Find lim f(x) where f(x) =  1 lim


x→0
 x sin x , x  0 f '(a ) h →0 h f (a  h )
 = lim
je

=
lim ( x 2  x )  1  where [.] denotes the greatest g '(a ) g (a + h ) − g (a ) h  0 g (a  h )
18. Evaluate x→0 x lim
iit

  h →0 h
integer function.
@

since f(a) = g(a) = 0


( n 1) 2
1 f (x)
19. Use Sandwich theorem to evaluate the limit lim
n 
 k
. = lim
x →a g ( x )
.
k n 2

Notice that we have assumed g'(a) ≠ 0.


1.18 L’ Hospital’s Rule
Comprehensive Proof of L’Hospital’s Rule
Suppose f and g are differentiable on an open interval
containing a, except possibly at a itself. Assume that g′(x) ≠ 0 We first present L’Hospital’s Rule as a theorem about one-
for all x in the open interval containing a, except possibly sided limits.
at a. Suppose that L’Hospital’s Rule I. Let f and g be functions which are
lim f(x) = 0 and lim g(x) = 0
x →a x →a
differentiable on a non-empty open interval (a, b) with g′(x)
≠ 0 for every x in (a, b). If , in addition,
or that lim f(x) = ± ∞ and xlim
→a
g(x) = ± ∞ lim f(x) = lim g(x) = 0,
x →a (i) x a  x a 

0
(In other words, we have an indeterminate form of type
0 f ( x ) f (x)
∞ (ii) lim = L, then lim = L.
or ) x a  g( x ) x a  g(x )

f (x) f '( x ) Proof:
Then lim = xlim , provided the limit on the right
x →a g ( x ) →a g '( x ) We may assume that f(a) = g(a) = 0. (If this is not the case to
side exists or is ∞ or – ∞ begin with, we simply define, or redefine, the values of f and
g to be zero at a.) Thus we ensure that f and g are continuous
Note: In many texts, the name L'Hospital is spelt as
on [a, x]. Let x be an arbitrary number in (a, b). Then f and g
.
are continuous on [a, x] (recall that differentiability at a point
L'Hospital's rule says that the limit of a quotient of implies continuity) and are differentiable on (a, x). Moreover,
function is equal to the limit of the quotient of their the derivative g′ does not take on the value zero in (a, x).

FREE BOOKS FOR JEE & NEET =>(@iitjeeadv)


Limits 1.83

Moreover, the derivative g′ does not take on the value zero in The hypotheses have been taken as weak as possible. If, as
(a, x). Hence, by the Generalized Mean Value Theorem and frequently happens, the functions f and g are also continuous
the fact that f(a) = g(a) = 0, we obtain at a, then (i) can be replaced by the simpler condition f(a)
f ( y) f ( x )  0 f ( x ) = g(a) = 0.
  ,
g( y) g ( x )  0 g ( x ) x1/ 2  a1/ 2
Example 2: Evaluate lim , where a > 0.
x a x1/ 3  a1/ 3
for some number y in (a, x). As x approaches a from the right,
so also does y, and hence Solution: If f(x) = x1/2 – a1/2 and if g(x) = x1/3 – a1/3, then
f (x) f ( y) 1
= lim 1
lim
x a g ( x )

x  a g ( y)

= L. This completes the proof. the derivatives are given by f′(x) = and g′(x) = 2/3
,
1/ 2 3 x
2x
Generalized Mean Value Theorem and it is clear that f and g are differentiable (and hence
continuous) and g′ is not zero on an open interval containing
Assume that a < b, and let f and g be functions which are a. Moreover f(a) = g(a) = 0. Hence, by L’Hospital’s Rule,
continuous on the closed interval [a, b] and differentiable on
the open interval (a, b). If g′(x) ≠ 0 for every x in (a, b) then 1
there exists a real number c in (a, b) such that x1/ 2  a1/ 2 2 x1/ 2 3a 2 / 3 3a1/ 6
lim = lim 1  1/ 2  .
f (b)  f (a ) f (c) x a x1/ 3  a1/ 3 x a 3a 2
 . 2/3
g (b)  g (a ) g(c) 3x

Example 1: Compute lim x  2 . Caution


x 2 x2 
ps
Solution: Let f(x) = x − 2 and g(x) = x − 2 . While applying L'Hospital's rule we are not supposed to
el
Obviously, lim f(x) = x
lim g(x) = 0, and since

differentiate f(x)/φ(x) as a fraction. The numerator and
x 2 2
denominator must be differentiated separately.
eh

1 1
f′(x) = and g′(x) = , x a
je

2 x 2 x−2 Example 3: Compute lim where a ≠ 0.


f and g are differentiable, and g′ does not take on the value zero
x a x2  a2
iit

on any open interval with left endpoint equal to 2. We obtain Solution: The fact that
@

1
f ( x ) x−a xa 1
lim = lim 2 x = = if x ≠ a,
x  2 g( x )

x 2 1 
x −a 2 (
2 x  a )( x  a ) x +a
2 x2 immediately implies that
x2 0 x−a 1 1
= lim  = 0. lim = xlim = .
x 2 x →a →a
x2 − a2 x+a


x 2 2a
x 2
And it follows by L’Hospital’s Rule that lim = 0. Of course, the same answer is obtained by L’Hospital’s Rule.
x2 x  2
If we let f(x) = x – a and g(x) = x2 – a2, then f(a) = g(a) = 0
It is a simple matter to verify that L’Hospital’s Rule I remains and f′(x) = 1 and g′(x) = 2x. Hence
true if (a, b) is replaced throughout by (b, a), and lim is x a  lim x − a = lim 1 = 1 .
lim x →a x →a
replaced throughout by x a . This fact, significant in itself,
 x2 − a2 2x 2a
also implies the following two-sided form of L’Hospital’s Rule. It is important to realize that L’Hospital’s Rule II can be
L’Hospital’s Rule II. Consider an open interval containing the applied only if lim f(x) = lim g(x) = 0. For example, if f(x) =
x →a
x →a
number a, and let f and g be functions differentiable and with x2 + 3x – 10 and g(x) = 3x, then
g′(x) ≠ 0 at every point of the interval except possibly at a. If
f (x) x 2  3x  10 0
(i) lim f(x) = lim g(x) = 0, lim = lim  = 0,
x →a x →a x →2 g ( x ) x 2 3x 6
but
f ( x )
(ii) lim = L, then f ( x ) 3x  3 7
x a g( x ) lim = lim  .
g( x )
x 2 x 2 3 3
lim
f (x)
= L. x  tan 2 x
x →a Example 4: Evaluate xlim

g( x )  0 x  tan 2 x

FREE BOOKS FOR JEE & NEET =>(@iitjeeadv)


1.84 Differential Calculus for JEE Main and Advanced

Solution: Since we have the indeterminate form 0/0, we Rule: If the limit of f(x)/ g(x) as x → a takes the form
apply L'Hospital's rule to give: 0/0, differentiate the numerator and denominator separately
w.r.t. x and obtain a new function f'(x)/g'(x). Now as x → a,
x  tan 2 x 1  2 sec 2 2 x 1  2 if it again takes the form 0/0, differentiate the numerator and
lim  lim   3.
x  0 x  tan 2 x x  0 1  2 sec 2 2 x 1 2 denominator again w.r.t. x and repeat the above process, till
indeterminate form persists.
n (1  2h )  2h
Example 5: Evaluate lim The following examples illustrate the repeated use of
h 0 h2 L'Hospital's rule.
n (1  2h )  2h  0
Example 8: Evaluate lim 3x  x  2 .
1/ 3
Solution: lim  form 0 
h 0h 2   2 x 1 3x  6x  3
2
2 Solution: Let f(x) = 3x1/3 – x – 2 and g(x) = 3x2 – 6x + 3.
= lim
1  2h [by L'Hospital's rule] Then f(1) = g(1) = 0, and the derivatives are given by f′(x) =
h 0 2h x–2/3 – 1 and g′(x) = 6x – 6. However, the value of
2h f ( x ) 2 / 3
1
= lim 1  (1  2h ) = lim  2 . lim
x 1 g( x )
= lim x
h 0 h h 0 h x 1 6 x  6

1  cos x is not obvious because f′(1) = g′(1) = 0. Taking derivatives


Example 6: Evaluate lim 2
x 0 x4 again, we get f′′(x) = – x–5/3 and g′′(x) = 6, and it follows that
3
1  cos x 0 2
Solution: xlim 0 f ( x )  x 5 / 2
ps
0 x4   lim = 1
x 1 g( x ) lim 3  .
sin x 1
el
x 1 6 9
= lim 0 [by L'Hospital's rule]
x →0 4x 3   Thus two applications of L’Hospital’s Rule yield
eh

=∞
f ( x ) f ( x )
je

f (x) 1
Note: I f t h e h y p o t h e s e s o f L’ H o s p i t a l ’s R u l e lim = lim = lim =– .
x →1 g ( x ) x 1 g( x ) x 1 g( x )
iit

9
II are satisfied for the functions f′ and g′, that is for
the derivatives of f and g, respectively, then we can
@

Example 9: Evaluate lim e  e  t .


2t t

f ( x ) f ( x ) t 0 t2
conclude that lim = lim . Solution: Recognizing the indeterminate form 0/0 we
x a g( x ) x  a g( x )
apply L'Hospital's rule to yield
This fact suggests the possibility of applying L’Hospital’s Rule
more than once, and in some problems it is necessary to take e2 t  et  t 2e 2 t  e t  1
second or higher derivatives to find the limit. lim  lim
t 0 t2 t 0 2t
sin x  x
Example 7: Evaluate lim 2e 2 t  e t 3
x 0 x3  lim 
t 0 2 2
sin x  x 0
Solution: lim 0 in which we were obliged to use (and justified in using) the
x 0 x3  
quotient of the second derivatives, since the quotient of the
cos x  1  0  first derivatives turned out to be in the indeterminate form 0/0.
= lim  0  [by L'Hospital's rule]
x  0 3x 2   In point of logic one should understand that we properly
 sin x 1 applied L'Hospital's rule to the quotient of derivatives first and
= lim  [by L'Hospital's rule] thereupon applied it again to the given quotient function. The
x 0 6 x 6
If f'(a) = f"(a) = ..... = fn–1(a) = 0 fact that we write equalities form left to right should not be
and g'(a) = g"(a) = ..... = gn–1(a) = 0 allowed to confuse the order of logical application.
but fn(a) and gn(a) are not both zero, then by repeated application
Caution
of L'Hospital's rule, we have
Before applying L'Hospital's rule at any stage be sure that
f (x) f n (x) the form is 0/0. Do not go on applying this rule even if the
lim  lim n
x  a g( x ) x  a g (x) form is not 0/0.

FREE BOOKS FOR JEE & NEET =>(@iitjeeadv)


Limits 1.85

ex  e x  x 2  2 then
f (x)
lim= L.
Example 10: Evaluate lim x 


x 0 sin x  x
2 2 g( x )
Solution: We obtain An analogous theorem holds if (a, ∞) is replaced by (–∞, a)
e x  e x  2x e x  e x  2x and if lim is replaced throughout by lim .
lim  lim x x
x  0 sin x cos x  2 x x  0 sin 2 x  2 x

By repeated uses of L'Hospital's Rule, we get Proof:


The result is a corollary of L’Hospital’s Rule I and the Chain
e x  e x  2 e x  e x 1
lim  lim Rule. Let t = , and set
x  0 2 cos 2 x  2 x  0 4 sin 2 x
x
ex  e x 11 2 1 1 1
lim    F(t) = f   = g(x) and G(t) = g   = g(x). Since
x  0 8 cos 2 x 8(1) 8 4 t t
t approaches 0 from the right if and only if x increases without
Caution bound,
lim lim  1  lim
Criticize the following use of L'Hospital Rule : t0 F(t) = t0 f  t  = x f(x) = 0,
 

 
x x x2
3 2
lim lim G(t) = lim g  1  = lim f(x) = 0.
x 2 x 3  3x 2  3x  2 t0 t 
t0
  x
ps
3x 2  2 x  1 1  1 
 lim By the Chain Rule, F′(t) = f′     2  and
t  t 
el
x  2 3x 2  6x  3
 
G′(t) = g′      .
eh

6x  2 6 1 1
 lim  lim  1  t   t2 
x 2 6x  6 x  2 6
je

Hence
The second equation is an incorrect use of L'Hospital's Rule,
iit

 1  1  1
f    2  f  
F( x ) t
@

since lim (3x2 – 2x – 1) = 7 and lim (3x2–6x+3) = 3. So,  t  t 


x →2 x →2 lim = lim = lim
t 0 G( x )

t 0 
 1  1  t  
0 1
the correct limit should be
7
. g     2  g  
3  t  t  t
xe x  log(1  x ) The last limit exists and is equal to L since
Example 11: Evaluate lim
x2 x 0
1
Solution: Since the form is 0/0, we shall apply f  
f x
lim   = lim
L'Hospital's rule. t
= L.
t 0
1 x  g  x 
xe x  log(1  x ) xe x  e x  {1 / (1  x )} g  
lim  lim , t
x 0 x2 x 0 2x F( t )
[form 0/0] By L’Hospital’s Rule I it follows that lim = L,
t 0 G ( t )


xe x  e x  e x  {1 / (1  x ) 2 } and the proof is complete.
 lim
x 0 2 An important observation is that all the forms of L’Hospital’s
Rule developed so far are valid whether L is finite or not.
0 111 3
  This fact requires no new proof and has really already
2 2 been established. The reason is that the basic conclusion of
A variation of L’Hospital’s Rule, not difficult to prove, is the L’Hospital’s Rule I is the
following: f (x) f ( x )
equation lim = lim ,and this holds good whether
L’Hospital’s Rule III. Let f and g be differentiable on an x a g ( x )

x a g( x )


open interval (a, ∞) with g′(x) ≠ 0 for x > a. If f ( x )


(i) lim f(x) = lim g(x) = 0, or not lim is finite or infinite.
x x x a g( x )


f ( x ) There is another significant variation of L’Hospital’s Rule,


(ii) lim = L, whose proof requires the Generalized Mean Value Theorem.
x  g( x )

FREE BOOKS FOR JEE & NEET =>(@iitjeeadv)


1.86 Differential Calculus for JEE Main and Advanced

It states that the condition (i) can be replaced by xlim


→a
|g(x)| g (c) f (c)
+|L| + .


= ∞. The specific statement which we prove is the following: g(x ) g( x )

L’Hospital’s Rule IV. Let f and g be functions which are Hence, the inequalities established in the first paragraph of
differentiable on a non-empty open interval (a, b) with g′(x) the proof imply that
≠ 0 for every x in (a, b). If f (x)
− L ≤ ε(1 + ε) + | L |ε + ε.
g( x )
(i) lim |f(x)| = lim |g(x)| = ∞,
x a  x a 
Since the right side of this inequality can be made arbitrarily
f ( x ) small by taking ε sufficiently small, it follows
(ii) lim = L, f (x)
x a g( x )

that lim = L, and the proof is complete.
x a g ( x )


f (x)
then lim = L. It is not difficult to derive variations of the preceding theorem
x a g ( x )


analogous to the modified versions described earlier. Thus,
Proof: with the obvious changes in the hypotheses, this last form of
Let ε be an arbitrary positive number. By hypothesis (ii), there
L’Hospital’s Rule also holds for two-sided limits and with a
exists a real number c in (a, b) such that
or L (or both) replaced by ± ∞.
f ( x )
 L < ε, for every x in (a, c). 3
x 1
g( x ) Example 12: Compute lim .
x  x  4
By hypothesis (i) there exists a real number d in (a, b), which Solution: Let f and g be the functions defined by
we shall for convenience assume to be in (a, c), such that,
f(x) = 3 x +1 and g(x) = x + 4, respectively. Since f′(x) =
ps
for every x in (a, d), the following three inequalities hold : 1
and g′(x) = 1, we see that f and g are differentiable
el
f (c) g (c)
g(x) ≠ 0, < ε, <ε 3( x + 1) 2 / 3
g( x ) g(x )
eh

on the interval (1, ∞) and that g′(x) ≠ 0. Moreover, x lim |g(x)|
(see Figure). It is a consequence of the last inequality that = lim |x + 4| = ∞, and
je

x 1
f ( x )
iit

lim
x  g( x )
= 3( x  1) 2 / 3 = 0.
lim
@

g (c) x  1
1− < 1 + ε, for every x in (a, d).
g(x ) ln x
Example 13: Evaluate lim
Now let x be an arbitrary real number in (a, d). By the x  x
Generalized Mean Value Theorem, there exists a real number Solution: Since lnx approaches ∞ as x approaches ∞,
y in (x, c) such that L'Hospital's Rule implies that
f ( x )  f (c) f ( y) lnx 1/x 1
 . lim  lim = lim = 0 .
x  x x  1 x  x
g (c)  g (c) g( y)
n ( x  1)
Hence Example 14: Evaluate lim
f ′( y) x  x2
f(x) = (g(x) – g(c)) + f(c).
g′( y) n ( x  1) 
Dividing by g(x), which cannot be zero, we get Solution: lim 
x  x 2
 
f ( x ) f ( y)  g (c)  f (c) 1
 1   .
[by L'Hospital's rule]
g ( x ) g( y)  lim x  1
g(x )  g(x ) =
x  2 x
An equivalent equation is
f (x)  f ( y)  g (c)  = lim
1 1 =0
–L=   L  1   x  
( x  1)2 x
g( x )  g ( y )  g (x)   
x
g (c) f (c)
–L + . Example 15: Evaluate lim x
x  e

g( x ) g( x )
Solution: Since ex approaches ∞ as x approaches ∞,
From the general properties of the absolute value, it follows that L'Hospital Rule implies that
f (x) f ( y) g (c) x 1
L  L 1− lim  lim 0
g( x ) g( y) g(x ) x  e x x   ex

FREE BOOKS FOR JEE & NEET =>(@iitjeeadv)


Limits 1.87

ln x x and f'(x) and g'(x) satisfy all the requirements that have been
Note: lim  0 and lim 0. stated for f(x) and g(x), we can then pass to the ratio of second
x  x x  e x
derivatives, etc. However, it should be borne in mind that the
f (x)
x100 limit of the ratio may exist, whereas the ratios of the
Example 16: Evaluate lim g( x )
x  ex derivatives do not tend to any limit.
 x 100
If f(x) = x2sin 1 , g(x) = x, then we have
Solution: lim 
x  e x   x
we apply L'Hospital's rule repeatedly 100 times lim f ( x ) = 0.
x→0 g ( x )
100. 99.........2. 1
= lim =0 f '( x ) 1 1
x  ex Here = 2x sin – cos
g '( x ) x x
Study Tip which oscillates when x → 0. Thus f/g may tend to a limit when
f′/g′ does not. Our condition is sufficient but not necessary.
While evaluating xlim
→ a {f(x)/g(x)}, when it is of the form
∞/∞, it is sometimes necessary to change it into the form 0/0, x  sin x
otherwise the process of differentiating the numerator and the Example 19: Evaluate lim
x  x  cos x
denominator will never end.
Solution: This limit has the indeterminate form ∞/∞. If
we try to apply L’Hospital’s rule, we get
ln x
Example 17: Evaluate lim 
ps
cot x x  0 x  sin x 1+ cosx
lim = lim
x  x  cos x x 1+ sinx
el
ln x
Solution: We have, lim , [form ∞/∞] The limit on the right does not exist, because both sinx
eh

x  0 cot x 

1/ x and cos x oscillate between –1 and 1 as x → ∞. Recall that


= lim , [form ∞/∞]
L’Hospital’s rule applies only if this limit exists. This does not
je

x  0  cos ec 2 x



mean that the limit of the original expression does not exist or
iit

= lim  sin x ,
2
[form 0/0] that we cannot find it. It simply means that we cannot apply

@

x0 x 
L’Hospital’s rule. To find this limit, factor out an x from the
2 sin x cos x 2  0  1 numerator and denominator and proceed as follows :
= lim  0  sin x 
x0 x 1 

1 1
x  sin x x 
lim  lim 
Example 18: Evaluate xlim m –x x  x  cos x x   cos x 
 x e . x 1 
 x 
xm
Solution: We have lim x m e  x  lim , sin x
ex 1
x  1 0  1
x x
= lim .
[form ∞/∞] x  cos x 1  0
1

m 1 x
= lim mx [form ∞/∞]

x x 1
e x 2 sin 0
Example 20: Evaluate lim x  
m(m  1) x m  2 0
 lim [form ∞/∞] x →0 sin x
x ex Solution: Applying L'Hospital's rule, we get
......... ......... .........
1 1 1 
m(m  1) ( m  2) ...3.2.1 lim m ! = 0, 2 x sin  x 2 cos   2 
 lim  x x x 
x e x
x   ex lim
x 0 cos x
[∵ ex → ∞ when x → ∞] 1 1
2 x sin  cos
Note: = lim x x . This limit does not exist.
f '( x ) x 0 cos x
If the quotient again yields an indeterminate form,
g '( x ) But this does not mean that the limit of the given function
at the point x = a, of one of the two above-mentioned types does not exist. We try another method.

FREE BOOKS FOR JEE & NEET =>(@iitjeeadv)


1.88 Differential Calculus for JEE Main and Advanced

1 1 2 / (1  x 2 )  2 
x 2 sin x 2 sin  lim = lim  . x 
x = lim x 0 x  0 sin x / cosx x  0  1  x2 
lim = =0.  tanx 
x →0 sin x x →0 sin x 1
x  2  x 
  lim  2 1 2
 x  0 1  x 2   x  0 tan x 
lim
This implies that L'Hospital's rule is inapplicable in this   
question.
x 5  2x 3  4x 2  9x  4
2  2 x  sin 2 x Example 24: Evaluate lim
Example 21: Evaluate x  lim x 1 x 4  2x 3  2x  1
(2 x  sin 2 x )esin x Solution: Since the form is 0/0, we shall apply
2  2 x  sin 2 x L'Hospital's rule.

Solution: xlim  5x 4  6 x 2  8x  9
 (2 x  sin 2 x )esin x   Limit  lim ,
2  2cos 2 x x 1 4x 3  6x 2  2
= xlim
 ( 2  2 cos 2 x )esin x  ( 2 x  sin 2 x )esin x .cos x [This is in 0/0 form. So, we again apply L´Hospital´s rule]
2 20 x 3  12 x  8
4 cos x  lim ,
= xlim x 1 12 x 2  12 x
 esin x (4 cos x  2 x  sin 2 x ) cos x
As x→ ∞, cosx becomes zero at several points. Hence, [This is in 0/0 form. So, we again apply L´Hospital´s rule]
the function is undefined at several points. Therefore the 60 x 2  12 60  12 48
limit cannot be evaluated. This implies that L'Hospital's rule  lim    4.
x  1 24 x  12 24  12 12
is inapplicable. We try to find the limit by another method.
ps
2  2 x  sin 2 x   x 2  2 cos x  2
el
2  sin x
lim = xlim 1  2 x  sin 2 x  e Example 25: Evaluate lim .
x  (2 x  sin 2 x )esin x    x0 x sin 3 x
eh

The first bracket approaches 1 as x→ ∞, but e −sin x oscillates x 2  2 cos x  2


je

between 1/e and e. Hence the limit does not exist. Solution: lim
x0 x sin 3 x
iit

Using L’Hospital’s Rule with Other Limit Properties  x 2  2 cos x  2 x3 


@

 lim  . 
( cos x)sin4x x 0 x4 sin 3 x 
Example 22: Evaluate lim 
x 3cosx
x0
3
Solution: This limit has the form 0/0, but direct application  x  x 2  2 cos x  2
 lim   lim
of L’Hospital’s rule leads to a real mess. Instead, we compute the x  0  sin x  x  0 x4
given limit by using the product rule for limits along with two
simple applications of L’Hospital’s rule. Specifically, using the x 2  2 cos x  2 , ∵ lim x  1
 lim  x  0 sin x 
product rule for limits (assuming the limits exist), we have x0 x4  
(1  cos x ) sin 4 x 2 x  2 sin x
lim  lim .
x 0 x 3 cos x x0 4x 3
(by L´Hospital´s rule for the form 0/0)
  
 1  cos x   sin 4 x   1 
=  lim   lim   lim 
 x 0 x 2   x 0 x   x 0 cos x  2  2 cos x
 lim
 sin x   4 cos 4 x   1  x0 12 x 2 [form 0/0]
=  lim   lim   xlim 
 x 0 2 x   x 0 1   0 cos x   lim
2 sin x

1
lim
sin x

1
.1 
1 .
1 x  0 24 x 12 x  0 x 12 12
=   (4)(1)=2.
2 cos 2 x
log(1  x )
2 Example 26: Evaluate lim
Example 23: Evaluate lim x  1/ 2 e 2 x  2ex
x0 log cos x
cos 2 x  0
Solution: Since the form is 0/0, we shall apply Solution: lim form 
e 2 x  2ex 
x  1/ 2
0
log(1  x 2 )
L´Hospital´s rule. lim 2 cos x .(  sin x )
x  0 log cos x
= x lim
 1/ 2 2e 2 x  2e

FREE BOOKS FOR JEE & NEET =>(@iitjeeadv)


Limits 1.89

  sin 2x  0 x cos x ln cos x


= xlim , form   2 lim
1/ 2 2e 2x
 2e  0 x 0 sin x . (1  x 2 ) ln(1  x 2 )

= lim 2 cos 2x  2 (1)   .


2 2 2 x cos x ln cos x
 2 lim . lim . lim
x 1/ 2 2c
4e 4e 2e x  0 sin x x 0 1 x 2 x 0 ln(1  x 2 )
Example 27: Evaluate the limit,  2  1  1  lim
ln cos x
, [form 0/0]
x  0 ln(1  x2 )
(e 2x2
 1  2 x )(cos x  1)
2
lim .
x 0  sin 3x  ln(1  3x )  x 4 1
. ( sin x )
 2 lim cos x
Solution: x 0 1
. (2 x )
 1  cos x  (e 2 x  1  2 x 2 ) x 2 1  x2
2

lim 
L= x→0  xlim
 x 2
 0 x · x 2 sin 3x  ln(1  3x ) 
2
1  sin x 1  x 2 
 2 . lim  .  = 1.
e  1  2t 2 x  0  x
2t 2
1 x cos x 
=– lim · lim
2 t  0 t 2 x  0 sin 3x  ln(1  3x )
where x2 = t Example 30: Find the values of a and b in order that
4(e y  1  y) x2 x (1  a cos x )  b sin x
1
= – ylim · lim lim may be equal to 1.
2 x 0 sin 3x  ln(1  3x ) x 0 x3
2 0 y
ps
where 2t = y
x (1  a cos x )  b sin x
x2 Solution: We have lim
el
1
=– · 2 · lim x 0 x3
x0 sin 3x  ln(1  3x )
eh

2
[form 0/0, so we shall apply L'Hospital's rule]
x2
Now l = lim
je

.
sin 3x  ln(1  3x ) 1  a cos x  ax sin x  b cos x ...(1)
x0
 lim
iit

x 0 3x 2
Using L'Hospital's rule, l = 2 .
@

9 Now the denominator of (1) → 0 as x → 0. Therefore if the


2 numerator of (1) does not tend to 0 as x → 0, then the given
Hence, L = – .
9 limit cannot be equal to 1. Hence for the given limit to be
Example 28: Evaluate lim tan x.log e sin x . equal to 1 the numerator of (1) must also → 0 as x → 0.
x  / 2
∴ 1 + a – b = 0 or a – b = –1 ...(2)

log e sin x  0
Solution: limit = lim form  Now if 1 + a – b = 0, then (1) takes the form 0/0. Hence by

x  / 2 cot x  0 applying L'Hospital's rule to (1), the given limit is equal to
1 a sin x  a sin x  ax cos x  b sin x
.cos x
[using L'Hospital's rule ] lim
= lim sin x x 0 6x
x   / 2  cos ec x
2
ax cos x  (b  2a ) sin x , [form 0/0]
= lim
x  / 2
  sin x.cos x  =0  lim
x 0 6x
ln ln(1  x 2 ) a cos x  ax sin x  (b  2a ) cos x
Example 29: Evaluate lim  lim
x  0 ln ln cos x x 0 6

ln ln(1  x 2 ) [by L'Hospital's rule]



Solution: We have, lim [form ∞/∞] a  b  2a 6  3a
x  0 ln ln cos x   1 [as given]
1 1 6 6
. . (2 x ) ∴ b – 3a = 6. ...(3)
ln(1  x ) 1  x 2
2

 lim Adding (2) and (3), we have –2a = 5 or a = –5/2.
x 0 1 1
. . ( sin x ) ∴ b = a + 1 = (–5/2) + 1 = –3/2.
ln cos x cos x Hence a = – 5/2, b = –3/2.

FREE BOOKS FOR JEE & NEET =>(@iitjeeadv)


1.90 Differential Calculus for JEE Main and Advanced

m n
Other Indeterminate Forms Example 33: Evaluate lim x (ln x ) , where m, n are
x 0 

Form 0 × ∞ positive integers.


m n
This form can be easily reduced to the form 0/0 or to the Solution: We have lim x (ln x ) , [form 0 × ∞]
form ∞/∞. x  0
n
Let lim f(x) = 0 and lim g ( x )    lim
(ln x ) , [form ∞/∞]
x →a x a


x 0 
x m
Then we can write lim f ( x ). g ( x ) n (ln x ) n 1 (1 / x )  n (ln x ) n 1 
x →a
 lim  lim   . ,
f (x) , x 0 
mx  m 1 x 0  m


x  m 
 lim [form 0/0]

x  a 1 / g( x ) [form ∞/∞, if n > 1]


n 2
or  lim
g( x ) , [form ∞/∞]  n  (n  1) (ln x ) .(1 / x )
 lim    .

x  a 1 / f (x) x 0  m 

mx  m 1
Thus lim f ( x ). g ( x ) is reduced to the form 0/0 or ∞/∞ which n (n  1) (ln x ) n  2
 lim  1
x →a
2
.
can now be evaluated by L'Hospital's Rule or otherwise. x 0 
m2 x m
n (n  1) (n  2)...upto n factors (ln x ) x  n ,
Example 31: Find lim cotx . log secx  lim  1 .
n
x  / 2
.
x 0 
mn x m
lim
Solution: x  / 2 cot x log sec x [0 . ∞ form]
ps [by repeated application of the above process]
n! m
= lim
log sec x
[form 0/0] . x  (1) n n ! . lim x m  0 .
 lim (1) n
mn
el
x  0 mn x  0 
x  / 2 tan x

Form (∞ – ∞)
eh

1
. sec x tan x
= lim sec x This form can be reduced to the form 0/0 or to the form ∞/∞.
 lim sin x cos x  0
je

x  / 2 sec 2 x x  / 2 When lim f(x) = ∞ and xlim→a


g(x) = ∞, then
x →a
iit

lim [f(x) – g(x)], [form ∞ – ∞]


@

Example 32: Evaluate lim (π – 2 tan –1 x) ln x x →a



x

(  2 tan 1 x )  0  1 1 
 lim   
lim  form 0  x  a 1 / f (x ) 1 / g( x ) 
Solution: x  1  
nx {1 / g ( x )}  {1 / f ( x )}
2  lim [form 0/0]
 x a
{1 / f ( x )} . {1 / g ( x )}
1  x2
Applying L’ Hospital’s Rule = lim 1 1 Now this can be evaluated by applying L'Hospital's rule or
x 
 otherwise.
(ln x ) 2 x  1 1
Example 34: Find lim   .
= lim 2 x (ln x )
2
 x0  sin x x

x  1  x 2   Solution: If x  0 , then sin x  0
1
= 2(ln x ) 2  2 x.2 ln x. and 1  1    
lim x sin x x

x  2x Similarly, if x  0 , sin x  0
 
= lim (ln x )  2 ln x
2
 form   and
1 1
− = → – ∞ – (–∞) = – ∞ + ∞
x 
 
x sin x x
1 2 Neither form reveals what happens in the limit. To find out,
2 ln x. 
= x x we first combine the fractions.
lim
x  x 1 1 x  sin x
 
1 sin x x x sin x
= lim [2ln x + 3] = 0.
x  x 2 and then apply L'Hospital's rule to the result.

FREE BOOKS FOR JEE & NEET =>(@iitjeeadv)


Limits 1.91

 1 1 x  sin x 0  1 
lim    lim    lim  2    1
x 0 sin x x  x 0 x sin x 0 x  0  sec x 

1  cos x 0 ∴ L = e–1 = 1/e.


= lim  
x0 sin x  x cos x 0  xx ) .
x
(x )


Example 37: Calculate lim ( x
sin x 0 x  0
= lim  0.
x0 2 cos x  x sin x Solution: We first calculate xlim

2 xx:
0 

 1 1  it is just exp( lim x ln x).


Example 35: Evaluate lim  2  x  0
x   x

tan 2 x 
ln x
Now lim x ln x  lim by L'Hospital's Rule
 1 1  x  0 x  0
Solution: xlim    (∞ – ∞) 1/ x
  x 2 tan 2 x 
1/ x
   lim  lim  x  0
 2   0
x  0 1 / x 2 x  0
= lim  tan 2
x  x   form 0  Hence lim xx = 1.
x 0  2   
2 tan x x0
x .x 2 
 x 2

 xx )  1 .
x
(x )
Therefore lim ( x
(tan x  x )(tan x  x ) x 0 

= lim
x 0 x4  
Example 38: Find lim e  e  2 cos   4
ps
(tan x  x )(tan x  x )
= lim 0 4
el
x 0 x . x3
e  e    2 cos   4  0
eh

tan x  x Solution: lim  form 0 


= 2 lim 0  4
 
x 0 x3
je

sec 2 x  1 e  e    2 sin 
iit

= 2 lim [by L'Hospital's rule] = lim (by L’ Hospital’s Rule)


x 0 3x 2 0 43
@

e  e    2 cos 
1  cos 2 x = lim
= 2 lim 2 2 0 122
x 0 cos x 3x
 
2 sin 2 x 2 = lim e  e  2 sin 
= lim 2
= . 0 24
3 x →0 1. x 3  
= lim e  e  2 cos  =
4 1
= .
Note: 0 24 24 6
The indeterminate forms 1∞, ∞0 are evaluated by first taking
Example 39: Determine lim (cot x)1/log x
logarithms and then finding the limit of the logarithm of the x→0

power [f1 ( x )]f ( x ) (which requires evaluating a form like 0.∞ ).


2 Solution: Let y = (cot x) 1/log x
.
1
Example 36: Evaluate xlim (cosec x)1/ln x log y = log (cot x)
→0 log x
lim log y = lim log cot x 
Solution: Let L = xlim (cosec x)1/ln x [form 1∞] ⇒ 

 
→0 x0 x→0 log x
∴  n L  lim
1
( n cos ec x ) , [form ∞/∞]  cos ec 2 x

nx
x 0 = (by L’ Hospital’s Rule)
lim cos x
(1 / cosec x) (-cosec x cotx) x 0 
1/ x
 lim
x 0 1/ x x 1
= lim . =–1
x 0 sin x cos x


 x  ,
 lim  
[form 0/0]
⇒ log lim y = – 1 ⇒ lim y = e–1 = 1/e.

x  0  tan x  x0 x0

FREE BOOKS FOR JEE & NEET =>(@iitjeeadv)


1.92 Differential Calculus for JEE Main and Advanced

Example 40: Find x→0lim 1  sin x  cos x  log(1  x ) However, in (2) we get L =
1
from where we can get the
x tan 2 x L
Solution: The inconvenience of continuously diffe- limit. We have L2 = 1 ⇒ L = ± 1.
rentiating the denominator, which involves tan2 x as a factor, Since the function is positively valued, the limit is non

-
may be partially avoided as follows. We write negative ⇒ L = 1.
1  sin x  cos x  log(1  x ) Here are some more examples:
x tan 2 x 9x  1 x
(i) lim (ii) lim
1  sin x  cos x  log(1  x )  x 
2 x  x 1 x 0 sin x
= .  
x3  tan x  cotx
(iii) lim
1  sin x  cos x  log(1  x ) x 0 cosecx
so that lim
x 0 x tan 2 x Leibnitz’s Formula for Differentiation of Integrals
1  sin x  cos x  log(1  x ) lim  x 
2
To differentiate an integral function, one way is to find the
= lim
x→0  tan x  integral and then differentiate as follows:
x 0 x3  
x2 x2
1  sin x  cos x  log(1  x ) t3 x 6 ( 2 x )3 x 6  8 x 3

2
= lim .1 Let f(x) = t dt =   
x 0 x3 3 3 3 3
2x 2x
1  sin x  cos x  log(1  x )
= lim 6 x − 24 x 2
5
x 0 x3 Hence, f '(x) = = 2x5 – 8x2.
3
ps
To evaluate the limit on the R.H.S., we notice that the numer-
But a better approach is given by Leibnitz rule:
ator and denominator both become 0 for x = 0.
el

d  
v( x )
1  sin x  cos x  log(1  x )   f ( t )dt  = f(v(x)) dv – f(u(x)) dv .

eh

lim
x 0 x3 dx  u ( x )  dx dx
 
cos x  sin x  [1 / (1  x )]
je

The proof will be dealt in the chapter of definite integral. We


= lim
iit

x 0 3x 2 can now differentiate the same function as follows:


d  2 
(by L’ Hospital’s Rule) x2
@

 sin x  cos x  [1 / (1  x ) 2 ]   t dt  = (x2)2 . 2x – (2x)2 . 2 = 2x5 – 8x2.


= lim du  2 x 
 x 2

x 0 6x
 t dt
2
 cos x  sin x[2 / (1  x )3 ] 3 1
= xlim   . Example 41: Evaluate lim
2x
0 6 6 2 x 0 x3
2
x
Cycling
 t dt
2
L'Hospital's rule does not help in evaluating the following limits. 0
When we apply it we just keep on cycling. We need to find lim 2 x 0
Solution: x 0 x 3  
the limits using some other method.
sec x    We apply L'Hospital's rule with the help of Leibnitz rule
Consider L  lim  ...(1)
 tan x  

x
2 2 x 5  8x 2 2 3 8 8
Limit = lim  x   .
sec x tan x x 0 3x 2
3 3 3
= lim
x sec 2 x
2 x3
tan x
= lim sec x

 ...(2)
 ntdt
x2
x  


2 Example 42: Evaluate lim
2
x  x4
sec x sec x
= lim = lim =L ...(3) x3
x  tan x
 ntdt
x sec x tan x
2 2
x2

Applying L'Hospital's rule again and again, we get the question 
Solution: xlim  
back in (3).  x 4

FREE BOOKS FOR JEE & NEET =>(@iitjeeadv)


Limits 1.93

(nx 3 )3x 2  (nx 2 ).2 x  3 x  1 ( x  1) ( x  1)  .


3/ 2
1
= lim = lim     2
x  4x 3 x 1  x  1 x 1 x  1  x  x  1
[by L'Hospital's rule] 
= lim
9 xnx  4nx 
 3
x 1   ( x  1)   ( x 3/ 2  1))  

=  lim    lim
    lim 
x  4x 2


x 1 x 1   x 1 x 1   x 1 x  1   
(9 x  4)nx 1
= lim lim
x  4x 2 x 1 x  x 1
2
.
 4  nx (by L' Hospital’s rule)
= lim  9  
x   x  4x
= lim  ( x )1/ 31  ( x )1/ 21  ( x )3/ 21  .
1 1 3 1
= 9 × 0 = 0. x 1  3 2 2  12  1  1
Example 43: Find the constants 'a' (a > 0) and 'b' 1 1 3  1
such that , =  (1) 2 / 3  (1) 1/ 2  (1)1/ 2  .
3 2 2  3
X
t 2 dt
 at
1 1 3 1
=    .
 239  1 7
=  . 3  9 .
lim 0
1. 3 2 2 3  6 
x→0
bx  sin x
0 3
Example 46: If lim 2 n n x dx = k then find the
Solution:
0
form hence using L`Hospital rule  5 n  n x 1
ps
x2 1
lim a x value of   , where [ . ] is the greatest integer function.
el
l = x→0 for existence of limit k
b  cos x
eh

lim b − cos x = 0 ⇒ b=1 2n n3x


x→0
n   n
je

Solution: lim dx (∞ × 0)
x5  1


lim x2 lim 1
iit

hence x→0
. x→0 =1
1 − cos x a+x 1
Put n =
@

2 t
=1 ⇒ a=4
a x 2
x
x   cos t 2 dt 1t x 5  1 dx
Example 44: Evaluate 0
0 t
lim . lim 0
x 0 x  6x
3 t 0 t 3
 
x
Applying L’ Hospital’s rule
x   cos t 2 dt 0
Solution: [form ] 2 1
0 0
lim
x 0 x  6x
3
t  2  1
  t  
32  t 2  1
Applying L'Hospital's rule we get
5
1 5
1  t 
d
x
lim t t
dx 0
1 cos t 2 dt t 0  3t 2
1  cos( x 2 )
= lim = lim 4 1
x 0 3x 2  6 x 0 3( x 2  2)  7
= 32  t 5
1  t5 = = k.
1  cos 0 0 lim 24
t 0 
=   0. 3
3(0  2) 6
 1   24 
3
x  x  x x 3
∴  k    7  = 3.
Example 45: Evaluate lim    
2
x3  1  x e x dx 
x 1
0
2


Example 47: Evaluate lim  
( 3 x  1)  ( x  1)  ( x 3/ 2  1)  0  x   x 
  0 e dx 
2
lim 2x
Solution: x 1 ( x  1)( x 2  x  1)  0 form 
   

FREE BOOKS FOR JEE & NEET =>(@iitjeeadv)


1.94 Differential Calculus for JEE Main and Advanced

e  x  1
2
2
Solution: If y =  g ( x )dx  , then 3 2
x

 0 
= .
5 4 x 0
lim
x2
Again using L' Hospital's rule, we get
 2   g ( x )dx  . g(x)
dy x

 

3 lim −e − x .(−2 x )
dx 0 2

3 3
= = .1= .
x 10 x→0 (2x ) 10 10
(  e x dx ) 2
2

0  
Now lim   form   2 x (1  t 3 ) 1 dt 
   x
x  x 2x2

0
e dx
Example 49: Find lim  4 x
x  3 1

.
  (1  t ) dt 
 3x 
2   e x dx  (e x . 1) 2  e x dx
x 2 x 2 2

 0  0  
 lim    form  Solution: Both the numerator and denominator tend to
 
2 2
x  e2 x ex zero as x → ∞, and hence L' Hospital's rule applies.
2
2x 1
2e x . 1 1 x 1  t 3 dt
 lim x2
 lim  0.
x  e .2 x x  x 4x 1
3x 1  t 3 dt
 1 x t2 1 1 
Example 48: Evaluate lim 
x 0  x5
0 e dt 
x4
 
3x 2  2 1

ps
= 1  8x 1  x3
3
x
3 e  t dt  3x  x 3  4 3
el
2

0
Solution: We have lim 0  form 0  
  1  64 x 3 1  27 x 3
eh

x 0 5
x
2 1
je

Using L' Hospital's rule, we get 



= 1/ x  8 1/ x 1
3 3
iit

3e x  3  3x 2
2

e  x  1  x 2  form 0  4 3
2

lim = 
@

lim  0  1 / x 3  64 1 / x 3  27
x 0 5x 4 x 0 x4 
Again using L' Hospital's rule, we get 2
1
x2 8 108
3 lim e .(2 x )  0  2 x = 4  .
= 3 7
5 x→0 
4x 3 64 27

Concept Problems P

1. Evaluate the following limits:
ln (1 + e −2 x ) a1/ x  b1/ x
a ( x ln a  1)  1
x (iii) lim (iv) lim
(i) lim x →−∞ ln (1 + e − x ) x  ln{x / ( x  1)}
x 0 x2
3. Evaluate the following limits:
(ii) lim
ln(tan 2 2 x )
x →0 ln(tan 2 x ) 2x  x sin 3x
(i) lim
x  5x 2  2 x  1
ln(sin x )
(iii) lim
x 0 ln(sin  x ) 2 x  sin x
(ii) lim
2. Evaluate the following limits: x  3x  sin x

ln (1 + e − x ) (ii) lim ln (1  e )
x
(i) xx  x
lim (iii) lim
x →∞ e− x x  x x  1 1  x  ln x

FREE BOOKS FOR JEE & NEET =>(@iitjeeadv)


Limits 1.95

ln x x
4. Show that lim
x x  m
 0, m > 0. (iii) lim 1 1  4 t 2 dt
.
x  sin x
x 1 x 1
5. Show that lim exists even if
x  2 x  sin x 7. Evaluate the following limits:
1  cos x x
lim
x  2  cos x
does not exist.
(i) 0 4  t 4 dt
lim
6. Evaluate the following limits: x  x3
x2

x2 t2
(i) lim 0
sin x dx
(ii) 
lim x
e dt
x 0 x3 x 1 x 1
x 1
(ii) lim 0 (tan x ) 2 dx
(iii) lim 0
x
1  t 2 dt
x 
x2  1 x  x2

Practice Problems N

ps
el
8. Evaluate the following limits: 14. Evaluate the following limits:
eh

2
3t
(i) lim (i) lim
1
sin x.sin 1
x  x2 
je

t  3(1  t )1/ 3  t  3 x 0 x6
iit

(ii) lim x  2 x  1 .
1/ 3


(ii) lim 1 tan x.tan 1 x  x 2 
@

1/ 2
x  x  3x  2 6
x 0 x
9. Evaluate the following limits:
sin 2 x  2 sin 2 x  2 sin x 15. Suppose that F is a function differentiable on the open
(i) lim
x 0 cos x  cos x 2
interval (0, ∞) and such that F′(x) =
1
, for every x > 0,
x
 1 ln(1  x )  Show that
(ii) lim   
x  0  x (1  x ) x2 
F( x ) F( x )
log a n (a) lim =0 (b) lim =0
10. Prove that lim 
 0 for a > 1, α > 1 x  x x  x2
n  n
(ln x ) n n F( x )
11. Show that lim = 0 and lim x  0 for all (c) lim = 0, for every positive integer n.
x →∞ n x  e x
x  xn
positive integers n.
a sin 2 t a
y e dt  
2

(1) n esin t dt
12. Prove that xlim (x – n)cosecxπ = , n being any 16. Evaluate lim y x .
→n x→0
integer.  x
17. Evaluate
1   (1) n  
13. Prove that lim cos ecx  
x n x  n 
 ( x  n ) 
 π 4 1/ h2
+h e π /3

(1) 
n lim ∫0
3 cos3 x dx − ∫
0
cos3 xdx
.
h→0
= , n being any integer. h 4 e1/ h
2

6

FREE BOOKS FOR JEE & NEET =>(@iitjeeadv)


1.96 Differential Calculus for JEE Main and Advanced

2
1.19 Geometrical Limits  tan( x / 2)   sin x 
 x / 2   x  x3
   .
Example 1: A circular arc of radius 1 subtends an angle = lim
x 0 ( x  sin x ) 4
of x radians, 0 < x < π/2 as shown in the figure. The point R is
the intersection of the two tangent line at P and Q. Let T(x) be 2
the area of triangle PQR and let S(x) be the area of the shaded = 1 lim  tan( x / 2)  lim  sin x  .
region then find 4 x→0  x / 2  x→0  x 
T( x )
(i) T(x) (ii) S (x) (iii) lim x3 3
x →0 S( x ) . lim = .
( x − sin x )


x→0 2
Example 2: A tangent line is drawn to a circle of radius
unity at the point A, and a segment AB is laid off whose length
is equal to that of the arc AC . A straight line BC is drawn to
intersect the extension of the diameter AO at the point P .
Prove that:
 (1  cos )
(i) PA = (ii) lim PA = 3
  sin  0
ps
el
eh

tan   =
x PR
Solution: (i) In ∆ OPR,
2 1
je

∴ PR = tan x = RQ
iit

2
( Length of tangent from a point outside the circle are
@

equal)
and ∠ PRQ = (π – x)
∴ T(x) = Area of ∆ PQR
Solution: See the labelled figure below:
1
= . (PR) (RQ) sin (π – x)
2
x
= 1 . tan2   sin x ...(1)
2

2
x sin x
= tan   –
2 2
(ii) S(x) = area of sector OPQ – area of ∆ OPQ
1 1
= (1)2 . x – . (1)2. sin x

2 2
(i) From the figure,
( x − sin x )   sin 
=
tan φ = ...(1)

2  x  sin x 1  cos 
tan   
lim T( x ) = lim 2 2 θ
(iii) x→0 Now in ∆ BPA , tan θ =
S( x ) x→0 ( x  sin x ) PA
2
tan 2 ( x / 2) sin x   (1  cos )
= lim ⇒ PA = =
( x  sin x ) tan    sin 

x 0

FREE BOOKS FOR JEE & NEET =>(@iitjeeadv)


Limits 1.97

 (1  cos ) 1 1
lim lim
(ii) 0   sin  3 2 0 
1 (2)3 
.
3 =   2  .... 
2 3 
1  cos  lim 3
= lim
0 0
3
 2   sin 
1 12 3
=  
1  cos  1 2 8 4
Now lim
0
=
 2 2
Example 4: The figure shows a fixed circle C1 with
  sin  3 t − sin 3 t equation (x – 1)2 + y2 = 1 and a shrinking circle C2 with radius
Let  = lim
0 = tlim
→0 r and centre the origin. P is the point (0, r), Q is the upper
3 27 t 3
point of intersection of the two circles, and R is the point of
where θ = 3t intersection of the line PQ and the x axis. What happens

-
3 t − (3 sin t − 4 sin t )3 to R as C2 shrinks, that is, as r → 0+?
= tlim
→0
27 t 3
t − sin t 4
= tlim
→0 +
9 t3 27
 4 8 4 1
= ⇒ ⇒ =
ps
+ =
9 27 9 27 6
el

3
Since lim Solution: We find the coordinates of the point Q by
eh

= 6,
0   sin 
solving (x – 1)2 + y2 = 1, and x2 + y2 = r2. On
je

lim PA = 1 . 6 = 3. 2
0 substraction, we get x = r .
iit

2 2
@

Example 3: In the figure shown, let f(θ) = area of ∆ Points P, Q and R are collinear. Hence mPR = mPQ
ABC and g(θ) = area of sector OBC – area of ∆OAC. Find
r2
the limit of ratio of f(θ) and g(θ) as C moves to B along the r 1 r
r0 4
circumference. ⇒ 
0x r2
0
2
r2
2
⇒ x=
r2
1− 1−
4
r2
Solution: We need to find Now, rlim x  lim 2
0 
r 0  1/ 2
1  r2 
(r  r cos )r sin  1  1  
f ()  4 
lim  lim 2
0 g ()

0 1 2 1

r2
r   r sin  .r cos 
2 2 = lim 2 = 4.
 1  cos   sin 
r 0 
1 r2
1  1  .  ....
   0 2 4
= lim   
2
 
0   sin  cos 

0 Example 5: Let (tan α) x + (sin α) y = α and
3 (α cosec α) x + (cos α) y = 1 be two variable straight lines, α

FREE BOOKS FOR JEE & NEET =>(@iitjeeadv)


1.98 Differential Calculus for JEE Main and Advanced

being the parameter. Let P be the point of intersection of the 2


lines. Find the limiting position of P when α → 0. 
= lim  lim 2 = 2
Solution: Here two straight lines, (tan α) x + (sin α)  0   0 
tan tan
y = α and (α cosec α) x + (cos α) y = 1 have their point of 2 2
intersection as,
 cos   sin    x tan  Again as, lim y = lim   x tan 
x= and y = 0
sin    sin   0 sin 
∴ when α → 0, we obtain the point P.
  x 
 cos   sin   0 form  = lim  
cos  
lim x =
Hence, 0 lim 0   0  sin 
 0 sin     
 sin   cos   cos   x
= lim = lim  lim =1–2
 0 cos   1  0 sin   0 cos 
(applying L' Hospital’s rule)

   [as lim x = 2]
0
 sin    2 sin cos 
= lim = lim  2 2
 0 2 sin 2  / 2 ∴ lim y = –1.

 0  0
2 sin 2
2 Hence, in limiting position, P is (2, –1).
ps
Concept Problems Q
el

eh

1. Show that the perimeter of a regular polygon of 5. A straight line AB moves so that the sum of its
je

4R θ intercepts OA, OB on two fixed straight lines OX, OY


iit

n sides is given by P(θ) = sin where


2π  2 is constant. If P be the ultimate intersection of two
θ=
@

is the central angle subtended by one side consecutive positions of AB, and Q the point where
n AB is met by the bisector of the angle XOY, then
of the polygon. Hence show that a circle of radius R has
prove that AP = QB.
circumference 2πR.
2. A straight line AB moves so as to include with two fixed 6. A segment AB = a (see fig.) is divided into n equal parts,
straight lines OX, OY a triangle AOB of constant area. each part serving as the base of an isosceles triangle with
Prove that the limiting position of the intersection of two base angles α = 45°. Show that the limit of the perimeter
consecutive positions of AB is the middle point of AB. of the broken line thus formed differs from the length
3. Find the limit of the sum of the lengths of the ordinates of AB despite the fact that in the limit the broken line
of the curve y = e–x cos πx, drawn at the points x = 0, “geometrically merges with the segment AB”.
1, 2, ...., n, as n → ∞.
4. Find the limit of the sum of the areas of the squares
constructed on the ordinates of the curve y = 21–x as on
bases, where x = 1, 2, 3, ...., n, provided that n → ∞.

Practice Problems O

7. The point C1 divides a segment AB =  in half ; the point 8. The side a of a right triangle is divided into n equal parts,
C2 divides a segment AC1 in half ; the point C3 divides a on each of which is constructed an inscribed rectangle
segment C2C1 in half ; the point C4 divides C2C3 in half, (see fig.). Determine the limit of the area of the step-like
and so on. Determine the limiting position of the point figure thus formed if n→∞.
Cn when n → ∞.

FREE BOOKS FOR JEE & NEET =>(@iitjeeadv)


Limits 1.99

of the substance available at the commencement of each


interval and to the length of the interval. Assuming
that the quantity of substance at the initial time is Q0,
determine the quantity of substance Qt(n) after the elapse
of time t if the increase takes place each nth part of the
time interval   t .
9. A weight hanging by a spring is made to vibrate by n
applying a sinusoidal force, and the displacement at time Find Q t  lim (Q) t .
(n )
t is given by n 

C 11. A straight line AB of constant length moves with its


f(t) = (sin t  sin t ) extremities on two fixed straight lines OX, OY which
  2
2
are at right angles to one another. Prove that if P be the
where C, α, β are constants such that α ≠ β. What happens ultimate intersection of two consecutive positions of AB,
to the displacement as β → α ? You may assume that α and N the foot of the perpendicular from O on AB, then
is fixed. AP = NB.
10. A certain chemical process proceeds in such a fashion 12. If PCP' be any fixed diameter of an ellipse, and QV any
that the increase in quantity of a substance during each perpendicular to this diameter ; and if the tangent at Q
interval of time τ out of the infinite sequence of intervals meet CP produced in T, prove that the limiting value of
(iτ, (i + 1)τ) (i = 0, 1, 2, ....) is proportional to the quantity the ratio TP: PV, when PV is infinitely small, is unity.
ps
el

1.20 Miscellaneous limits  nx   0 


lim  lim
1  nx 
= lim   =0
eh

n   x 0 n  1  0 
Repeated Limits
je

The operations of proceeding to the limit zero with each of


Caution
iit

two variables x and y may or may not be commutative when


@

applied to a function f(x,y). Thus


Here we note that interchange of limits is giving different
lim { lim ( x  y)} = lim x  0 , results. Hence it is advisable not to interchange the order of
x 0 y 0 x 0
limits indiscriminately.
lim { lim ( x  y)} = lim y  0
y 0 x 0 y 0 Representation of Functions Using Limits
but on the other hand
lim xn.
Example 2: Simplify the function y = n
xy x
lim { lim ( )}  lim  lim 1  1
x 0 y 0 x  y x 0 x x 0 Solution: When x2 < 1 i.e –1 < x < 1,
xy y lim x 2 n  lim ( x 2 ) n  0
lim { lim ( )}  lim  lim(1)  1 n  n 
y 0 x 0 xy y 0 y y 0

When x2 > 1 i.e. x < –1 or x > 1,


Hence the limits may or may not be commutative.
 nx  lim x 2 n  lim ( x 2 ) n   .
Example 1: Evaluate lim  lim n  n 
 and
x 0  n  1  nx 


When x = 1 i.e. x = ±1,


2
 nx 
lim  lim 
n   x 0 1  nx 

lim x 2 n  lim ( x 2 ) n 1 .
  n  n 
 nx   x 
Solution: xlim  lim  = lim  lim
0  n  1  nx 
   0, if x2  1
x 0 n  1
  x 




 n  ∴ lim x 2 n   1, if x2  1
x n 

= lim   = 1. , if x2  1
x 0  x  




FREE BOOKS FOR JEE & NEET =>(@iitjeeadv)


1.100 Differential Calculus for JEE Main and Advanced

 0, if 1  x  1
Example 4: If F( x )  lim x f ( x )  g ( x ) , find F(x)
2n

  1, if x  1 n  x 2n  1
, if x  1 or, x  1
 in terms of f(x) and g(x).
Here y is defined only when –1 < x ≤ 1. It is equal to 0 when x 2 n .f ( x )  g ( x )
–1 < x < 1 and to 1 when x = 1. Note that the points x = 1 Solution: ∴F(x) = lim
n  x 2n  1
and x = –1 are the points of change in definition.
 0.f ( x )  g ( x )
 1  x 2n   , 1  x  1
Example 3: Evaluate lim lim   0 1

x 1 n   1  x 2 n
   1.f ( x )  g ( x ) , x  1
 11
 1  x 2n  = 
Solution: Let f(x) = lim    f (x )  g(x )
n  1  x 2 n 
   x 2n , x  1 or x  1
We first simplify this function. We have  1
 1  2n
 0, if 1  x  1  x
lim x 2 n   1, if x  1  g( x ), if  1  x  1
n 
, if x  1 or, x  1  f ( x )  g( x )
 
∴ F(x) =  , if x  1
Hence, f(x) 2

1  0 f ( x ), if x  1or x  1
; 1  x  1
ps
1  0

 0 1
Example 5: Evaluate lim lim tan x 2 ( x  1) n.sin x
el
2 n
=  ; x  1, x  1
 0 1 x 0 n  x  ( x  1)
eh

1  1
1  1 ; x  1
je

 tan x 2  ( x  1) n .sin x
Solution: Let f(x) = lim
iit

n  x 2  ( x  1) n
 1 
 1 
@

using lim  x 2n  = 0 when x < – 1, x > 1.  tan x 2  ( x  1) n .sin x


n    lim , left nghd of x  0
1
   n  x 2  ( x  1) n
 1 
 x 2n  
 tan x 2
f(x) =   sin x
1 , 1  x  1  lim ( x  1)
n
  n  , right nghd of x  0
= 1 , 1  x  1 
x2
 1
0 , x  1  ( x  1) n
 
The graph of f(x) can be drawn as follows:  tan x 2
 , left nghd of x  0
f(x) =  x 2
sin x , right nghd of x  0


Since xlim f ( x )   , lim f ( x )  0 , the required limit does


0 
x 0 

not exist.
Example 6: Let f: R → R be defined by
 
Now we find the limit of f(x) as x → 1. f ( x )  lim  lim (cos m !x )  n  .
m   n  
lim f ( x )  1 Prove that f(x) is 1 for rational values of x and, 0 for irrational
x 1
values of x.
lim f ( x )  1 .
x 1 Solution: Let x be a rational number say p/q, where p,
 1  x 2n  q are integers prime to each other. By taking m sufficiently
Thus, the limit lim lim   does not exist. large, m! πx can be made an even integral multiple of π, so
x 1 n  1  x 2 n
  that cos m! πx = 1.

FREE BOOKS FOR JEE & NEET =>(@iitjeeadv)


Limits 1.101

Example 8: Evaluate
Hence f(x) = nlim 2n
 (1) = 1 where x is rational.  n n n 
If x is irrational, then cos m! πx will always lie between –1 lim   2  ....... 2 
n   n 2  12 n  22 2n 
and 1 for any integral value of m so that
(cos m! πx)2n = (rm)2n where |rm| < 1 1  n2 n2 n2 
for a fixed value of m. Solution: nlim  2 2  2  .......

 n  n 1 n  22 2n 2 
Thus, f(x) =
lim lim (r )2n = 0, when x is irrational.
m n  m 1 1 1 1 
lim   ..... 
Hence, f(x) is 1 for rational values of x and, 0 for irrational = n  n   1 2 2
2
n
2

values of x. 1 
 n 1    1  n 
   n   
Example 7: Show that the function f: R → R defined by 1
1 n 1 1 1

= n  n 
 sin 2 (n ! x )  lim  dx  tan 1 x 0
f ( x )  lim  lim  is equal to 0 when x is r 1 r
2
0 1 x 2
n   t 0 sin 2 ( n ! x )  t 2 
  1  
n
rational and to 1 when x is irrational.
Solution: Let x = p/q be a rational number. Then by = tan–11 – tan–10 = π .
4
taking n sufficiently large n! πx can be made an integral
multiple of π so that sin n! πx = 0. Example 9: Evaluate lim
n n
0 n 
Hence f ( x )  tlim
0 0  t 2
 0 when x is rational. n
ps
1  1.2.3........n 
If x is irrational, then 0 < sin2 n! πx < 1. Solution: nL  lim n  
n  n  n.n.n.......n 
el

1
Then f ( x )  nlim lim  1 n
eh

1 2 3
 t 0 1  t 2 / sin 2 ( n ! ) = lim
n  n  n n  n n  n n  ......n n 
je

1 1
= 1 when x is irrational.
iit

 nxdx  1 .
=
1+ 0 =
@

0
Thus, f(x)=0 when x is rational and 1 when x is irrational.
Hence, L = e–1.
Summation of infinite series using definite 1 2n r
integral as the limit of sum Example 10: Evaluate lim
n  n
 2 2
r 1 n  r
From the definition of definite integral, we have
Solution: On dividing numerator and denominator by n,
1 ψ(x)  r 
b
lim ∑ f  n  = ∫ f (x )dx 1 2n r
n →∞ n
r = φ( x ) a we get lim
n  n
 2 2
where, r 1 n  r

(i) Σ is replaced by ∫ sign, 1 2n r/n


r = lim
n  n

r 1 1  ( r / n )
(ii) is replaced by x, 2
n
1 Now we find
(iii) is replaced by dx,
n 1 2n
(iv) To obtain the limits of integration, we use a = lim  0 and b = lim 2
n  n n  n
φ( x ) ψ (x)
a = lim and b = lim 1 2n r/n x

2
n →∞ n n →∞ n lim  dx
n  n 0
The limiting value is obtained by finding the definite integral. r 1 1  ( r / n ) 1 x2
2

Special case of definite integral as limit of sum


2

1 n r
1 = 1+ x2
lim  f     f ( x )dx 0
r 1  n 
n  n
0 = 5 −1 .

FREE BOOKS FOR JEE & NEET =>(@iitjeeadv)


1.102 Differential Calculus for JEE Main and Advanced

Example 11: If A = [ai j ]n × n, where ai j = i100 + j100, then lim f ( x ) lim g ( x )


n lim  = x a x a 1 2
  1m 2   2 m1 .
find lim
 a ii .
x a lim u ( x )
x a
lim v(u )
x a
m1 m2
i 1
n  101
n
n  [x 2 ] sec x 
Example 13: Evaluate lim  
Solution: We have  a ii  a11  a 22  ......  a nn x 0  (sin 2 x ) x

2

x cot x 
i 1
n  [x 2 ] sec x 
Now, lim  a ii = lim 
2 1 100
2 100
 ......  n100
 Solution: x 0 
lim
2 x 2

x cot x 
i 1 n   (sin x )
n  101 n101
n
 lim[ x 2 ] lim sec x 
1 =  x 0 x 0 
100
r
100 2  
1
  n  = 2 x dx 
100 2 x 2

= 2 lim .  lim (sin x ) lim x cot x 


n 
n 101  x 0 x 0 
r 1 0
 0 1
Example 12: Evaluate the limit =  
 1 1
lim n ( ) (11 . 22 .... n n ) . 1 sin x n (1  x )
− 1+ 1
2
1
n n2
1

x →∞ Example 14: Evaluate xlim


0 x 2 tan x x sec x
ps
Solution: Taking the logarithm of the expression inside
the limit, we find that it is Solution: 1 sin x n (1  x )
el
lim
x 0 x 2 tan x x sec x
eh

1 1 1 n
1 n k k
 1   ln n  2
2 n
 k ln k   ln .
n k 1 n  n  sin x n (1  x )
je

n k 1
x x
iit

= lim
We can recognize this as the integral x ln x dx, Evaluating x 0 tan x x sec x
@

this integral using integration by parts, we find that x x


1 1 1x 1 1 1
0 x ln dx  2 x 0 2 dx   4 .
1
2
ln x 0  =  
1 1
Therefore, the original limit is e–1/4. sin 2 x  x tan x 2
Example 15: Evaluate lim
x 0 cos ec3 x cot x
Limit in Matrices and Determinants
 f (x) g(x )  sin 2 x  x tan x 2
Let A =   then we have Solution: xlim
0
 u(x) v( x )  cos ec3 x cot x
Here the limit of each of the functions does not exist
 lim f ( x ) lim g ( x )  simultaneously. Therefore, the above limit can be evaluated
lim A   
x a x a
 lim u ( x ) by finding limit of the expansion of the determinant.
x a lim v( x ) 
 x a x a 
∴ limit = lim (sin 2 x cot x + x tan x2 cosec3x)
Note that the limit exists if and only if the limits of each of x→0
the functions exist simultaneously.
f (x) g(x )  sin 2 x x tan x 2 
Let ∆ = then we have
= lim   
x 0  tan x sin 3 x 
u ( x ) v( x )
 2 sin 2 x x tan x 2 
lim   lim (f(x)v(x) – g(x)u(x)).  
x a x a x x2
= x 0  tan x   = 2 + 1 = 3.
lim 2x
If the limits of each of the functions exist simultaneously then  sin 3 x 
the above limit can be also be evaluated as  x 
 x3 

FREE BOOKS FOR JEE & NEET =>(@iitjeeadv)


Limits 1.103

Concept Problems R


1. Define the function without limit
(iii) y = lim x 2 + a 2
y  lim (cos 2 n x ) . a →0
n 
2. Define the following functions without limit: (iv) y  nlim

(arc cot nx )
(i) y  lim n1 x n
(x ≥ 0)
n 
3. lim x 2 n ( x 2 + 1) + 2 x , find lim f(x).
Let f(x) = n x→1
x 1 + x 2n
(ii) y  lim (x ≥ 0)
x  1  xn

Practice Problems P

ps
tan  x 2  ( x  1) n sin x n
el
4
4. Define f (x)= lim
n  x  ( x  1)
2 n , and find 10. Evaluate lim
n 
 n  4k .
eh

lim f ( x ) . k 1
x→0
je

3
5. Sketch the graph of the following functions for n
2  2k 
11. Evaluate lim  n   1
iit

x≥0 n 
k 1 n 
lim  1 
(a) φ(x) = n
@

x  1 x 
n

n
3 3i
n
lim log e (1+ x )
(b) Ψ(x)= n→∞
12. Find the limit lim
x →∞
∑n 1+
n
i =1
log e n
n  i2 π 
[ x x ] , x  0 13. Evaluate lim
i
∑ n 2 sin  n 2  .
 lim f(x) x →∞
i =1
6. Let f(x) =  t , find x→0
 sin x 
 lim   ,x  0 1/ n
 t   x  lim  (2n )! 
14. Evaluate n n
where [.] is G.I.F.  n !. n 

 ax 2  bxy  cy 2  15. Evaluate


 
7. Evaluate lim  lim 2 
x 0 
 y  0 dx 2
 exy  fy 
  1/ k
(e1/ k ) 2 (e1/ k )3 (e1/ k ) k 
lim  e 2 3  ......  
2 2 2
k
 k k k k 

 ax 2  bxy  cy 2 

– lim  lim 2 
, ad ≠ 0, cf ≠ 0.
y 0 
 x  0 dx 2
 exy  fy 
 2n
10n
16. Evaluate lim
n 
 ( n  2k ) 2 .
1 k 1
lim 1
then find the values of x.
8. If
 
x  n
sin 1 x 1
3n / 2
  k 
9. Simplify the function φ(x) = n
lim (2/π) arc tan(nx). 17. Evaluate lim
n 
 n sin  n 
.
k 1

FREE BOOKS FOR JEE & NEET =>(@iitjeeadv)


1.104 Differential Calculus for JEE Main and Advanced

Target Problems for JEE Advanced

Problem 1: If α and β are the roots of the quadratic (cos ) x  (sin ) x


equation ax2 + bx + c = 0, then evaluate Problem 2: Evaluate lim
x 2 x2
1  cos(cx  bx  a ) 
 2
 Solution: Limit
lim  
x
1

 2 (1   x ) 2


= lim (cos )  (sin )  cos   sin 
 x x 2 2

Solution: ax2 + bx + c = 0 has roots α and β, then x 2 x2


a b x 2 x 2
= lim cos ((cos )  1)  sin ((sin )  1)
2 2
 c  0
x 2
x x 2 x2
1 1 Put x – 2 = t
i.e., cx2 + bx + a = 0 has roots and
α β
 cos α ) t − 1 ((sin α ) t − 1)
= lim cos 2 α  +
 t →0
lim sin 2
α
 b a  1  1 t →0  t  t
⇒ c  x2  x    c  x    x  
 c c      = cos2α ncosα + sin2α n(sinα).
ps
1  cos(cx 2  bx  a ) 
  xx  aa

el
lim   Problem 3: Evaluate lim
x 1/  
 2(1  x ) 2


x a a x  aa
eh

enx  ena
x a
je

 2  cx  bx  a  
2 lim
 sin   
Solution: Limit = x a (a x a  1)
aa (x  a )
iit

  2  (x  a)
= lim  
@

 
e 
x 1/  2
 (1 x )  xnx
 eana
  = lim
  x a a na ( x  a )
a

 cx  bx  a 
sin 
2
 = lim 
eana e xnx  an a  1  ( xnx  ana )
= 2 a a na .( xnx  ana )( x  a )
lim   x a

x 1/  (1  x ) a a  xnx  ana 


= lim
c 1  1  x a a a .na .( x  a )
sin   x    x   
= lim 2     put x – a = t
x 1/   1 ( t  a )n ( t  a )  ana
  x   = lim
   t 0 (na ) t

c 1  1  c 1 a (n ( t  a )  na )  t.n ( t  a )


sin   x    x    = lim
  x   t 0 (na ) t
 2      . lim  2  
= lim
x 1/  c 1  1 x 1/    t
x   x   n  1  
2      a   lim n ( t  a )
= lim
t 0 t t 0 na
na  
c 1 1 a
  
= 2  c  1 1 .
1.     1
 2     = + 1 = logae + 1.
na

FREE BOOKS FOR JEE & NEET =>(@iitjeeadv)


Limits 1.105

Problem 4: The integral value of n for which the Problem 6: Evaluate a, b, c and d, if
x 3 lim . 4 3 2
cos 2 x  cos x  e x cos x  e x  x ( x + ax + 3x + bx + 2

lim 2
x 0 xn x 4 − 2 x 3 − cx 2 + 3x − d ) = 4
is a finite nonzero quantity.
Solution: Here, lim ( x 4 + ax 3 + 3x 2 + bx + 2
x 
Solution: The limit is equal to
− x 4 + 2x 3 − cx 2 + 3x − d ) = 4 (∞ − ∞ form)
1 2 4 6   2 4 
lim n 1  x  x  x  ...  1 1  x  x  ..  Rationalizing
x →0 x 

2 ! 4 ! 6 !   2 ! 4 ! 
(a  2) x 3  (3  c) x 2  (b  3) x  (2  d )
 x2 x3  x3  lim
 1  x    .....     x 
x 4  ax 3  3x 2  bx  2  x 4  2 x 3  cx 2  3x  d
 2! 3!  2 
   Since, limit is finite, so the degree of the numerator must be
 x2 x4 x6   x3 2x 5  x3 2. So, a – 2 = 0
    x  x    ..... 
2

2 ! 4 ! 6 !   3! 5!  2 i.e., a = 2.
 lim 
x0 xn (3  c) x 2  (b  3) x  (2  d)
lim
 x3 x4 x5 x5  x3 x 
x 4  ax 3  3x 2  bx  2  x 4  2x 3  cx 2  3x  d 
     .....  
2 2 12 24 2
 lim   Dividing numerator and denominator by x2.
ps
x0 n We get
x
el
= nonzero if n = 4.
(3  c)  (b  3) / x  (2  d ) / x 2
lim
eh

Problem 5: Evaluate x  a 3 b 2 2 c 3 d
1     1  2  3  4
lim ( x  p)( x  q )( x  r )( x  s)  x x x 2 x3 x 4
je

4 x x x x
x 
iit

3+ c
4 ( x  p)( x  q )( x  r )( x  s)  x ⇒ .
Solution: xlim
@

 2
3+ c
It is given that =4 ⇒ c=5
( x  p)( x  q )( x  r )( x  s)  x 4 2
= lim
x  ( 4 ( x  p)( x  q )( x  r )( x  s)  x ) Hence, a = 2, c = 5 and b, d are real numbers.

.
1
( ( x + p)( x + q )( x + r )( x + s) + x 2 ) Problem 7: Evaluate lim
x  ln  x2 1  x 
3
x0 x
a 4  b4
[using a – b =
(a  b)(a 2  b 2 )
] Solution: Put ln  x2 1  x = t  ...(1)

Ax 3  Bx 2  Cx  D
x2 +1 – x = e
t –t
lim or x2 +1 + x = e
x
 4 ( x  p)( x  q )( x  r )( x  s)  x  On subtraction, 2x = e–t – et
. 1
( ( x + p)( x + q )( x + r )( x + s) + x 2 )  e t  e t 
or x =  
 2
[where A = Σp, B = Σpq, C= Σpqr and D= Πp]  
B C D As x → 0, t → 0 from (1).
A  2  3
lim x x x e t  e t
x   p  q  r  s  t e t  e t  2t
4 1 lim
 x  1  x  1  x  1  x   1    lim 2 t 0
 e2 t  1 
3
     t 0 3 =
 e t  e t  2.t 3  
A 1     2t 
= (1  1)(1  1)  4 (p  q  r  s) .  2 

FREE BOOKS FOR JEE & NEET =>(@iitjeeadv)


1.106 Differential Calculus for JEE Main and Advanced

e t  e t  2t  e2 t  1  Problem 9: Find the value of


  lim  as lim  1
t 0 2t 3  2
 t 0 2 t   2
 x

2

(put t = 3y) lim  sin 1 x  tan 1 x  .


x 0  x 3 

e3 y  e 3 y  6 y
= lim
y 0 2.27 y3 3
Solution: sin–1x = x + x +
3 5 5 7 .....
x + x
e   
3
y
 e y  3 e y  e y  6 y 6 40 112
= lim
3 5 7
y 0 54 y3 x x x
and tan–1x = x    .........
 y
   3 5 7
3
y
 e e 3(e y  e  y  2 y) 
  lim   
2
54 y3 54 y3  2
 x

y 0
(1∞ form)
2




 lim  3 sin 1 x  tan 1 x 
x 0  x 
  2y 3 
8 e 1  1 e y  e y  2 y  L 
( say )

= lim     2 2 
y  0  54 2y  9 3  = lim
x 2  x 3
 sin 1
x  tan 1 x  1
 
2y 

x 0

or 
4 
 ⇒
8

4

1
. Now L = 2 lim

2 sin 1 x  tan 1 x  x 3 
27 9 9 27 6 5
x0 x
ps
 x3 3 5 x3 x5 
n
r3  8 2  x   x  ..  x   .....   x 3
 r3  8
el
= 6 40 3 5
Problem 8: Evaluate lim
n 
2 lim  
eh

r 3 x0 x5
n
r 8
3 1
je

 r3  8
=–
Solution: lim 2
n 
iit

r 3
1
 r  2   r  2r  4 
@

n 2 Limit = = e–1/2.
= lim     2  e
r 3  r  2   r  2r  4 
n 

Problem 10: A sequence of numbers xn is determined


 r  2  n  r  2r  4 
n 2
= lim      2  x n 1  x n  2
n 
r 3  r  2  r 3  r  2r  4  by the equality x n  and the values x0 and x1.
2
 1 2 3 4 5 (n  5) (n  4) (n  3) (n  2)  Compute xn in terms of x0, x1 and n. Also prove that
= lim  ....  ×
n   5 6789 (n  1) (n ) (n  1) (n  2) 
lim xn =
x 0 + 2 x1
19 28 39 
.
 (n 2  2n  4) ( n 2  3) ( n 2  2n  4)  n
   .... 2  2  2  3

 7 12 19 ( n  6n  12) ( n  4n  7) (n  2n  4)  
x n 1  x n  2

 1.2.3.4 (n 2  3)(n 2  2n  4) 
 Solution: x n 
= lim    2
n   ( n  1) n ( n  1)( n  2) 7.12 
 
x1 + x 0 x + x1 x + x2
2  (n 2  3)(n 2  2n  4) 
  x2 = ; x3 = 2 , x4 = 3 etc.
= lim   2 2 2
7 n  
 ( n  1) n ( n  1)( n  2 ) 

  3  2 4  
x 0 − x1
x2 – x1 =
 1  2  1  n  2  
2   n  n   2
= lim   x1 − x 2
7 n 
  1   1 1    1  2  
1 1 ( x 0 − x1 )
x3 – x2 = =–
 
 n   n   n  
 2 22
2 (1  0)(1  0  0) 2 x 2 − x3 ( x 0 − x1 )
=  . x4 – x3 = =+
7 (1  0)1(1  0)(1  0) 7 2 23

FREE BOOKS FOR JEE & NEET =>(@iitjeeadv)


Limits 1.107

Problem 12: If α1, α2, ...., αn are the roots of equation


x5 – x4 = = – ( x 0 − x1 ) xn + nax – b = 0, show that (α1 – α2)(α1 – α3) ....(α1 – αn) =
24 n(α1n–1 + a).
  

x n − 2 − x n −1
xn – xn – 1 = = (–1)n · ( x 0 − x1 ) Solution: Since α1, α2, ..., αn are the roots of equation
2 2n xn + nax – b = 0 we have
add ——————————————————— xn + nax – b = (x – a1)(x – a2)...(x–an)
xn – x1
x 0 − x1 x n  nax  b
 1 1 1 1  ⇒ = (x – α2)(x – α3)....(x – αn)
1  2  2  3  ......  (1) n  2 
n
=
 2 2 2  x  1
2
lim x 0  x1  1 
n (xn – x1) = lim   x n  nax  b lim [(x–α )(x–α )...(x–α )]
n 2  1  (1 2 )  ⇒ lim = x
x 1 x  1 2 3
1
n

lim x – x = x 0 − x1 × 2
nx n 1  na
n n 1
2 3
lim = (α1 – α2)(α1 – α3)...(α1 – αn)
lim x = x 0 − x1 + x = x 0 + 2 x1
x  1 1
n n
3 1
3 (using L’ Hospital’s rule on L.H.S.)

⇒ (α1 – α2)(α1 – α3)....(α1 – αn) = na1n–1 + na.
Problem 11: For n ∈ N, let xn be defined as
ps
n xn
 1
1  n  = e, then find nlim nx n 1  (n  1) x n  1
el
xn .

  Problem 13: Find lim
eh

n xn x 1 (e x  e) sin x
Solution: We have  1  
1 if n = 100.
= e ...(1)
je

 n  nx n ( x  1)  ( x n  1)
iit

Taking log on both sides of equation (1), we get Solution: l = lim


x 1 (e x  e) sin x
@

(n + xn) ln  1   = 1
1

 n Put x = 1 + h so that as x → 1, h → 0

⇒ n + xn =
1
 1 ∴ l = – lim

h · n(1  h ) n  (1  h ) n  1 
ln 1+  h 0 e(e  1) sin h
h
 n

 
1
⇒ l = – lim  n · h 1  n C1 h  n C2 h 2  n C3 h 3  ...
xn =  1 – n ...(1) x 1 
l n 1 + 

 n
n +1 
 1  n C1 h  n C 2 h 2  n C3 h 3  ....  1 
 

Let =u ⇒ nu = n + 1
n 1
.
 1   sin h 

1 e h

⇒ n= e (h 2 )   
u −1 
 h   h 
lim x n = lim  1  1 
∴ n  n 2  n C2  2n 2  n (n  1) 
u1   n u u 1  =– =–  
e  2e 
0
(u  1)  ln u
= lim  0  form
(u  1) ln u
u1   n2  n n (n 1)
=– =–
1 1 2(e) 2(e)
1 2
u u 1
= lim u  1 = lim = .  5050 
u1 u1 1 1 2
 ln u  If n = 100 then l = –  .
u u2 u  e 

FREE BOOKS FOR JEE & NEET =>(@iitjeeadv)


1.108 Differential Calculus for JEE Main and Advanced

Problem 14: Let f(n , θ) 1 a 


= lim   n 
 2      2   n   n ! n! 


= 1  tan  1  tan 2 2  1  tan 3  ....
 2  2   2   1 1 a 
= lim    n 1  [using (1)]
θ n   n ! (n  1)! (n  1)! 
to n factors . Show that nlim f (n , θ) = .
 tan θ  1 1 1 1 1 a 
= lim     ......    1
θ cos θ n   n ! (n  1)! (n  2)! ( 2)! 1 ! 1 ! 
Solution: 1 − tan2
cos 2 θ
= ; {a1 = 1 as given}
2 2
cos θ
 1 1 1 1 1 1
    ......    =
θ 2 = nlim
  n ! (n  1)! (n  2)! (2)! 1 ! 1 
e
1 − tan2 2 = 2 θ
and so on.
2 cos
2 2  1 1 1 
Hence f(n , θ) as, e  1     ........ 
θ  1 ! 2 ! 3 ! 
cos θ θ
cos cos n −1
θ 2 ...... 2 lim x , when x 2 = a + x
= Problem 16: Solve n
cos 2 . 2 θ θ n n n–1
and
2 cos cos 2 n
22 2 x0 = a.
cos θ 1 Solution: Here, x0 = a , x1 = a + a which shows
.
θ
= cos cos θ θ θ x1 > x 0
....cos n cos n
ps
2
2 2 2
2 2 ∴ assuming xn > xn – 1 ⇒ x n−1 – a < xn – 1
el
θ θ θ θ   
But cos. cos 2 . cos 3 ...... cos n  4a  1  1  4a  1  1 
⇒  x n 1    x n 1   <0
eh

2 2 2 2

 2 
  2 

sin θ
je

= n 4a + 1 + 1
2 sin θn ⇒
iit

2 xn – 1 < .
θ 2
Hence f(n , θ) = cot θ . 2n tan n
@

2 ∴ lim x = lim x = l
 tan n n n– 1 n n
2
= cot θ .
 
 ⇒ l2 – l – a = 0

2 n
1  1  4a
⇒ l= , since l ≥ 0
Now lim f(n, θ) = θ cot θ. 2
n

Problem 15: Evaluate ∴ l = 1 + 1 + 4a



2
 a 1  a2 1   an 1 
lim  1   ........  , 1  4a  1
n  lim x n 
 a1   a 2   an  or .
n  2
where a1 = 1 and an = n (1 + an – 1) ∀ n ≥ 2.

 an 1  a+b
 a1  1   a 2  1  Problem 17: If a1 = ab1 , b1 = ,
Solution: nlim    ......   2

 a1   a 2   an 
a1 + b1
an a2 = a1b 2 , b2 = ...
We know, an – 1 + 1 = ...(1) 2
n
and so on for (a > b > 0) then prove that
 a  a  a   a n 1  1
lim  2   3   4  .....  n  1  a .a .....a
n   2   3   4    1 2 a 2  b2
n lim a n  lim b n 
n  n   a 2  b2 .
a 1 an tan 1  
= lim n 1 = lim [using (1)]  b 
n  ( n  1)! n  n !  

FREE BOOKS FOR JEE & NEET =>(@iitjeeadv)


Limits 1.109

θ 
Solution: Put b = a cos θ , we get b1= a cos2 and 2 sin n 1
θ 2  π
= 2θ = 2 · π = .
2
a1 = a cos l = 2(n + 1) 2 sin = 
2 n1 3
2 6
θ θ θ 2n 1
b2 = a cos cos2 and a2 = a cos cos θ
2 4 2 4 1n
Similarly we get  3n
Cn 
  a
Problem 19: Given nlim
  Cn 
= where a and b
θ θ θ 
2n
b
an = a cos cos ...... cos and
2 4 2n are relatively prime, find the value of (a + b).
θ θ θ
bn = a cos cos .... cos2 3n
Cn (3n )! n !n !
2 4 2n Solution: Consider = ·
2n
Cn n !(2n )! (2n )!
a sin θ
⇒ an = n and
2 sin θ / 2n 3n (3n  1)......(2n  1)(2n )!
=
(2n )!(2n )(2n  1)....(2n  1)n !
bn = a sin θ cos θ / 2
n

2n sin θ / 2n ( 2n + 1)( 2n + 2)( 2n + 3).....( 2n + n )


=
a sin  a sin  . ( n + 1)( n + 2)( n + 3)..........( n + n )
Now, lim a n  lim 
n  n  2 n sin  / 2 n  1n
 
Let P = lim 1  n  1  n  ...... 1  n  
ps
n  n 1
 n2  nn   
a (a 2  b 2 ) a 2  b2
lim a n  
el
1
⇒ n  a cos (b / a )  a 2  b2 . Taking logarithm, ln P
eh

tan 1  
 b 
  1  n   n   n 
lim ln 1   ln 1    ......  ln  1  n  n  
je

= n n   n  1   n  2   
and lim b n  lim a sin  cos  / 2  a sin  .
n
iit

n  n
n  2 sin  / 2
n
 1  n  1  1 
@

where Tr = ln 1+  = ln  1+
a b2 2 n  n+r  n  1+( r n ) 
lim a n  lim b n 
⇒ n  n   a 2  b2 . 1 n  1 
tan 1   ∴ ln P = lim
n 
 ln 1  
n r 1  1  (r n ) 
 b 
 
1
π
1
 1   x+2 
Problem 18: Let x0 = 2 cos and xn = 2  x n 1 , n = 1,
= ∫ ln 1+ 1+ x  dx = ∫ ln  x+1  dx
6 0 0
( n 1)
2, 3, ........ find lim 2 · 2  xn . 1 1
n  = ∫ ln(x + 2)dx − ∫ ln(x +1)dx
π 0 0
Solution: Let = θ.
6 = (x + 2) ln (x + 2) − (x + 2)
1
0
 π θ
x1 = 2 + x0 = 2  2 cos   = 2 cos = 2 cos
6 12 2 – (x +1) ln (x +1) − (x +1) 1

0
  π θ
x2 = 2 + x1 = 2  2 cos   = 2cos = 2 cos
22 = [3 ln 3 – 3]– [2 ln 2 – 2]– [(2 ln 2– 2) – (0 – 1)]
 12  24
= [ln 27 – 3] – [ln 4 – 2] – [ln 4 – 1]
   π θ
x3 = 2 + x2 = 2  2 cos   = 2 cos = 2 cos = [ln 27 – 1] – [2 ln 4 – 1]
 24  48 23 = ln 27 – ln 16
27 27
∴ ln P = ln ⇒ P=
θ   16
xn = 2 cos ; 2 − xn = 2  2 cos = 2 sin 16
2n 2n 2n1 ∴ a = 27 ; b = 16 ⇒ a + b = 43.

FREE BOOKS FOR JEE & NEET =>(@iitjeeadv)


1.110 Differential Calculus for JEE Main and Advanced

Alternative:
2n 3  3n 2  n n 1  1 
3n   1
16 32  2n  1 
Cn (3n )! n !n ! =
2n
= · 24
Cn n !(2n )! (2n )!
n 3 n 2 5n 1  n 
 (3n )(3n  1)(3n  2).............  3n  (3n  1)   =   
12 8 48 16  2n  1 
=
.  n (n  1)(n  2).............  n  (n  1)  
n 3 n 2 5n f ( n )

1
= + + + (given)
. A B C D

[2n (2n − 1)(2n − 2)........2n − (2n − 1)] 2
So A = 12, B = 8, C = 48 ; D = 16
3n n n
=  
(3n ) (n ) 27 Hence, A + B + C + D = 84

( 2n ) 4n
 16  Problem 21: Evaluate
1n
 27  n  27 1  n 1n  2n  n 2n  3n  n 3n  4n  ......  n (m  1) n  m n 
∴ lim   
 
n   16  
= . lim lim  
  16 m  n  m2
 

Solution:
r4 n
Problem 20: Let  1  n 1n  2n  n 2n  3n  n 3n  4n  ......  n (m  1) n  m n 
r 1 ( 2r  1)( 2r  1)
 
lim lim  
m  n  m2
3 2  
ps
= n + n + 5n + f (n ) , (A, B, C, D,∈ N)
A B C D  n n n 
1 2  m 1 
el
 1  2 n    1  3n    1  ......  m n   1 
where f (n) is the ratio of two linear polynomials such that  2 3  m  
= lim  lim
eh

2 
m  n 
1  m 
lim f (n )  . Find the value of (A+B+C+D).  
je

n 2  
r4
iit

Solution: Tr = 1  2  3  4  ....  m
(2r  1)(2r  1) =lim
@

m  m2
1  16r 4  1  1    1 n n
=   2
16  (2r  1)(2r  1)      0 as n  ;    0 as n  ;
  2  3

1  (4r 2  1)(4r 2  1)  1 

n
Tr =    m 1 
16  4r 2  1 .....;    0 as n   
  m  
1  2 1 
1 1
=  (4r  1)   m(m 1)
= lim = 1 = 1.
2  m 
16  (2r  1)(2r  1)  lim
m  2m 2 m 
2
1  2 1  (2r  1)  (2r  1)    (1  [ x ])1/{x} 
1/{x}
=  (4r  1)    Problem 22: Evaluate lim  
16  2  (2r  1)(2r  1)   x 0  e
 
1  2 1 1 1  if it exist, where {x} denotes the fractional part of x.
=  4r  1    
16  2  (2r  1) (2r  1)   Solution: L.H.L. lim f(x)
= x 0
1/{0  h}
1 1 1  1 1   1  {0  h})1/{0  h} 
∴ Sn =
4
 r 2  16 1  32   2r  1  2r  1  = lim f(0 – h) = lim 
h 0 e

h→0  
n (n + 1)(2n + 1) n 1/(1 h )
= +  (1  1  h )1/(1 h )  2
4· 6 16 = lim   =  .
h 0
 e  e
1  1 1 1  1 1  lim f(0 + h)
  1        .....     R.H.L. = lim f(x) = h→0
32  3 3 5  2n  1 2n  1   x0 

FREE BOOKS FOR JEE & NEET =>(@iitjeeadv)


Limits 1.111

1/{h} 1/ h We choose x and y such that x + ny = u and x – ny = v


 (1  {h})1/{h}   (1  h )1/ h 
= lim   = lim   where u, v ∈ R and n ∈ N.
h 0  e h 0  e
    2
(3) becomes, |f(u) – f(v)| ≤ for arbitrary n ∈ N
=
1
lim
h →0
((1  h )1/ h  e) 3n
ee 2
h i.e., as n → ∞ ⇒ |f(u) – f(v)| ≤ lim
n  3n
1  e(1/ h )ln(1 h )  e 
=
ee
lim
h →0   ⇒ |f(u) – f(v)| ≤ 0 ⇒ f(u) = f(v)
 h  Hence, f is constant.
 ln(1 h )  h  Problem 24: The function un takes on the following
lim e h 1
= e h →0   1 1 1
 h  values : u1 = , u2 = + .....,
  4 4 10
ln( 1+ h ) 1 1 1
(e h −1)  ln(1+ h ) − h  un = + 2 + ........ n ....
= lim
h →0
 ln(1+ h ) − h  
.
h2  ...(1) 3 +1 3 +1 3 +1
e  

h 1
Prove that lim u n  .
ln(1  h )  h n  2
Now let P = lim ...(2) 1 1 1 1 1 1
Solution: We have < ;    ; .....

h 0 h2
replacing h by –h then 4 3 4 10 3 9
1 1 
n
ln(1  h )  h 1    
ps
P = lim ...(3) 1 1 1 3   3  
h2 Thus, un < + 2 + ...... n < 

h 0
el
3 3 3 1
adding (2) and (3), 1
eh

3
ln(1  h 2 )  1
n   1 n 
ln(1  h 2 ) 3 1 1    
1  3  .
je

2P = lim = – lim = –1 un < <


   3 
  
h 0 h2 h 0 (h 2 ) 2 2  
iit

1   1 n 
1     = 1 .
@

∴ P=– from (1), lim u < lim


n 2  
n n
 3 
  
2
R.H.L. = e1.(–1/2) = e–1/2.
e x sin( 2007 ) x  1
L.H.L. ≠ R.H.L. Hence P does not exist. Problem 25: Let L = lim and
x 0 x ln(1  x )
Problem 23: Suppose that function f : R → R satisfies

the inequality,
n
 3 {f (x  ky)  f (x  ky)} ≤ 1
k

M = lim x  2 x 
x   3
x 3  x 2  1  3 x 3  x 2  1 
 
k 1 then find the value of LM.
for every positive integer n and for all x, y ∈ R.
e x sin( 2007 ) x  1 x sin( 2007 x )
Prove that f is a constant function. Solution: L = lim ·
x 0 x sin( 2007 x ) x ln(1  x )
Solution: Replacing n by (n – 1) in,
sin(2007 x ) (2007 x )
n = lim · = 2007.
 3 {f (x  ky)  f (x  ky)}
k
≤1 ...(1) x0 (2007 x ) ln(1  x )
k 1
lim  x [( x 3  x 2  1)1 3  ( x 3  x 2  1)1 3  2 x ]
M = x
we get:
n 1   13 13 
1 1   1 1 
 3k {f (x  ky)  f (x  ky)} ≤1 ...(2) = lim  x  x 1 
x  

x x 3 
 x 1   3 
 x x 
 2x 

k 1
Subtracting (1) and (2) we get, 1
Put x =

|3n {f (x + ny) – f(x – ny)}| ≤ 2 t
2 [(1  t  t 3 )1/ 3  (1  t  t 3 )1/ 3  2]
⇒ |{f (x + ny) – f(x – ny)}| ≤ ...(3) = lim
n
3 t 0 t2

FREE BOOKS FOR JEE & NEET =>(@iitjeeadv)


1.112 Differential Calculus for JEE Main and Advanced

1  1 x x
= lim  1  ( t  t )  ( t  t )
3 3 2
Problem 27: Let f (x) = tan secx + tan .
t 0 t2  3 2 22
1 1  x x x x x
1 ( t  t 3 )  ( t  t 3 )2  2  sec + tan sec + ..... + tan sec
3 9  2 23 22 2n 2n −1
2 2   x
t  ...... and g (x) = f (x) + tan where x   ,  and
=
lim 9 2 2  2 2
n
t 0 t n N. Evaluate the following limits:
2 1 1
=
9  g( x )  x (b) lim  g ( x )  x
2
2 (a)
∴ LM = × 2007 = 2 × 223 = 446. lim 
x0  x 
 
x0  x 

9
1
  g( x )  x 3

Problem 26: Let an = 2 cos n


 1 then show that lim (c) lim  
2 n x0  x 
2 cos   1  x
(a1a2a3....an–1an) = , θ ∈ R. sin  x   x
3 sin
 2 2
Solution: Consider an · a n–1 · a n–2 ............a2 · a 1 Solution: T1 = x = x
cos cos x cos cos x
     
ps
2 2
=  2 cos n  1  2 cos n1  1
 2   2  x
el
= tanx – tan

     2
eh

..........  2 cos  1  2 cos  1 x


   x

2 2
2
je

Similarly, T2 = tan – tan 2


       2 2
iit

=  2 cos  1  2 cos n  1  2 cos n 1  1 x


 2n  2  2  x
T3 = tan – tan
@

22

1 23
       
.......  2 cos 2  1  2 cos  1   

 2  2   2 cos n  1 x x
 2  Tn = tan n −1 –


    2 2n
Now  2 cos n  1  2 cos n  1
_________________________
 2  2 
x
f (x) = tanx – tan
    2n
= 4 cos n  1 = 2  1  cos n 1   1
2
x
2  2  ⇒ f (x) + tan = g (x) = tan x
2n

= 2 cos +1 1

Now (a) L = lim  tan x 


n1
2 x
x0  x 
     
Similarly,  2 cos n 1  1  2 cos n1  1
 2   2  1  tan x 
x
tan x  x
Limit
= x  x 
= x 0
x2
e

e
 2     
=  4 cos n 1  – 1 = 2  cos n  2  1 – 1
 2   2  tan x  x
Consider l = lim

x 0 x2
2 cos 1
2n 2 tan x  x
= = lim lim x  0 ⇒ L = e0 = 1.
x3

x 0 x 0
n
(2 cos   1)
Hence  ai      tan x 
1/ x 2
 tan x  1
i 1  1 2
 2 cos n  1 x 0 
(b) = lim
lim    x x
 2  x0  x  e
n
2 cos   1
∴ lim
n 
 ai  3
. = limit
x→ 0
tan x − x
x3
= e1/3.

i 1 e

FREE BOOKS FOR JEE & NEET =>(@iitjeeadv)


Limits 1.113

1 4  3a n 1 4  3a n
 tan x  x 3

= el ∴ an +2 – an +1 = 
(c) lim  3  2a n 1 3  2a n
x0  x 
(4  3a n 1 )(3  2a n )  (4  3a n )(3  2a n 1 )
1  tan x  =
1 
where l = limit
x 0
x 3  x  (3  2a n 1 )(3  2a n )

tan x  x tan x  x 1 a n 1  a n
= lim = (3  2a )(3  2a )  0
lim
= x . = dne.
4 x3 x {∵ an + 1 > an}

x 0 x 0
n 1 n
⇒ limit does not exist.
∴ an+2 – an+1 > 0
n
x an+2 > an + 1 whenever an + 1 > an
Problem 28: Let f (x) = lim
n 
 3n 1 sin3 3n and ∴ The sequence of values an is increasing and since
n 1 a1 = 1, an > 0 for all n,
g (x) = x – 4 f (x). Evaluate lim 1  g ( x ) 
cot x
. Now let, l = nlim a n  lim a n 1
x 0  n 

Solution: Using sin 3θ = 3 sinθ – 4 sin3θ 4  3a n


lim a n 1  lim
n  n  3  2a n
x 1  x 
T1 = sin3 = 3 sin  sin x  4  3l
3 4  3  ⇒ lim
l = n
3  2l
x 3  x x ⇒ 3l + 2l2 = 4 + 3l
T2 = 3sin3 2 = 3 sin 2  sin 
3 4  3 3 l2 = 2
ps
and so on........ ∴ l= .
el

1 x  Problem 30: Find the values of a, b and c so that
∴ 3 sin  sin x 
eh

4  ae x  b cos x  ce  x
T1 =
3  lim 2
x 0 x sin x
je

1 2 x x
3 sin 2  3 sin  x
4  Solution: We have lim ae  b cos x  ce  2 ...(1)
x
T2 =
iit

3 3
x 0 x sin x
@

1 n x n 1 x  As x → 0, numerator → a – b + c
Tn =  3 sin n  3 sin n 1  and denominator → 0
4 3 3 
Since limit is finite a – b + c = 0
1 n x  or b = a + c ...(2)
∴ 3 sin n  sin x 
4 
f (x) = lim

From (1) and (2)
n  3 
ae x  (a  c) cos x  ce  x
 x  lim 2
x sin n
1  x 0 x sin x
=  lim
3  sin x  = 1 (x – sin x)
4  n  x  4 ...(3)
  ⇒
 3n 
a (e x  1) c(e  x  1)
g (x) = x – 4 f (x) = sin x 
x (  x ) (a  c) sin 2 x
⇒ lim  lim 2
Now lim (1  sin x )
cot x lim (cot x )(sin x )
= e x→0 =e x 0 sin x x  0 (1  cos x ) x sin x
x 0
4 + 3a n a (e x  1) c(e  x  1)
Problem 29: If an = 1 and an + 1 = , n ≥ 1, 
x (a  c) sin 2 x ...(4)
3 + 2a n ⇒ lim x  lim 2

x 0 sin x x 0 (1  cos x ) x
then show that an+2 > an + 1 and if an has a limit l as n → ∞,
Since limit is finite, as x → 0,
then evaluate nlim an .
 a (e x − 1) c(e − x − 1)
− = a – c must be 0
Solution: a1 = 1 x (− x )
43 7 ∴ a–c=0 ⇒a=c ...(5)
and a2 =  1

3 2 5 x
a (e  e  2)
x
2a sin x
∴ a2 > a1 lim  lim 2
x 0 x sin x x  0 (1  cos x ) x
Assuming an +1 > an

FREE BOOKS FOR JEE & NEET =>(@iitjeeadv)


1.114 Differential Calculus for JEE Main and Advanced

From (4), c = 1 and from (2), b = 1 + 1 = 2.


⇒ a (e x  1) 2 2a sin x
lim  lim 2
x 0 e x x sin x x 0 (1  cos x ) x Problem 31: Let a1, a2, ....., an be sequence of real
 ex  1 
2 numbers with an + 1 = an + 1 + a 2n and a0 = 0. Prove that
a    a  4
x  lim  n  
⇒ lim   lim
2a sin x
2 n   2 n 1  
x 0 x  sin x  x 0 (1  cos x ) x .
e . 
 x  Solution: Here, an + 1 = an + 1 + a 2n ,
a (1) 2 2a.1 let an = cot (αn) ⇒ an + 1 = cot(αn) + cosec(αn) ⇒ an + 1
⇒  2
1.1 2
∴ a=1 cos( n )  1 2 cos 2 ( n / 2)  
  cot  n  = 1
From (5), c = 1 sin( n ) 2 sin( n / 2) cos( n / 2)  2 
And from (2), b = 1 + 1 = 2
Alternative : Putting n = 1, a1 = cot (α1) and a1 = a0 +
Here we use L’ Hospital’s rule
ae x  b cos x  ce  x = 1 + a 02 = 1
We have lim 2 ...(1) π
x 0

x sin x
As x → 0, numerator → a – b + c and ⇒ cot(α1) = 1 or α1 = 4
denominator → 0 ∴ a–b+c=0   
⇒ b=a+c ...(2) Again, a2 = cot  1  = cot  
 2  8

ps
From (1) and (2)
ae x  (a  c) cos x  ce  x  0     
a3 = cot  2  = cot  
el
lim  2  form   4.22 
x 0 x sin x  0  2 
eh

Using L’ Hospital’s rule     


a4 = cot  3  = cot   .........
ae x  (a  c) sin x  ce  x
je

 2   4.23 
lim 2 ...(3)
( x cos x  sin x )   
iit

x 0
As x → 0, numerator → a – b + c an cot  .
 4.2n 1 
@

and denominator → 0
Since limit is finite a – c = 0 Put =x
⇒ a=c ...(4)
  

From (3) and (4) cot  n 1 
 an 
ae x  2a sin x  ae  x  0 Hence, lim   = lim  4.2 
2 n   2 n 1 
lim
x cos x  sin x  form 0  n  2n 1
x 0  
Again using L’ Hospital’s rule
ae x  2a cos x  ae  x =
lim 2
x 0  x sin x  cos x  cos x

a  2a  a  a  4
⇒ 2 ∴ lim  nn1   .
0 11 n  2  
∴ a=1

Things to Remember
1. Theorems on Limits (iv) xlim [f(x) . g(x)] = m
Let f and g be two functions such that →a
lim f(x) = , lim g(x) = m. Then
x →a x →a (v) lim  [f(x) / g(x)] =  / m, (m ≠ 0)
x a
(i) lim k(f(x)) = k., (vi) For any positive integer n,
x →a

(ii) xlim
→a
[f(x) + g(x)] =  + m lim n f ( x ) = n lim f ( x ), provided lim f ( x ) > 0 when
x →a x →c
x →a
(iii) lim [f(x) – g(x)] =  – m n is even.
x →a

FREE BOOKS FOR JEE & NEET =>(@iitjeeadv)


Limits 1.115

 
m/n 5.
Also lim  f ( x ) m / n  lim f ( x ) where m/n is a rational lim sin x = 1 = lim tan x
x a x a (i) x→0
x→0
number and the limit on the right exists. x x
(vii) If lim f ( x )  b and g(x) is continuous at x = b, then −1
lim tan x = sin −1 x
x a

 
= x→0
lim g (f ( x ))  g lim f ( x ) = g(b).
x x
x a x a
a −1
x
(ii) = ln a (a > 0).
(viii) If , with f(x)¹ b for every x in some x
neighbourhood of 'a' and if lim g ( x )  c , then lim ln(1+ x ) = 1
x b (iii) x→0
lim g  f ( x )   c . (This theorem is sometimes useful if g is x
x a log a (1+ x )
discontinuous at b. lim = loga e, ( a > 0, a ≠ 1)
x→0 x
(ix) Domination Law : If two function f(x) and g(x) satisfy
the inequality f(x) < g(x) for all the values of x belonging (1  x ) n  1
(iv) lim =n
to a neighbourhood of a point a except possibly a then x 0 x
lim f ( x )  lim g ( x ) provided that the limits of
lim x  a
n n
x a x a
 n a n 1 .
both functions, as x → a, exist. x →a
x a
2. For all elementary functions, limit at any point in the
(v) lim xnx  0
domain is equal to the function's value at that point. psx 0
3. The important indeterminate forms are : 1 x
∞ (vi) l im (1  x ) x  l im 1 
 1
0
  e
∞ , 0 × ∞, ∞ − ∞, 0º, ∞º and 1 x 
, ∞
el
0 x 0 x  
eh

4. Following are some of the frequently used series


expansions : a 0 x m  a1x m 1  ....
6. l im
je

(i) a x  1 
x 1n a x 2 1n 2 a x 3 1n 3a
   .........a  0
x  b0 x n  b1x n 1  ...
iit

1! 2! 3!
 a0
 b , when m  n
@

x x 2 x3
(ii) e  1  
x
  ............  0
1! 2 ! 3!
 0, when m  n
x 2 x3 x 4 
(iii) ln (1+x) = x     .........for  1  x  1 =  a0
 , when m  n and 0
2 3 4
b0
x3 x5 x7 
(iv) sin x  x     .......  a0
 , when m  n 0
3! 5 ! 7 ! and
x2 x4 x6  b0
(v) cos x  1     ......
2! 4! 6! sin x cos x
x 3 2x 5 7. (i) lim  lim 0
(vi) tan x = x + + + ........ x  x x  x
3 15 (ii) In fact, if n is a positive integer then
x3 x5 x7 ex
(vii) tan x = x 
–1
   ....... lim n   , lim x n e x  0 .
3 5 7 x 

x  x

12 3 12.32 5 12.32.52 7 (iii) For sufficiently large values of x we have


(viii)sin-1x = x + 3! x + 5! x + 7 ! x + ....... logax << xp << ax << x! where

a>1 p > 0 a > 1 x ∈ N.

x 2 5x 4 61x 6
(ix) sec x = 1 + + + + ...... (iv) lim log a x = 0 , for a > 1.
2! 4! 6! x
x
nx x2 n a = 0 for a > 0,
(x) (1 + x)n = 1 +  n (n  1) +.... for –1< x< 1 (v) lim
1 2 n  a n

 1 11  n
(xi) (1 + x)1/x = e 1  x  x 2  .....  (vi) lim a  0
 2 24  n  n !

FREE BOOKS FOR JEE & NEET =>(@iitjeeadv)


1.116 Differential Calculus for JEE Main and Advanced

8. The line x = a is called a vertical asymptote of the curve containing a, except possibly at a.
y = f(x) if atleast one of the following statements is true: Suppose that
lim f(x) = ∞ lim f(x) = ∞ lim f(x) = ∞ lim f(x) = 0 and lim g(x) = 0
x →a x →a x a 


x →a x →a
lim f(x) = – ∞ lim f(x) = – ∞ lim f(x) = – ∞ or that f(x) = ± ∞ and lim g(x) = ± ∞.


x →a x →a x a 
x →a
9. The line y = L is called a horizontal asymptote of the (In other words, we have an indeterminate form of type
curve y = f(x) if either
0 ∞
lim f(x) = L or lim f(x) = L or ).
x 
x 0 ∞
 0, 0  a  1 Then lim
f (x)
= lim
f '( x )
, provided the limit on
 1, a 1 x →a
g( x ) x →a g '( x )
10. lim a x  
 the right side exists or is ∞ or – ∞.
x 
 , a 1 15. Leibnitz’s formula for differentiation of integrals
dne a0 d 
v( x ) 
  f ( t )dt  = f(v(x)) dv – f(u(x)) dv .
11. If lim f ( x ) = A > 0 and lim f ( x ) = B, a finite quantity dx  u ( x )  du dx
x→a x→a  
then lim  f ( x ) 
g (x)
A . B
 0, if −1 < x < 1
xa 2n 
16. lim x  1, if x = ±1
Also, lim  f ( x ) 
g (x) lim g ( x ) n ( f ( x )) n 
e x a
∞, if x < −1 or, x > 1
xa 
ps
12. Let lim f (x) = 1 and lim g(x) = ∞ then 1 (x)  r 
b
x →a x →a 17. lim  f     f (x)dx
el
r  ( x )  n  a
n  n
lim (f (x) ) g (x) f ( x ) − 1
g (x) lim
=e
eh

( x ) (x )
x →a

x →a
where, a = lim and b = lim .
13. Sandwich Theorem : Suppose that g(x) ≤ f(x) ≤ h(x) for n  n  n
je

n
all x in some open interval containing c, except possibly
iit

at x = c itself.  f (x) g(x ) 


18. Let A =   then we have
If lim g(x) = lim h(c) = l, then lim f(x) = l.  u(x) v( x ) 
@

x →c x →c x →c
14. L'Hospital's rule: Suppose f and g are differentiable  lim f ( x ) lim g ( x ) 
lim A   
x a x a
on an open interval containing a, except possibly at a
itself. Assume that g′(x) ≠ 0 for all x in the open interval x a  lim u ( x ) lim v( x ) 
 x a x a 

Objective Exercises
Single Correct Answer Type

 sin x  (B) 1
1. The value of lim   , (where [.] denotes greatest (C) a cannot be determined
x 0  x 
  (D) none of these
integer function) is
esin h − (1 + sin h )
(A) 0 (B) does not exists lim
(tan (sin h ))
4. h →0 −1
2 is
(C) –1 (D) 1
 x2 
2. The value of lim   , (where [.] denotes the (A) 1 (B) 2
x 0  sin x tan x 
  (C)
1
(D) none of these
greatest integer function) 2
(A) 0 (B) 1
  · (1 + a
x b
(C) does not exist (D) none of these 5. lim 1  a 2 2 x is (a, b R)
x  )
3. If lim ( x 4  x 2  1  ax 2  a ) exists then a is equal to (A) √b (B) b
x 
(A) –1 (C) b2 (D) none of these

FREE BOOKS FOR JEE & NEET =>(@iitjeeadv)


Limits 1.117

 2x 2  1  2  tan x  sin x   x 3
6. lim x 2  tan 1 2  tan 1 2  is lim
14. x→0 lim is equal to
x   x 2  x0 x5
3 3 (A) 1/4 (B) 1/2
(A) (B) − (C) 1/3 (D) None of these
5 5
5 5
lim 1  cos x  2 sin x  sin x  x  3x equals
3 2 4
(C) (D) −
3 3 15. x→0
7. If f(x) = 0 be a quadratic equation such that tan 3 x  6 sin 2 x  x  5x 3
 32 f (x) (A) 1 (B) 2
f(–π)= f(π) = 0 and f     , then lim (C) 3 (D) 4

2 4 x sin(sin x)
is equal to lim log cos x
16. x→0 is equal to
(A) 0 (B) π p
cos x − cos x
q
(C) 2π (D) None of these
pq q−p
(A) (B)
20
q+p pq
8. lim  cos 2 n ( x  10) is equal to
n 
x 1 pq pq
(A) 0 (B) 1 (C) (D)
q−p p−q
(C) 19 (D) 20
 3x  17. The limit
1  sin     1  1  1   1 
1  5   1  2   1  4  ....  1  2 
9.  1  x 2  is equal to lim
lim n    5  5  
n

1  cos x 5 
ps
x 1
is equal to
(A) 0 (B) 1
el
(A) 0 (B) 5/4
(C) 2 (D) None of these
(C) 4/5 (D) 1/5
eh

10. If f : (1, 2) → R satisfies the inequality 1/ x


 xn ( 2x 1)
 (2x  1) x sin x 
cos(2 x  4)  33 x | 4x  8 | lim  e
je

2
 f (x)  , ∀ x ∈ (1, 2). 18. The value of x→0   is
x2  e xnx 
iit

2
equal to
Then lim f ( x ) is
@

x  2 (A) e (B) 1 n2


(A) 16 e
(B) cannot be determined from the given information (C) en2 (D) None of these
1/ n
(C) –16  n! 
(D) does not exist 19. lim   (m ∈ N) is equal to
n  n
 2  (mn ) 
 
11. lim (1  x ) x  (where {x} denotes the fractional part of (A) 1/em (B) m/e
x 0

 
 (C) em (D) e/m
x) is equal to    ay     by   
 exp  xn 1     exp  xn 1  
(A) e2 – 7 (B) e2 – 8   x    x   
lim  lim
20. y→0
 x 
2
(C) e – 6 (D) None of these y
12. The value of (A) a + b (B) a – b
 3 1 1  (C) b – a (D) – (a + b)
 x  4x   x  2x 2 
lim        is
x 2   x 3  8 
 x2 x  2  
3
1  tan 1 3x  3 1  sin 1 3x
   lim
21. x→0 is equal to
(A) 1/2 (B) 2 1  sin 1 2 x  1  tan 1 2 x
(C) 1 (D) None of these (A) 1 (B) –1
 
(C) 2 (D) None
ax
13. xlim
a  a 2 − x 2 cot  2  is equal to (n (1  x )  n 2)(3.4 x 1  3x )
 ax  22. lim equals
x→1
[(7  x )1/ 3  (1  3x )1/ 2 ].sin( x  1)
a 2a
(A) (B) 9 4 9 e
π π (A) n (B) n
4 e 4 4
a 4a
(C) − (D)
π π (C) 4 n e (D) None of these
9 4

FREE BOOKS FOR JEE & NEET =>(@iitjeeadv)


1.118 Differential Calculus for JEE Main and Advanced

 1   x 3x 
2m / x
x 5 tan  2   3 | x |2 7 lim  sin  cos 
32. The value of the limit x→0 is
23. lim  x  is equal to  m m
x
| x |3 7 | x | 8 (A) 1 (B) 2
1 (C) e6m (D) ln 6m
(A) π  
lim  x  x 2  3x cos 1  is equal to

(B)
π 33. x
(C) –
1
(D) None of these 
 |x| 
π (A) 3/2 (B) –3/2

lim
  
cos 2 1  cos 2 1  cos 2 .........cos 2 ( x )  (C) –1 (D) none of these
34. lim x + x 2 + x 2 sin(1 / x ) is equal to
24. x→0 x
  x  4  2 
sin     
(A) 0 (B) 2
  x   (C) –2 (D) none of these
(A) 2 (B) 2 x 2 n sin n x
lim
35. If x→0 is a non zero finite number, then
(C) – 2 (D) None of these x − sin 2 n x
2n

 a x  ax  1  
n must be equal to
25. If lim 2 2
a2x2 1 (A) 1 (B) 2
x
(C) 3 (D) none of these
  36. If b < 0, b ≠ –1 and a is a positive constant then
lim  x  x  x  x
= K. x   then the value of K is ax
lim equals
(A) 1 (B) a x
| x |  b2 x 2  x
ps
(C) 2a (D) None of these 1 1
el
lim (x–4
26. The value of x (n x)10) is (A) (B)
| b | −1 −b − 1
eh

(A) 1 (B) 0 1 1
(C) (D)
1 1 b −1
je

1− | b |
(C) (D) –
2 2 1/ n
m 
iit

x −1 37. lim   r n  is equal to, (n ∈ N)


tan( x − 1).log e x n   
27. lim  r 1 
@

is equal to
| x − 1 |3
x→1
(A) m (B) m/2
(A) 1 (B) – 1 (C) em (D) em/2
(C) 3 (D) None of these 38. If k is an integer such that
lim 1  1  1  1  + x n 1  a1/ x , a > 0, a ≠ 1    kπ  
n n
kπ  
28. x→0 lim   cos  −  cos   = 0, then
x  x 2 x  n   4 6 
(A) a (B) 1 (A) k is divisible neither by 4 nor by 6
(C) 1 + a (D) None of these (B) k must be divisible by 12, but not necessarily by 24
1
 n 4n (1) n  (C) k must be divisible by 24
29. lim    is equal to (D) either k is divisible by 24 or k is divisible neither by
n  2n  1 
 n 2
2
4 nor by 6
(A) 2 (B) 1 sin x
lim
(C) 0 (D) None of these 39. x  1 1  where [.] denotes greatest
2 cos  4 (3 sin x  sin 3x ) 
x ; x  0  
30. If f(x) =  and g(x) = f(x) + |x|. Then integer function, is equal to
 x ; x  0 2
lim (log (A) (B) 1
x)g(x) is
|sin x|
π
x0
(A) 0 (B) 1 4
(C) (D) does not exist
(C) 1/2 (D) not exist π
cot 1 ( x  a log a x )
31. The value of the limit lim n2
n
 n

a  n 1 a (a > 0) is 40. The value of lim
x  sec 1 (a x log x a )
(a > 1) is equal to

(A) n a (B) ea (A) 1 (B) 0


(C) e–a (D) none of these (C) π/2 (D) does not exist

FREE BOOKS FOR JEE & NEET =>(@iitjeeadv)


Limits 1.119

n 49. Let P(x) = a1x + a2x2 + a3x3 + ...... + a100x100, where a1 =


n pnq 
41. lim   , p, q > 0 is equal to 1 and ai ∈ R i = 2, 3, 4, ...., 100.
n   2  100 1  P ( x )  1
  then lim has the value equal to
(A) 1 (B) pq x0 x
pq 1
(C) pq (D) (A) 100 (B)
2 100
(C) 1 (D) 5050
cot 1 ( x  1  x )
lim 50. Assume that lim1
f(θ) exists and
42. x    2 x  1  x  is equal to
sec 1    f () 2  2  1
  x  1   2    2
 2 
  3  3
(A) 1 (B) 0
π holds for certain interval containing the point
(C) (D) non-existent θ = –1 then lim f(θ)
2 1
(tan({x}  1)) sin{x} (A) is equal to f(–1) (B) is equal to 1
43. The value of lim , where { x }
x0 {x} ({x}  1) (C) is non-existent (D) is equal to –1
denotes the fractional part function, is 6 x 2 (cot x )(cos ec2 x )
(A) is 1 (B) is tan 1 lim
51. x0      has the value equal to
(C) is sin 1 (D) is non-existent sec  cos x   tan    1
  4 sec x  
lim {x}
44. x→0 (n ({x} + |[x]|)) is equal to

(A) 6 (B) –6
ps
(A) 0 (B) 1 (C) 0 (D) –3
1 52. Suppose that a and b are real positive numbers then the
(C) ln 2 (D) ln
el
2 1/ t
 b t 1  a t 1 
eh

where [ ] is the greatest integer function and { } is the value of lim   has the value equals to
t 0 
fractional part function.  ba 
je

  100 x   99 sin x  
45. The value of  lim     is (where [.] (A) a ln b − b ln a (B) b ln b − a ln a
iit

 x  0  sin x   x   b−a b−a


@

1
denotes greatest integer function)
(D)  b  b a
b
(A) 199 (B) 198 (C) b ln b - a ln a
 a 
(C) 197 (D) None of these a 
 2x   1 x 2 
n
 1 
46. The value of lim
n 
 1  22 n is

53. If f(x) = cot–1  2 
 1 x 
and g(x) = cos–1 
 1 x
2  , then

n 0
(A) 1 (B) 2 f ( x )  f (a )  1
lim , where  0  a   is
(C) 4 (D) none x  a g ( x )  g (a )

 2
47. The value of (A) 1 (B) –1
lim
16  26  36....... n 6 (C) 2 (D) 1
n

(12  22  32.......n 2 ) 13  23  33  .......n
n3  2
n (1  x  x )  n (1  x  x 2 )
2
54. lim is equal to
(A) 14 (B) 21 x 0 sec x  cos x
7 8 (A) 1 (B) – 1
(C) 132 (D) 12 (C) 0 (D) ∞
17 7 2x
48. Let a, b, c are non zero constant number then 55. If f(x)= nlim
 
tan–1 nx, then value of lim [f(x) – 1] is,
x→0
a b c where [.] represents greatest integer function
cos  cos cos
lim r r r (A) 0 (B) –1
r b c equals (C) 1 (D) does not exist
sin sin 1 cos( x 1)
r r  3 
56. The value of lim  x  2 x  x  1 
2 ( x 1) 2
a b c
2 2 2 c a b
2 2 2
is
(A) (B) x→1  2
  
2bc 2bc  x 2 x 3 
(A) e (B) e1/2
(C) b  c  a
2 2 2
(D) independent of a,b,c (C) 1 (D) none of these
2bc

FREE BOOKS FOR JEE & NEET =>(@iitjeeadv)


1.120 Differential Calculus for JEE Main and Advanced

x2
1 lim e
tan x
− ef {f [ f ( x )]}
∫ sin x
2
57. If f(x) = , x→0 is equal to 66. If lim dx is a non zero definite number, then
1− x tan x − f {f [f ( x )]} x→0 0
(A) 0 (B) 1 xn
(C) – 1 (D) none of these value of n is

7 29 133 5n  2 n 
 (A) 1 (B) 3
58. lim   2  3  .....   equals
10 10
n  10 10n 
 (C) 5 (D) 4
(A) 3/4 (B) 2 67. If a, b and c are real numbers then the value of
(C) 5/4 (D) 1/2 1 t 
lim ln   (1  a sin bx )c x dx  equals
 
1  t 
2
x t 0
t0
2

59. lim  tan 
  dt is equal to
 1 t
2
x→0
x  ab
(A) abc (B)
sin 2x c
(A) 0 (B) 1 bc ca
(C) 1/2 (D) does not exist (C) (D)
a b

r 3  (r 2  1) 2 68. If C0, C1, C2, .........Cn are binomial coefficients then lim
60.  (r 4  r 2  1)(r 2  r) is equal to n
r 1  2 n 
2 2 2
(A) 3/2 (B) 1  Cn    c n 1    Cn  2  ......( 1) n   C0  is
 3 3 3 
(C) 2 (D) infinite
(A) 0 (B) 1
ps
729 x  243x  81x  9 x  3x  1
61. If lim = k(ln 3)3, then k (C) –1 (D) 2
3
x  sin [ x ]  2 x 
el
x
is equal to  if [ x ]  0
69. If f(x)=  [x] , where [.] denotes the
eh

(A) 4 (B) 5  if [ x ]  0
 0
(C) 6 (D) none
je

greatest integer function, then lim f(x) is


sin x − (sin x )sin x x→0
iit

62. lim is equal to


x / 2 1 − sin x ln sin x (A) 0 (B) 1
@

(A) 1 (B) zero (C) − 1 (D) none of these


(C) 2 (D) 2/3 1
70. Let f(x) = lim 2n
. Then the set of
n 
3 1 
xn x
5
63. lim x 2  is equal to   tan 2 x 
 
x 
n 0 n ! values of x for which f(x) = 0, is
(A) 1 (B) e
(A) |2x| > 3 (B) | (2 x ) |< 3
(C) 2e–1 (D) 0
64. If {tn} be a sequence such that tn = , S denote the
2n (C) |2x| ≥ 3 (D) |2x| ≤ 3
3n + 1 n 2 n 4 1
sum of the first n terms and
n !  5n 5 1
n Sn 1  Sn 71. The value of lim  n  is equal to
  lim , then n   n 
n  2 n
k (A) 1
2/5
(B) 0
k 1 1
n 1 (C)   (D) e2/5
  lim   2  3  .......  (n  1)
2 2
equals e
n 
(A) 18 (B) 9 1
72. The value of lim ([12x + 12] + [22x + 22] + [22x + 22]
(C) 3 (D) 6 n3n 
n + ....... + [n2x + n2]), is (where [.] denotes the greatest
1.n  (n  1)(1  2)(n  2)(1  2  3)  ..1. r integer function)
r 1
65. lim x 1
n  n4 (A) (B) x +
is equal to 3 3
x 1
(A) 1/12 (B) 1/24 (C) + (D) none of these
(C) 1/6 (D) 1/48 3 3

FREE BOOKS FOR JEE & NEET =>(@iitjeeadv)


Limits 1.121

73. The value of (A) 0 (B) 1


(C) 2 (D) 3
 1 1 1 1 
lim     ........  75. A0 is an equilateral triangle of unit area, A0 is divided into
nb 
, is
n  na na  1 na  2 four equal parts, each an equilateral triangle, by joining
the mid points of the sides of A0. The central triangle is
a b
(A) log   (B) log   removed. Treating the remaining three triangles in the
b a same way of division as was done to A0, and this process
(C) log (ab) (D) none of these is repeated n times. The sum of the area of the triangles
74. Let f(x) be defined for all x ∈ R such that removed in Sn then lim Sn is
n
  1   (A) 1/2 (B) 1
lim f ( x )  ln 1  f ( x )   ln(f ( x ))   0 ,
x 0   e   (C) –1 (D) 2
then f(0) is

Multiple Correct Answer Type for JEE Advanced

76. Which of the following limits vanish? 80. The true statement(s) is / are
1
lim x 4 (A) If lim f(x) = 0, then there must exist a number d
x x →c
1 such that f(d) < 0.001.
(A) sin lim
ps
x x  /2 (B) lim f(x) = L, is equivalent to lim (f(x) – L) = 0.
x →c x →c
(B) (1 − sin x) . tan x
el

2x  3
2 (C) lim (f(x) + g(x)) may exist even if the limits lim
eh

(C) lim . sgn (x) x →a x →a


x x2  x  5 (f(x) and lim (g(x) do not exist.
je

x →a
[ x ]2 − 9
(D) lim
iit

(D) If lim f(x) exists and lim (f(x) + g(x)) does not exist,
x3 x2 − 9 x →a x →a
@

77. The true statement(s) is / are then lim g(x) does not exist.
(A) If f(x) < g(x) for all x ≠ a, then lim f(x) < lim g(x). x →a
x →a x →a 81. Which of the following functions have a graph which lies
(B) If lim f(x) = 0 and |g(x)| ≤ M for a fixed number M between the graphs of y = |x| and
x →c
and all x ≠ c, then lim f(x)· g(x) = 0. y = – |x| and have a limiting value as x → 0.
x →c (A) y = x cos x (B) y = |x| sin x
(C) If lim f(x) = L, then lim |f(x)| = |L| and conversely 1 1
x →c x →c (C) y = x cos (D) y = x sin
if lim |f(x)| = |L| then lim f(x) = L. x x
x →c x →c
(D) If f(x) = g(x) for all real number other then x = 0 82. The false statement(s) is / are
and lim f ( x ) = L, then lim g ( x ) = L. (A) If P(x) is a polynomial, then the function f(x) = P(x)
x →0 x →0 x −1
78. Which of the following functions has two horizontal has a vertical asymptote at x = 1.
asymptotes (B) A polynomial function has no vertical asymptote and
x 2x
a rational function has atleast one vertical asymptote.
(A) y = (B) y = (C) If f(x) has a vertical asymptote at x = 0, then f is
x +1 x2 +1
undefined at x = 0.
sin x
(C) y  (D) y = cot–1(2x + 1) (D) A function can have move than two horizontal
x2 1 asymptotes.
79. Which of the following functions has a vertical asymptote 83. The function(s) which have a limit as n → ∞
at x = – 1. 2 2
| x2 −1 | x 2 − 6x − 7  n 1   n 1 
(A)   (B) (1) n  
(A) y =
x +1
(B) y =
x +1  n 1   n 1 

x2 +1 sin( x + 2) n2 +1 n2 1
(C) y = (D) y = (C) (D) (1) n
x +1 x +1 n n

FREE BOOKS FOR JEE & NEET =>(@iitjeeadv)


1.122 Differential Calculus for JEE Main and Advanced

84. The function(s) which have a limit as x → ∞ 1


2 x 2 n sin x
sin xπ x
(A) (B) a cos2xπ + b sin2xπ 89. Let f(x) = lim then which of the following
x n  1 x 2n

(C) x sin xπ (D) tan xπ alternative(s) is/are correct ?


(A) lim x f(x) = 2
85. Which of the following limits exist? (where [.] indicates x
greatest integer function all throughout) (B) lim f(x) does not exist
1n
sin[ x ]  en  x→1
(A) lim (B) lim   (C) lim f(x) does not exist
x→1 [x] n 
   x→0
(D) lim f(x) is equal to zero.
(C) lim sin(sin 1
x )  (D) lim sin 1 (sin x )  x
  x  2 x
x1
 ax  1 
    90. Consider the function f (x) =   where
86. Given l1 = lim cos–1 sec  x    ;  bx  2 
x

  4  a2 + b2 ≠ 0 then lim f ( x )
4 x
    (A) exists for all values of a and b
l2 = lim sin–1  cos ec  x    ; (B) is zero for a < b
x
   4 
4 (C) is non existent for a > b
1 1
       
l3 = lim tan–1 cot  x    ; 
e a e b
   4  (D) is or if a = b
x
4 mx 2  n for x0

ps
   
l4 = lim cot–1  tan  x    91. Let f (x) =  nx  m for 0  x  1 where m, n ∈ R
   4 
 nx 3  m for
el
x
4 x 1
eh

where [x] denotes greatest integer function then which then which of the following must be correct
of the following limits exist (A) lim f ( x ) exist for all values of m and n.
je

(A) l1 (B) l2 x→0


(B) lim f ( x ) exists only if m = n.
iit

(C) l3 (D) l4 x→0


87. Give a real valued function f such that (C) lim f ( x ) exists for all values of m and n.
@

x→0
 tan 2 x
 2 for x0 (D) lim f ( x ) exists for no values of m and n.
 ( x  [ x ])
2 x→1
 sin m t
f(x) =  x0 2x

1 for
92. lim (m, n, ∈ N) equals
 x →0 x
+
tn
 {x}cot{x} for x0
 (A) 0 if m ≥ n (B) ln 2 if n – m=1
 (C) +∞ if n – m = 1 (D) None of these

 
where, [.] is the integral part and {.} is the fractional part
of x, then 93. If lim x 2  x  1  ax  b  0 , then for k ≥ 2, k ∈
x
(A) lim f(x) = 1
x→0 N which of the following is/are correct ?
(B) lim f(x) = cot 1 (A) 2a + b = 0
x0
(B) a + 2b = 0

  (C) lim sec 2 n  k !b   1


2
(C) cot–1 lim f ( x ) =1 n 
x 0
(D) none of these (D) lim sec 2 n  k !a   1
n 
88. lim f(x) does not exist when
x →c x  cos 2 x
where, [.] is the greatest integral part and {.} is the 94. Let l1 = lim and
x x  sin x
fractional part of x, then
1
(A) f(x) = [ [x] ] – [2x – 1], c = 3 h dx
(B) f(x) = [x] – x, c = 1
l2 = lim
h0  h 2  x 2 . Then
1
(C) f(x) = {x}2 – {–x}2, c = 0 22
tan(sgn x ) (A) both l1 and l2 are less than
(D) f(x) = ,c=0 7
sgn x (B) one of the two limits is rational and other irrational.

FREE BOOKS FOR JEE & NEET =>(@iitjeeadv)


Limits 1.123

(C) l2 > l1 Reason (R) : Let the triangle have side length 1 and
(D) l2 is greater than 3 times of l1. radius of circles be r. Then 2(n – 1) r + 2r 3
95. In which one of the following cases, limit tends to e n (n −1)
1 = 1. There are circles. the area ratio
2
(A) lim x x 1
 n (n  1)
x 1
= which approaches 3π
 1
x 2 3 (n  ( 3  1)) 2 6
(B) lim  1   as n→ ∞.
x   x
x 3 101. Assertion (A) : Let f : (0, ∞) → R be a twice continuously
 x4
(C) lim   differentiable function such that
x   x  2 
|f′′(x) + 2x f′(x) + (x2 + 1) f(x) | ≤ 1 for all x.
1 Then lim f(x) = 0.
(D) lim 1  f ( x )  f ( x ) when lim f ( x ) → 0 x
x  x
Reason (R) : Applying L’Hospital’s rule twice
x2
Assertion (A) and Reason (R)
f ( x )e 2
Code: on the function
x2
we get lim f(x) = 0.
x
(A) Both A and R are true and R is the correct explanation e2
of A.
102. Assertion (A) : A circle C1 is inscribed in an equilateral
(B) Both A and R are true but R is not the correct triangle ABC with side length 2. Then circle C 2 is
explanation of A. inscribed tangent to BC, CA and circle C1. An infinite
ps
(C) A is true, R is false. sequence of such circles is constructed, each tangent to
(D) A is false, R is true. BC, CA and the previous circle. The sum of areas of all the
el

96. Assertion (A) : If [x] denotes the greatest integer function infinitely many circles is .
eh

x 8
then lim does not exist. 1 1
je

x [x] Reason (R) : Radius of C1 is , that of C2 is and


3 3 3
iit

Reason (R) : lim [ x ] does not exist.


x 1
radius of the remaining circle each shrink by a factor .
@

97. Assertion (A) : The value of 3



n 3  e1/ x  1 
 2n (n 2  3n  2)
1 103. Assertion (A): lim[ x ]   where [.] represents greatest
is .
 e 1 
x 0 1/ x
n 1
2
integer function does not exist.
Reason (R) :  e1/ x  1 

n 3 1  2 
1  Reason (R) : lim   does not exist.
x 0  e1/ x  1 
 2n (n 2  3n  2)   2n  n  1  n  2   
n 1 n 1

2 104. Assertion (A) : lim



sin cot 2 x 1
98. Assertion (A) : lim ( x  x  x  x )  3 3 2 3 3 2
x  3 x  2 (  2x ) 2
2
 1 3 3  sin  tan 
Reason (R) : lim  x   x  x 2  = 0 and Reason (R) : lim  1 and lim  1 , where θ
x   3  0  0 
is measured in radians.
 1 
lim  x   3 x 3  x 2  = 0 105. Consider the function f (x) = cos–1[cot x] where [ ]
x   3 
indicates greatest integer function.
99. Assertion (A) : lim x x − x x
x → 0+
( x

) = –1 Assertion (A) : lim f ( x ) exists


x

2
Reason (R) : lim xx (x – 1) = –1
x0 Reason (R) : Both lim f ( x ) and lim f ( x ) are finite.




100. Assertion (A): An equilateral triangle is filled with n, x x
 
2 2
rows of congruent circles. The limit of the ratio of area
of circle to the area of triangle as Comprehension - 1
3π Let f(x) = x[x[x]], x ∈ [–1, 3] where [.] represents the greatest
n → ∞ is . integer function.
6

FREE BOOKS FOR JEE & NEET =>(@iitjeeadv)


1.124 Differential Calculus for JEE Main and Advanced

113. If [ . ] represents greatest integer function, then which


106. If the value of x so that f(x) = 13 is a , a, b ∈ N and of the following, is identical to f of (x) for x ∈ (π, 2π)
b
x
g.c.d. (a, b) = 1 then the value of a + b is : (A) sgn (x) (B)
(A) 14 (B) 18 x
(C) 22 (D) None of these sin x
(C) sinπ[x] (D)
107. The number of points of where limit of f(x) does not sin x
exist is :
114. The limit of function g(x) = sgn f(x) as x tends to 0 is
(A) 3 (B) 4
(C) 5 (D) None of these (A) 0 (B) 1
(C) – 1 (D) does not exist
108. The value of lim f(2sin x) is :

x
2 Comprehension - 4
(A) 2 (B) 4 x
t 2 dt
(C) 8 (D) None  (a  t r )1/ p
Suppose lim 0
  where p ∈ N, p ≥ 2, a > 0, r >
Comprehension - 2 x 0 bx  sin x
Consider a new definition for approximating the values of a 0 and b ≠ 0.
function f(x) in the neighbourhood of a point x = a Approx
x →a 115. If  exists and is non zero then
f(x) = lim f(x) whenever [ | lim f(x) (A) b > 1 (B) 0 < b < 1
x a  x a  (C) b < 0 (D) b = 1
– lim f(x) | ] ≤ 1, otherwise Approx f(x) is said to be
116. If p = 3 and  = 1 then the value of ‘a’ is equal to
ps
x a  x →a
(A) 8 (B) 3
el
non-existent. Here [.] denotes greatest integer function.
(C) 6 (D) 3/2
eh

109. For which of the following functions, Approx f(x) exists :


117. If p = 2 and a = 9, b = 1 and  exists then the value of
x2 −1 2{x} − 4
 is equal to
je

(A) Approx | x − 1 | (B) Approx


[x] − 3
iit

x→1 x→2 (A) 3/2 (B) 2/3


1 (C) 1/3 (D) 7/9
@

(C) Approx 1
(D) None of these
x 0
2  2x Comprehension - 5
[ x ]| x |
x (e  2) Let a1 > a2 > a3 ....... an > 1 ; p1 > p2 > p3..... > pn > 0; such
110. The value of Approx is
x 0 [ x ] | x | that p1 + p2 + p3 + ......+ pn = 1.
(A) –1 (B) 0 Also F(x) = (p1a1x + p2a2x + ....... + pnanx)1/x
(C) 1 (D) None 118. lim F(x) equals
x0
 [x x ]  a, x0
 (A) p1 n a1 + p2 n a2 + ..... + pn n an
111. Let f(x) =   sin x 
t
. The complete set
 lim   , x0 (B) a1p + a p2 + .....a pn
  
1 2


n
t x
of the values of ‘a’ for which Approx exists is
(C) a1 · a 2 + ..... a n
p1 p2 p n
x→0

(A) (0, 2] (B) (–2, 2) n


(C) [–1, 1] (D) None of these (D)  a r pr
r 1
Comprehension - 3
119. lim F(x) equals
Consider the function f(x) =  sin x  x
1
0. Where [ . ] x
x 
(A) n a1 (B) ea 1

represents greatest integer function. (C) a1 (D) an


112. Which of the following function will be periodic lim F(x) equals
120.
(A) f(x) (B) f(|x|) x

  (A) n an (B) ea 1

(C) f(–x) (D) f  | x |  


 2  (C) a1 (D) an

FREE BOOKS FOR JEE & NEET =>(@iitjeeadv)


Limits 1.125

Match the Columns for JEE Advanced


121. Column - I Column - II


1
lim cos2 (π ( n 3 + n 2 + 2n )) where n is an integer, equals.
3
(A) (P)


n 2
1
lim n sin(2π 1 + n ) (n ∈ N) equals.
2
(B) (Q)


n 4
 (n  1) 
lim (–1)n sin (π n 2 + 0.5n + 1 )  sin is (where n ∈ N). (R) π
2n 
(C)



n
x
 xa 
(D) If lim   = e where 'a' is some real constant then the (S) non existent
x   x  a 


value of 'a' is equal to.

122. Column - I Column - II

(A) If lim ( ( x − x − 1) – ax – b) = 0, where a > 0, then there
2
(P) y = –3
x 


exists atleast one a and b for which point (a, 2b) lies on the line

(1 + a 3 ) + 8e1/ x
(B) If lim = 2, then there exists atleast one (Q) 3x–2y–5= 0
+ (1 − b3 ) e1/ x

x→0 1
ps
a and b for which point (a, b3) lies on the line

el

(C) If lim ( ( x  x  1) – ax2 – b) = 0, then there exists


4 2
(R) 15x–2y–13=0
eh

x 

atleast one a and b for which point (a, –2b) lies on the line
je

x7  a7
iit

(D) If lim = 7, where a < 0, then there exists atleast one (S) y = 2
x  a x  a

@

a for which point (a, 2) lies on the line.



123. Column-I Column-II

(A) If f(x) = |x – a| + |x – 10| + |x – a – 10|, where a ∈ (0, 10), (P) 0

then the minimum value of f is (Q) 1


x (1  cos 2 x ) 2  a (sin x  tan x ) 2
(B) lim is equal to (R) 4
tan 5 x  a sin 8 x

x0

n a sin 2 (n !)
(C) lim , 0 < a < 1, n ∈ N, is equal to (S) 10
n 1

n

(D) The number of solutions of the equation (T) depends on a



loga(2a – x) = loga3 x, a > 0, a ≠ 1 is

124. Column - I Column - II


(A) lim ( x  x  x  x ) equals (P) –2

x
sin 2 x  2 tan x
(B) The value of the limit, lim is (Q) –1
ln(1  x 3 )

x0

(C) lim (ln sin3 x – ln(x4 + ex3)) equals (R) 0



x0
(D) Let tan (2π | sin θ | ) = cot (2π | cos θ | ), where θ ∈ R (S) 1

 2 
and f(x) = ( | sin θ | + cos θ | )x. The value of lim   equals

x  f ( x ) 

(Here [ ] represents greatest integer function)



FREE BOOKS FOR JEE & NEET =>(@iitjeeadv)
1.126 Differential Calculus for JEE Main and Advanced

125. Column - I Column - II




1  cos 2 x
(A) lim equals (P) 1
ex  ex  x
2
x 0


1/ x
 (3 / x )  1 
(B) If the value of lim   can be expressed in the (Q) 2
x 0  (3 / x )  1 

form of ep/q, where p and q are relative prime then (p + q) is equal to



tan 3 x  tan x 3
(C) lim equals (R) 4
x5

x0

x  2 sin x
(D) lim (S) 5
x 2  2 sin x  1  sin 2 x  x  1

x0

Review Exercises for JEE Advanced


1. Evaluate the following limits : 8. Evaluate lim (logsinx sin 2x)log sin 2x.
tan(a  2h )  2 tan(a  h )  tan a x0
(i) lim
h 0 h2 9. Evaluate
3
1  tan 1 3x  3 1  sin 1 3x 1  (tan x  sin x )  (tan x  sin x )  ..
(ii) lim
ps
x 0 1 1 lim
1  sin 2 x  1  tan 2x x→0 1  x 3  x 3  ..........
el
2. Evaluate the following limits :
1 − x 02
1  cos x cos 2 x cos 3x
eh

(i) lim 10. Find lim , where


n x1x 2 x 4 .........x n
x 0 sin 2 2 x
je

(sin x  tan x ) 2  x (1  cos 2 x ) 2 1+ xr


iit

(ii) lim xr+1 = .


x→0 8 tan x  5 sin x 8 5
2
@

n n 1 n 2
3. Evaluate the following limits :
1.  r  2. r  3.  r  ....  n.1
1  1  x 2 cos x 11. Evaluate lim 1 1 1
(i) lim 4 n  n4
x→0 tan x
n
12 + 22 + 32 + ... + r 2
(ii) lim
1  x sin x  cos 2 x 12. If Tr =
13 + 23 + 33 + ... + r 3
and Sn = ( −1) r Tr then find∑
x 0 x r =1
tan 2 lim Sn.
2 x

 
4. Evaluate the following limits :
1  x2 x2 x2 x2  13. Evaulate lim ln x 2
1 x2 1  x .
(i) lim 1  cos  cos  cos cos  x
x→0 x8  2 4 2 4  x 1

 x2  x3 + 1
sin sin tan   14. Prove that the graph of f(x) = is asymptotic to the
(ii) lim  2  x
x→0
n cos 3x graph of g(x) = x2 using lim (f(x) – g(x)).
x
5. Evaluate
sin(3x  a )  3 sin(2 x  a )  3 sin( x  a )  sin a   x 1  1  x  
lim 15. Evaluate lim x  tan 1    tan  x  2  
x→0 x 3 x    x  2   
(1 − x )(1 − x ).....(1 − x )
2 2n
6. Evaluate lim (sin x-tan x) 2 + (1-cos 2x) 4 + x 5
[(1 − x )(1 − x 2 )......(1 − x n )]2 16. Evaluate lim
x→1 x →0 7 tan 7 x + sin 6 x + 2 sin 5 x
7. Evaluate 17. Find
n
1   1  1   1 
lim (n  1)  n    n  2  ..........  n  n 1   (A) lim f ( x ) (B) lim f ( x )
  2    
2

n nn 2 2 x →1/ 2 x →3/ 2

FREE BOOKS FOR JEE & NEET =>(@iitjeeadv)


Limits 1.127

| x 1 | 2 1
 x | x  1 | 2  27. Simplify the function y = lim
n  1  n sin 2 x
where f ( x )  2 x
1 28. Simplify the function
x2
x  ( x )  n( x ) sin 2 x
18. Prove that y = lim
n  1  n sin 2 x
1 x  x 2
1 2  2 cos x
lim  lim  log 0.005 . x tan x  2 x  1
x 0 x x  / 4 sin( x   / 4) 29. Show that the function f(x) = does not
x 1
1 11 2 tend to a unique limit as x → ∞, through all real values,
(1 + x )1/ x − e + ex − ex but if x ranges through the sequence of values x = nπ +
2 24 π
19. Evaluate lim (n = 0, 1, 2, ....), then f(x) → 3.
x →0 x3 4
 2 30. The function f(x) and φ(x) are such that f(0) = φ(0) = 0.
20. If λn + 1 = 2   n , λ0 = x , find lim r , for By considering the case where
x 4 x  4
f (x)
r = 1 and 3. f(x) = x2 sin 1/x, φ(x) = tan x, show that lim may
f '( x ) x→0 ϕ( x )
x n f ( x ) + g( x ) exist when lim does not exist.
21. Simplify the function φn (x) = as n → ∞. x→0 ϕ '( x )
x n +1
31. Show that the function
 a sin x   b tan x  f ( x )  n( x ) sin 2 x
22. Evaluate lim  
x   x 
where a, b are positive
x 0  y = lim
n  1  n sin 2 x
integers and [ ] denotes the greatest integer function.
ps
is equal to f(x) when x is an integer, but is equal to φ(x)
x in every other case.
el
23. If f1(x) = + 10 , ∀ x ∈ R and defined by fn(x) =
2 32. Evaluate
eh

1n
f1{fn–1(x)}, ∀ ≥ 2. S then evaluate lim f n ( x ) .  (n 3  13 )(n 3  23 )......(n 3  n 3 ) 
n  lim  
je

1/ x
n  
 n 3n 
 1 a x 1 
iit

24. Evaluate lim  .  where a > 0, a ≠ 1. x3


x  x a  1 
   3x 2  1 1 x
@

33. If lim  2  = k, then find the value of [6 nk],


25. Simplify the function x  3x  1 
 

y = 1 – lim lim {cos(m !x )}2 n
m  n 
 where [.] is greatest integer function.

1 2 x 2  5  3 2 x 3  3x  5
34. If lim = b, then find the value
26. Let f(x) = x 3 (1 − x ) 3 . x 2 3
x  6  5 4x 2  6x  4
f (x) of 230b.
Find m = lim and b = lim (f(x) – mx). Hence cos 4 x  a cos 2 x  b
x x x 35. If lim = a finite quantity, then find
show that lim (f(x) – mx – b) = 0. x 0 x4
x the value of ab + 1

Target Exercises for JEE Advanced


 x x 1  x  1 
x
  1 
1 x
1. Find lim   4. Evaluate: lim x ln  e 1   
x 
 x x 1  x   x  

 ( x 3 − 2ax 2 − a 2 x + 2a 3 ) ( x − 2a ) −1 + 2a ( x + a ) 2  (3  ax )5 / 2  b nx  c sin( x  1)
2. lim  −  5. If lim = 2, then find the
x→a 
 x 3
− a 2
x x − a  x 1

( x  1) 2

1
value of the expression a2+ b2 + c2.
 (a x a
 1)( x  a ) p p  x a 1  1 1  ax 
 6. If the lim 3    exists and has the value
3. Evaluate p 1 
 x0 x  1  x 1  bx 
 ( x  a ) (ln a )a
2 p

1 2 3
where a > 0, p ∈ Q equal to l, then find the value of   .
a l b

FREE BOOKS FOR JEE & NEET =>(@iitjeeadv)
1.128 Differential Calculus for JEE Main and Advanced

1 1 19. Find values of α and β such that


7. Evaluate lim x2 (sin x. sin3 – (x tan3 )sgn (x))
x x x lim (ax 2  2bx  c  x   =0 and prove that
x 
2 1 2 1
n cos n  sin n
8. Find lim
n   2 1
2 2
2 1
n  cos n  n sin n 

lim x
x 
 (ax 2  2bx  c  x     ac  b 2
2a a
 2 2 
20. Show that the functions y = 1 – x – [x] – [1–x] and y
9. If an + 1 = (an + x)–1, a0 = 0, and bn + 1 = (bn + y)–1, b0 = 0,
= 1 – x – lim (cos 2 n 1 2x ) are identical.
a n  bn n 
find lim for n = 2, 3.
xy x y
2 2
21. If f(x + y) = f(x) + f(y) for all x, y ∈ R and f(1) = 1,
π
π 4 1/ x2 2f (tan x )  2f (sin x )
+x e
4 then find lim
∫ cos t dt − ∫ cos tdt
n n
0
4 x 0 x 2 f (sin x )
10. Evaluate lim 0
,n∈ N n n
1 C
 n k (k k 3)  e  2
x→0
x 4e x
2
22. Show that lim
n 
k 0
n x  x  23. Calculate the limit of the sequence {xn} if it is defined
 n  
(1  x ) 2  1 x  by the recursive relation :
11. Evaluate lim
x0 n (1  x 2 )  2 x 1 a 
xn + 1 =  x n   ∀n ≥ 1, a > 0,
12. Let g(x) = f(x – 1) + f(x + 1) where 2 xn 
ps
where x1 is an arbitrary negative number.
1 | x |, | x | 1
el
f(x) =  find lim g(sin x). 24. Show that if –1 < x < 1, then
| x | , | x |  1 
eh

x m(m  1)....(m  n  1) n  m  n
2 un = x   x
n! n
je

 3 1
n  r r r   tends to zero as n → ∞ .
 cot
iit

1
13. Prove that lim   25. If cn, sn denote the sums of the first n terms of the series
n 
r 1  2

2
@

  1
+ cosθ + cos2θ +...., and
(1  cos x )(e x  cos x ) 2
14. Evaluate lim . sin θ + sin 2θ + sin 3θ .......
x 0 xn
and θ is not a multiple of 2π, then prove that
15. If α, β are two distinct real roots of the equation ax3 + lim (c1 + c2 + ....+cn)/n = 0,
x – 1 – a = 0, (a ≠ –1, 0), which are not equal to unity n
then prove that 1 θ
lim (s1 + s2 + .... + sn)/n = cot
n 2 2
ln((1  a ) x 3  x 2  1  a )  
lim is a  1  . 26. Draw a graph of the function y defined by the equation
x1/  (e1x  1)( x  1)  
1
x 2 n sin x  x 2
 −1 
4 1 − 3x + x 2  x4 −1  y = lim 2
16. lim  −  +3 n  x 2n  1
x →1   x 2 − x −1 1 − x3  x 3 − x −1 
 
27. Show that the graph of the function
17. If 2f(sin x) + 2 f(cos x) = tan x , then evaluate lim x n φ ( x ) + x − n Ψ( x )
x→−1 y = lim
n →∞ xn + x −n
 1  2 −1
1− x f(x) if f   = . is composed of parts of the graphs of φ(x) and Ψ(x),
 2 2 together with (as a rule) two isolated points.
18. Find 28. Prove that the function y which is equal to 0 when x is
 −1 
−1
rational and to 1 when x is irrational, may be represented
  a + x  2 4 ax 
lim    − 3/ 4 1/ 4 1/ 2 1/ 2 1/ 4
x →a   4 a − 4 x 
in the form
 x − a x + a x − a 3/ 4  y = lim sgn{sin2(m!πx)}
 
m 2
−( 2 )log 4
a 8
} where sgn x = lim arctan(nx ) .
m  

FREE BOOKS FOR JEE & NEET =>(@iitjeeadv)
Limits 1.129

29. Evaluate 32. If f(x) = lim {sinx + 2sin2x + 3sin3x +....nsinnx} then

cos 2 (1  cos 2 (1  cos 2 (...cos 2 ( x )))...) n
lim
x 0  ( x  4)  2 
1



 evaluate lim {(1  sin x ) 2 f ( x )}(sin x 1)
x x  / 2
 
30. Let the sequence xn be defined as 33. The function un attains the values
x0 = a 1 1 1 1
u1 = ;u = + + ,
x1  a  a 2 2 2 2.4 2.4.6

1 1 1
x2  a  a  a un = +
2.4
+ .... ,.....
2 2.4....(2n )

x3  a  a  a  a Prove that lim un < 2.


n

..................................., find lim x n . 1k  2k  ....n k 1


n  34. Prove that, if k > 0, lim k 1
 .
1 1 1 n  n k 1
[ x ]  [2 x ]  [3x ]  .....  [nx ]
2 3 n F(n )
31. Evaluate lim Hence evaluate lim , where F(n) is the
n  12  22  32  ......n 2 n  (n 1) 2 k  2
where [.] denotes the greatest integer function
ps coefficient of xn–2 in (x – 1k) (x – 2k) .... (x – nk).
el

Previous Year's Questions (JEE Advanced)


eh
je

A. Fill in the blanks


iit

x2

1. If f(x) = sin x, x ≠ nπ, n = 0, ± 1, ± 2, ± 3, 0 cos t 2 dt


[IIT - 1997]
@

6. lim = ..................


............. x 0 x sin x
= 2, otherwise
ln(1  2h )  2 ln(1  h )

and g(x) = x2 + 1, x ≠ 0, 2 7. lim = ......... [IIT - 1997]
h2

h 0
= 4, x = 0
B. True / False

= 5, x = 2
[IIT - 1986]

then lim g[f(x)] is............. 8. If lim [f(x)g(x)] exists then both lim f(x) and

x→0 x →a
x →a
 4 1 2  lim g(x) exist [IIT - 1981]
 x sin  x   x 

x →a
2. lim     = ....... [IIT - 1987]

x  (1 | x |3 ) 
  C. Multiple Choice Questions with
  ONE correct answer
3. ABC is an isosceles triangle inscribed in a circle of radius
r. IF AB = AC and h is the altitude from A to BC then x  sin x
9. If f(x)= , then lim f(x) is
the triangle ABC has perimeter x  cos 2 x x

P = 2( 2hr − h
2
+ 2 hr ) and area (A) 0 (B) ∞
(C) 1 (D) None
A = ...... also lim
A
= ....... [IIT - 1989] [IIT - 1979]
h →0 P3  

1 2 n
x4
10. lim    ....   is equal to
 x6 n  1  n 2 1 n2 1 n2 
4. lim   = .................. [IIT - 1990]
x   x  1 

1
(A) 0 (B) –
1/ x 2 2
 
lim  1  5x 2 
2
1
5. = .................. [IIT - 1996] (C) (D) none of these
 

x→0
 1  3x 
2 [IIT - 1984]

FREE BOOKS FOR JEE & NEET =>(@iitjeeadv)


1.130 Differential Calculus for JEE Main and Advanced

sin[ x ] ((a − n )nx − tan x ) sin nx


11. If f(x) = , [x] ≠ 0 20. If lim = 0, where n is non
[x] x→0 x2
= 0, [x] = 0 zero real number, then a is equal to
where [x] denotes the greatest integer less than or equal n +1
to x, then lim f(x) equals (A) 0 (B)
x→0 n
(A) 1 (B) 0 1
(C) n (D) n +
(C) –1 (D) none of these n
[IIT - 1985] [IIT - 2003]
n 21. The value of lim ((sin x) 1/x
+ (1 + x) sin x
), where x > 0 is
12. lim x = 0 for x→0
x ex (A) 0 (B) –1
(A) No value of n (C) 1 (D) 2 [IIT - 2006]


(B) n is any whole number sec 2 x


(C) n = 0 only f ( t )dt
(D) n = 2 only [IIT - 1992] 2

22. lim equals
13. lim   equals 2
x 
x

x→0  tan 1 2 x  4 x  2
16
(A) 0 (B) 1/2 (A) 8 f(2) (B) 2 f(2)
(C) (D) ∞ [IIT - 1992] π π


1/ x
14. lim    (C) 2 f(1/2) (D) 4f(2) [IIT - 2007]
ps
x→0 
tan   x   equals


π
  4 
el
x
1 t ln(1  t )
(A) 1 (B) – 1 23. The value of lim  dt is
eh

(C) e2 (D) e x 0 x 3
0 t4  4
[IIT - 1993]
je

1
1 2n r (A) 0 (B)
15. lim  12
iit

equals
n  n
r 1 n  r 1
2 2
(C) (D) 1 [IIT - 2010]
@

24

64
(A) 1 + 5 (B) –1 + 5
1
(C) –1 + 2 (D) 1 + 2 24. If lim 1  x ln (1  b 2 )  x = 2bsin2θ, b > 0 and
[IIT - 1997] x   
x tan 2 x − 2 x tan x θ ∈ (–π, π), then the value of θ is
16. lim is equal to
x→0 (1 − cos 2 x ) 2 π
π
(A) 2 (B) – 2 (A) ± (B) ±
4 3
1 1
(C) (D) – [IIT - 1999]
π π

2 2
(C) ± (D) ± [IIT - 2011]

x 6 2
 x 3 
17. For x ∈ R, lim   equals
x  x  2   x2  x 1 
25. If lim   ax  b   4 then,
(A) e (B) e x   x  1 
 
(C) e–5 (D) e5 [IIT - 2000]

(A) a = 1, b = 4 (B) a = 1, b = – 4
sin(π cos x ) 2
(C) a = 2, b = –3 (D) a = 2, b = 3
18. lim equals
x→0 x2 [IIT - 2012]
(A) – π (B) π
(C) π/2 (D) 1 [IIT - 2001] D. Integer Answer Type Questions

19. The integer n for which 26. Let m and n be two positive integers greater than 1.
(cos x − 1)(cos x − e x )  ecos( a ) − e   e
n

lim is a finite nonzero number is m


x→0 xn
If lim   = −  2  then the value of is
a →0  a m
 n
(A) 1 (B) 2
(C) 3 (D) 4 [IIT - 2002] [2015]


FREE BOOKS FOR JEE & NEET =>(@iitjeeadv)
Limits 1.131

27. Let a, b ∈ R be such that 1 − x (1 + |1 − x |)  1 


31. Let f(x) = cos  , for x π 1
x 2 sin bx |1 − x |  1 − x 
lim = 1. Then 6(a + b) equals [2016]
x → 0 ax − sin x

then


E. M ultiple C hoice Q uestions with ONE (A) lim f ( x ) does not exist
x →1−
or MORE THAN ONE correct answer
(B) lim f ( x ) = 0
x →1−
1
(1 − cos 2 x) (C) lim f ( x ) = 0
2 x →1+
28. The value of lim
x → 0− x
(D) lim f ( x ) does not exist [2017]
x →1+
(A) 1 (B) –1
(C) 0 (D) none of these
[IIT - 1991] F. Subjective Problems
1 − cos 2(x − 1) a  2 x  3x
29. lim 32. Evaluate lim , a  0
x →1 x −1 x a 3a  x  2 3x
[IIT - 1978]
(A) exist and it equals 2
2 33. Evaluate : lim
(a + h )2 sin (a + h ) − a 2 sin a
(B) exists and it equals –
ps
h →0 h
(C) does not exist
[IIT - 1980]
el

(D) none of these [IIT - 1998]


eh

35. Use the formula
x2
a− a −x − 2 2
ax − 1 2x − 1
je

4 lim = loge a to find lim [1982]


30. Let L = lim , a > 0. If L is finite, then x → 0 (1 + x )1/ 2 − 1
iit

x →0

x
x →0 x4
@

 n  1 cos1    n.
(A) a = 2 (B) a = 1 2 1
35. Find the value of lim
1 1 x   n
(C) L =
64
(D) L =
32 [IIT - 2004]

[IIT - 2009]

Previous Year's Questions (JEE Main Papers)

1. lim
(1 - cos 2 x ) (3 + cos x )
is equal to tan( x - 2) { x 2 + (k - 2 ) x - 2 x}
x Æ0
3. If lim = 5, then k
x tan 4 x x Æ2 x2 - 4 x + 4
(a) 1/2 (b) 1 is equal to

(a) 0 (b) 1
(c) 2 (d) –1/4

(c) 2 (d) 3 [2014, online]

[2013]

sin (p cos x ) 2 2
e x - cos x
2. lim 2
is equal to 4. If lim is equal to
x Æ0 x x Æ0 sin 2 x
(a) p/2 (b) 1 3

(a) 3 (b)
(d) p

(c) –p [2014] 2

FREE BOOKS FOR JEE & NEET =>(@iitjeeadv)
1.132 Differential Calculus for JEE Main and Advanced

5 cot x - cos x
(c) (d) 2 [2015] 11. lim equals
(p - 2 x)3

x Æp / 2


4
(1 - cos 2 x )(3 + cos x ) (a) 1/4 (b) 1/24
5. If lim is equal to


x Æ0 x tan 4 x (c) 1/16 (d) 1/8 [2017]


(a) 4 (b) 3 12. For each t ŒR, let [t] be the greatest integer less than


1 or equal to t. Then
(c) 2 (d) [2015]
Ê È1˘ È2˘ È15 ˘ˆ

2
lim x Á Í ˙ + Í ˙ º + Í ˙˜
xÆ0 + ËÎ x˚ Î x˚ Î x ˚¯


1
6. Let p = lim 1 + tan 2
xÆ0+
( x ) 2x , then log p is equal to (a) is equal to 0


(b) is equal to 15


(a) 2 (b) 1 (c) is equal to 120


1 1 (d) does not exist (in R) [2018]


(c) (d) [2016]


2 4 x tan 2 x - 2 x tan x
13. lim equals
xÆ0 (1 - cos 2 x)2

2x
Ê a 4ˆ
7. If lim Á1 + - 2 ˜ = e3. Then a is equal to
x Æ• Ë x x ¯ (a) –1/2 (b) 1/4

3
ps
(a) 2 (b) (c) 1/2 (d) 1 [2018]


2
el
1 2 (27 + x)1/ 3 - 3
(c) (d) [2016, online] 14. lim equals
eh

xÆ0 9 - (27 + x)2 / 3





2 3
je

1 1
(1 - cos 2 x )2 (a) - (b)
iit

8. L = lim is

x Æ 0 2 x tan x - x tan 2 x
3 6
@

1 1 1
(a) 2 (b) - (c) - (d) [2018]

6 3

2

(c) –2 (d)
1
[2016, online] 1 + 1 + y4 - 2
15. lim


2 yÆ0 y4

3x - 3 1
9. lim is equal to (a) exists and equals
2x - 4 - 2

x Æ3 4 2
(b) does not exist

(a) 3 (b) 1/ 2 1

(c) exists and equals

3 1 2 2
(c) (d) [2017, online]
1

2 2 2
( )
(d) exists and equal [2019]
2 +1

2 2
1a + 2a +  + n a
10. If lim = =
nƕ
[n + 1]a - 1[(na + 1) + (na + 2) +  + (na + n)] 16. For each x ŒR, let [x] be the greatest integer less than

1 or equal to x. then
for some positive real number a, then a is equal to
60
x([ x ] + | x |) sin [ x ]
(a) 7 (b) 8 lim is equal to
x Æ0 -

|x|

15 17 (a) –sin 1 (b) 0
(c) (d) [2017, online]


2 2 (c) 1 (d) sin 1 [2019]

FREE BOOKS FOR JEE & NEET =>(@iitjeeadv)
Limits 1.133

17. For each t ŒR, Let [t] be the greatest integer less than x cot (4 x )

or equal to t, then 19. lim is equal to
x Æ0 sin 2 x cot 2 (2 x )


Êp ˆ (a) 2 (b) 0
(1 - | x | + sin |1 - x |) sin Á [1 - x ]˜


Ë2 ¯ (c) 4 (d) 1 [2019]


lim
x Æ1+ |1 - x | [1 - x ] cot 3 x - tan x

20. lim is
cos ( x + p / 4)


x Æp / 4
(a) equals –1 (b) equals 1

(c) does not exists (d) equals 0 [2019] (a) 4 (b) 8 2



(d) 8 (d) 4 2 [2019]


18. Let [x] denote the greatest integer less than or equal to

x. Then p - 2 sin -1 x
21. lim is equal to
x Æ1- 1- x


tan (p sin x ) + (| x| - sin ( x[ x ]))
2 2
lim 1 p
x Æ0 x2

(a) (b)
2p 2
(a) equal p (b) equals 0

2
(c) equals p + 1 (d) does not exist (c) (d) p [2019]
p

ps
ANSWERS
el
eh
je
iit

Practice Problems—A Concept Problems—C


@

1. (i) 1 (ii) does not exist 1. 1


2. (i) 0 (ii) does not exist 2. L.H.L. = 1, R.H.L. = 2, lim f(x) does not exist.
x →1
(iii) –1 3. (i) lim f(x) = does not exist
3. No, we donot conclude anything. x→0

4. No, no, we donot conclude anything. (ii) lim f(x) = 0,


x→1
5. (a) No (b) No
(iii) lim f(x) = 1
(c) Yes (d) 0 x→3
7. π/2 8. 0 4. (a) 1 (b) 1
9. does not exist (c) 1 (d) 2
5. (a) 2 (b) 2
Concept Problems—A (c) 1 (d) 2
1. (i) – ∞ (ii) –∞ 6. (i) does not exist, 0 (ii) 0
(iii) 1 (iv) 1 
2. (i) does not exist (ii) –5 7. (i) 0 (ii) 
2
(iii) 4 (iii) 1/2 (iv) does not exist
3. ∞ 4. does not exist. 8. limit does not exist since function is undefined in the
5. 0 6. 0 neighbourhood.
7. does not exist 8. 0 9. 0 11. 0 , 1

12. does not exist 13. 8
Concept Problems—B 14. (a) 3 (b) does not exist
2. –1 3. 1/π (c) does not exist

4. 0 15. –3
6. since limit of g(x) does not exist. 16. lim f(x) is not known but lim f(x) = –7.
x  2 x 2

FREE BOOKS FOR JEE & NEET =>(@iitjeeadv)


1.134 Differential Calculus for JEE Main and Advanced

Practice Problems—B Concept Problems—F


17. (i) –1/3 (ii) 1 1. 2
18. (i) does not exist (ii) 0 2
π
2. (i) − 5 (ii)
3
19. (i) (ii) 0 1
2 (iii) (iv) 1
20. 0 21. –2 3 2
22. 0 23. does not exist.
24. 1/2, 0 3. –1/4 4. 1


25. (i) ∞, ∞ (ii) ∞,0 5. a = 2, b = 1
(iii) 1/2, 0 (iv) 0, 0
Practice Problems—E
26. (i) –2, 2 (ii) 1, 4
(iii) a2, a2– 1 (iv) 2, 5 6. loga6
27. (i) 2, 2 (ii) 10n –1, 10n 1
7. (i) 7 /4 (ii)
1 1 8 3
(iii) , – (iii) 32 (iv) 1.2
2 2
Concept Problems—D 8. –
1
9.
34
0 3 23
1. (i) determinate form (ii) form 10. (i) 2/3 (ii) 1/6
0
(iii) (0 × ∞) form (iv) (∞ – ∞) form (iii) 9/25
(v) (0)º form (vi) (∞)º form
(vii) (1)∞ form Concept Problems—G
ps
(viii) determinate form
5. (i) 0 (ii) 0 1. (i) –1/9 (ii) – 2/3
el
(iii) 0 (iv) 0 (iii) 0 (iv) 2
eh

7. 7 2. (i) 1/3 (ii) 24


8. (i) 4 (ii) –2 (iii) 1/3 (iv) – 1/2
je

2
Practice Problems–C 3. 4. 1/2
iit

5
9. 0 if a < 0, dne if a ≥ 0 5. limit does not exist.
@

10. (a) 0 (b) ∞


(c) 0 (d) – ∞ Practice Problems—F
(e) ∞ (f) – ∞ 7. (i) –1 (ii)0
11. ∞ 12. does not exist (iii) ∞ (iv)–1
8. (i) a/m–b/n (ii)a/m + b/n
Concept Problems—E (iii) –19/3 (iv)∞
15 9. (i) 1/80 (ii)–1/5040
1. (i) (ii) – ∞ (limit dne)
11 (iii) 1/2 (iv)– 11/6
(iii) 2x (iv) ∞ (limit dne) 10. 13/60 11. 1/2
2. (i) 0 (ii) ∞ (limit dne) 1 1 9
3. (a) 0 (b) 3/4 12. − , 1, 13. –3,
2 24 2
(c) dne (d) 2/3
4. ∞ (limit dne). 1 3 1
14. 72, a = ,b= ,l= −
5. f (5+) = 1 , f (5–) = 0 ; limit dne 4 4 32
6. (i) 1 (ii) – ∞ (limit dne) Concept Problems—H

(iii) 0 (iv) 1
π
Practice Problems—D 1. (i)
180
(ii) 0
7. (i) 1 (ii) –1/4 2. (i) 2/5 (ii) 8/3
(iii) 1 (iv) –7.2 (iii) – 3/2 (iv) 25/6
1 n
8. (i) (ii) –3 3. (i) 4 (ii)
4 m
(iii) 3/4 9. c = – 6 limit = 4.
8
10. (i) 2, 4 (ii) 0, –1 (iii) log2e (iv)
7

FREE BOOKS FOR JEE & NEET =>(@iitjeeadv)


Limits 1.135

1 1 1
4. (i) (ii) 1 31. (i) (ii)
ln 3 2 2π 2

1 1 a
5. (i) − (ii) ln 32. (i) 5 2 (ii) 1
m n b 33. (i) –2 (ii) ∞
2 (iii) –1/12 (iv) 3/2
6. (i) (ii) –1

3
Concept Problems—I
(iii) 4 (iv) 4/5 1. (i) 0 (ii) 8


3 (iii) 2 (iv)
7. (i) No (ii) No 2. (i) 25 (ii) 1

(iii) No (iv) 0 (iii) 1 (iv) 9

8. (i) 1 (ii) –2 3. (i) 3 (ii) 0

(iii) 3/4 (iv) 1/8 
(iii) 

(iv) 1
9. (i) 1/2 (ii) 25/9 2

(iii) 2 (iv) 3 5. (i) 0 (ii) –3

10. (i) ln(8/7)/ ln(6/5) (ii) 9/4 (iii) 1 (iv) – 1/3

(iii) (ln 5 .ln 4)/(ln 3 .ln 6) (iv) 1/3 6. 0
11. (i) ∞ (ii) 1/2
Practice Problems—H

12. 1/2
ps
13. (i) − 2 / 4 (ii) 1 2 , − 2

7.
el
1 4
(iii) − (iv) ∞ 4

eh

4
 0, if m  n
2 1
14. (i) (ii)  3, if m  n
je

π 

e 8. lim it  
  n and n  m odd
iit

(iii) – ln 2 (iv) 5  , if m

, if m  n and n  m even
@

Practice Problems—G
17. – 2 18. tan10–10 2 1
9. (i) (ii)
19. 0, if n > m, 1 if n = m, ∞ if n < m. 3 24
20. – sin a (iii) 510/325 (iv) ∞
1
21. (i) a (ii) 324 10. 5 11. 2
12. (i) ∞ (ii) ∞
(iii) – 6 (iv) 1 (iii) 0 (iv) 0
22. (i) 3 (ii) 2/3 13. (i) 1 (ii) 0
(iii) 2/3
(iii) does not exist (iv) 0
23. (i) 3/5 (ii) 3 ln 3
(iii) 16 14. 100 15. 1

16. –2
24. 0 for n = 1; 4.9 for n = 2
25. (i) –1/3 (ii) ln 4 Concept Problems—J
sin 2β a 1. (i) x = –2, y = 2 (ii) y = 2
26. (i) (ii) 
2β 
Y

27. (i) –∞, 1 , No (ii) − 2 , 2 , No


28. 1 1
1 2. (i)
29. (i) 2 (ii)
2 –2 –1 1 2 X
(iii) aa ln ea
30. (i) 1/3 (ii) 1

FREE BOOKS FOR JEE & NEET =>(@iitjeeadv)


1.136 Differential Calculus for JEE Main and Advanced

Y Practice Problems—I
1
6. (i) (ii) 3
5
1
(ii) (iii) 0 (iv) 1
–2 2 X 4
7. 1 8. (i)


3
1 1
(ii) (iii)
Y 2 4
9. log 22 x 10. 3
y= 21
x –x 1
1 11. (i) − (ii) –1
3. (i). 2
1
2
2 X
sin x
Asymptote x=0 Asymptote x=1 (iii)
x
Concept Problems—L
–4

1. (i) 0 (ii) 2a / π
Y
(iii) a/π (iv) 0
y= 41 2 2. (i) 0 (ii) 0
x –x
ps
(iii) 0
1
3. (i) – 1/2 (ii) 0
el
1 2 X (iii) 0 (iv) 1/2
(ii) –1 2
eh

Asymptote x = –1 Asymptote x = 1 4. (i) 0 (ii) 0


(iii) − 2 (iv) 0
je

–4
5. (i) ∞ (ii) 2/3
iit

(iii) – 3/5 (iv) (a – c)/2


@

6. Both are wrong. The limit is –13.


Y 1 1
7. (i) (ii) −
Asymptote y = 1 2 2
(iii) y= x(x–1)
2 3
x +1 (iii) – (iv) 0
2
–2 –1 0 1 2 3 X 8. (i) ∞ (ii) 4/3
Y (iii) 2
9. (i) ln a
(ii) ∞ for n > 1 ; a for n = 1 ; 0 for n < 1
4. (i) e–1 (iii) a (iv) 0
1 2 X Practice Problems—J
Y 11. (i) 2/9 (ii) − 5
2
(iii) − 1 (iv) ∞
(ii) 2
12. (i) 1/2 (ii) 2
1 X
π2 a+b+c
(iii) (iv)
Concept Problems—K 8 3
1. (i) 5 (ii) 8/3 1
13. 1, 14. 2
(iii) ∞ (iv) 4 2
2. (i) 1/2 (ii) 1/3 15. a = 1, b = –1. 16. 5
(iii) 5 (iv) 1/2 4

FREE BOOKS FOR JEE & NEET =>(@iitjeeadv)


Limits 1.137

1 Concept Problems—O


17. (i) –2 (ii) ln  
2 1. 5 2. limit is 1
π 2
(iii) (iv) –π /4 3. limit is 1 at x = –1 and 1; limit is 0 at x = 0.
4
1 1
1 5. g(x) = − , h(x) = , limit = 0
18. (i) (ii) 1 x2 x2
4 2
6. b
1
(iii) −
4 Practice Problems—M
19. (i) 0 (ii) 1/2 8. 0 9. 4


(iii) 1/120 10. (i) 0 (ii) 0
(iii) 2 (iv) dne
Concept Problems—M 1
2 11.
1. 2. 0 2
3 13. (i) 1 (ii) 0
3. 0 4. 0 (iii) 0 (iv) 0

1 x
5. 5 6. 14. 15. 1
e
3
Practice Problems—K 16. they have same limits as before.
1 1
ps
7. 8. 17. 0 18. 1

4 4 19. 1
el
9. e2 10. 1
12. eπ Concept Problems—P
eh

11. 0
13. 1 14. 99 1
1. (i) (lna)2 (ii) 1
je

2
Concept Problems—N
iit

(iii) 1
2. (i) e–1 (ii) e
@

2. (i) 1 (ii) 1
2
(iii) − (iv) e3 a
e 3 (iii) 2 (iv) ln
b
1 e 3. (i) 0 (ii) 2/3
3. (i) (ii)
e (iii) –2
(iii) ae (iv) 1 5. 1/2
4. (i) e5 (ii) 1 6. (i) 2/3 (ii) π2/4
(iii) ∞ if n < 1; e if n = 1; 1 if n > 1.
(iv) 1/e (iii) 5
Practice Problems—L 7. (i) 1/3 (ii) 1/e
(iii) 1/2
m2 n
5. (i) − (ii) e1/6
e 2 Practice Problems—N
(iii) e − x / 2 2
(iv) n! 8. (i) – 9 (ii) 2/3
6. 0 if 0 < a < 1; lna if a > 1. 1
7. (i) 1 (ii) e–1/2 9. (i) 4 (ii) –
2
(iii) 1 (iv) e–1/3
14. (i) 1/18 (ii) 2/3
8. (i) e–1/2 (ii) e1/2
e 16. esin 2
y 17. 0
(iii) e1 (iv)
e1− e
n −1
9. (i) 1 (ii)
e 2a
Concept Problems—Q
–2 2α
(iii) e (iv) e e 1
3. 4. 1
10. λ = 1, µ ∈ R e −1 3

FREE BOOKS FOR JEE & NEET =>(@iitjeeadv)


1.138 Differential Calculus for JEE Main and Advanced

Practice Problems—O 19. A 20. B 21. B


22. B 23. C 24. B
 ab
7. lim ACn  8. 25. A 26. B 27. D
n  3 2 28. D 29. C 30. B
Ct 31. A 32. C 33. B
9. cos t 34. D 35. B 36. B
2
n
37. A 38. D 39. A
10. Q(t n )  Q0 1  kt  , where k is the proportionality factor 40. A 41. B 42. A
n   43. D 44. C 45. B
(law of compound interest) ; Qt = Q0ekt. 46. B 47. D 48. C
49. B 50. D 51. D
Concept Problems—R 52. D 53. B 54. A
1. If x ≠ kπ (k∈Ι), then cos2x < 1 and y = 0 ; but if x = kπ, 55. B 56. D 57. B
58. C 59. A 60. A
then cos2 x = 1 and y = 1.
61. C 62. B 63. B
2. (i) y = 1 when 0 ≤ x ≤ 1; y = x when 1 < x < ∞. 64. D 65. B 66. D
(ii) x > 1 67. A 68. A 69. B
(iii) y = |x|
π 70. A 71. C 72. C
(iv) y = π when x < 0; y = when x = 0; y = 0 when 73. B 74. A 75. B
2
x > 0. 76. ABD 77. BD 78. ABD
3. 2 79. CD 80. ABCD 81. ABCD
82. ABCD 83. A 84. A
ps
Practice Problems—P 85. ABCD 86. AB 87. BC
88. BC 89. ABD 90. BCD
el

sin x x0 91. BC 92. ABC 93. BCD


eh

4. f(x) = lim  0 ; lim = π


; x  −
94. ABCD 95. ABD 96. D
0 x→ 0
97. A 98. A 99. B
je

tan x 2
x0 100. C 101. A 102. D
x2
iit

103. B 104. D 105. D


; limit does not exist 106. B 107. A 108. A
@

109. C 110. A 111. B


5. (a) (b) 112. D 113. C 114. A
115. D 116. A 117. B
118. C 119. C 120. D
121. (A)–(Q) ; (B)–(R) ; (C)–(P) ; (D)–(P)
a c
6. 0 7. d f  122. (A), (Q) ; (B)–(P) ; (C)–(R); (D), (S)

  123. (A)–(S) ; (B)–(R) ; (C)–(P) ; (D)–(Q)
8. x ∈ [0, sin 1) 124. (A)–(S) ; (B)–(P) ; (C)–(Q) ; (D)–(R)
9. φ(x) = 1, (x > 0); φ(x) = 0, (x = 0; φ(x) = –1, (x < 0). 125. (A)–(R) ; (B)–(S) ; (C)–(P) ; (D)–(Q)
10. ln 5 11. 40
REVIEW EXERCISES for JEE ADVANCED

14
12. 13. 1/ π
3
2 sin a
14. 4 15. 1 1. (i) (ii) –1
cos3 a

e
16. 8 17. 1 2. (i) 7/4 (ii) − 4
5
3. (i) 1/3 (ii) 6
OBJECTIVE EXERCISES 1 1
4. (i) (ii) −
1. A 2. A 3. B 28 9
4. C 5. B 6. B 2n !
5. –cos a 6.
7. C 8. B 9. A (n !) 2
10. C 11. A 12. A
13. D 14. A 15. B 7. e2 8. 2
16. C 17. B 18. B 9. (1/2) 10. cos–1x0

FREE BOOKS FOR JEE & NEET =>(@iitjeeadv)


Limits 1.139

1 2 24. If m is a positive integer, u n = 0 for n > m.


11. 12. –
u mn

24 3
Otherwise n 1  x   x , unless x = 0
1 un n 1
13. – 1 15.
2 1, n  I ψ( x ), n ∈ I
16. 1/2 25.  26. 
2 0, n  I  φ( x ), n ∉ I
17. (a) − 3 2 (b) 5
2 3 28. If x is rational then sin 2 (m!nx), and therefore
sgn{sin2(m!πx)}, is equal to zero from a certain value
−7e
19. 20. 4–2, 4–4 of m onwards; if x is irrational then sin 2(m!πx) is
16 always positive, and so sgn {sin2(m!πx)} is always
1 equal to 1
21. φ(x) = f(x), (|x| > 1); φ(x) = g(x), (|x| < 1); φ(x) =
{f(x)
30. 1 + 1 + 4a
2
+ g(x)}, (x = 1); and φ(x) is undefined when x = –1. 29. 4
π


2
22. a+b–1
31. 0 32. e
23. 20
–2
24. 1 if 0 < a < 1; a if a > 1 34. (k + 1)

PREVIOUS YEAR'S QUESTIONS


25. The function is equal to 0 when x is rational and to 1
when x is irrational.
27. The function is equal to 0 except when x is an integer, (JEE ADVANCED)
ps
and then it is equal to 1.
2 1
el
26. m = 1, b = – 1. 1 2. –1 3. 4. e5
3 128r
eh

28. The function is equal to φ(x) unless x is an integer, and 5. e2 6. 1 7. –1 8. F


then it is equal to Ψ(x).
je

9. C 10. B 11. D 12. B


30. The former limit is 0, the second does not exist. 13. B 14. C 15. B 16. C
iit

 17. C 18. B 19. C 20. D


3
32. 4 · e 33. 4
@

3 21. C 22. A 23. B 24. D


25. B 26. 7 27. 2 28. B
34. 400 35. 256
2

29. C 30. AC 31. B, D 32.
TARGET EXERCISES for JEE ADVANCED 33. a2 cos a + 2a sin a 34. 2 ln 2
3 3

1. 1 2
35. 1 –
2. –1/a for a ≠ 0, for a = 0 there is no limit. π
p-1+ alna 3
3. 4.
e 2a 2 QUESTIONS FROM PREVIOUS
YEAR'S (AIEEE/JEE MAIN PAPERS)
5. 41 6. –24

7. –1 8. does not exist

10. If n is even, limit does not exist. If n is odd limit is 0)
11. ∞ 12. 3 1. (c) 2. (d) 3. (d) 4. (b)
 0 if n3 5. (c) 6. (c) 7. (a) 8. (a)

14. 1 / 2 if n3 16. 3 9. (b) 10. (a) 11. (c) 12. (c)
  if n3
 13. (c) 14. (c) 15. (a) 16. (a)
1 17. (d) 18. (d) 19. (d) 20. (c)
17. 18. a2
2 21. (c)

21. 1 ln 2 23. − a
2

FREE BOOKS FOR JEE & NEET =>(@iitjeeadv)

You might also like